Graphic Era Hill University, Dehradun

Graphic Era Hill University, Dehradun
Introduction :
Reasoning plays an important role in a candidate’s performance in the aptitude
tests including campus placement and therefore needs to be practiced and
perfected. Reasoning, in the exams can be primarily divided into 2 broad categories
Verbal and Non Verbal. It has been observed that in the exams, the Verbal
questions have a higher weightage in the overall composition. This book will
primarily focus on all the categories under the Verbal part and some categories
under Non Verbal Reasoning.
The broad topics covered include Arrangement problems which can be Deterministic
and Non Deterministic, Blood Relations, Grouping or Condition based problems,
Coding-Decoding, Direction Sense, Series Completion, Mathematical Reasoning ,
Miscellaneous problems and puzzles.
Our effort through this book will be to understand each of these types of problems,
their variety, and practice them in order to improve our performance in the
Reasoning Section. The main concepts in every topic have been discussed
alongwith relevant examples. At the end of every topic, a basic level practice
exercise has been provided . Additionally, advanced level practice exercises (2
exercises for each topic) have been provided towards the end.
Students are advised and encouraged to do as many varied types of problems in
practice in order to extend the range of problems that can be handled by them
effectively.
Page 1 of 204
Graphic Era Hill University, Dehradun
Arrangement Based Problems
Concepts: There are few aspects to be learnt before we can start doing the
reasoning problems on Arrangement. Let us try to understand them.
1.
An arrangement problem can be Linear, Circular, Horizontal or any other
(across a Rectangular Table etc).
2.
In a Linear Arrangement problem, the items can be arranged either facing
you or away from you. One can take either of the 2 cases but one must try to
always follow a particular case for all problems depending on the convenience. Also,
since we have to always face an arrangement while looking at the arrangement, it
is better if one can consider the items in the arrangement facing you.
3.
The Left and Right of an arrangement and an item within the arrangement is
Opposite. Let us take an example to illustrate point no. 2 and 3.
4 boys Anil, Bipin, Chandra and Deepak are standing in a straight line facing you,
the Reader of the text. If each one is represented by their names, the arrangement
will look as under :
Anil
Bipin
Chandra
Deepak
Let us now try to answer 2 questions.
Q1 : Which person is 3rd from the left of the arrangement?
When we are answering a question like the one above, we will always face the
arrangement starting with Anil and therefore Chandra will be the 3rd person
counting from the left end of the arrangement.
Q2 : Which person is to the immediate right of Chandra?
Does Chandra’s right mean your right? Absolutely not because you are facing the
arrangement and therefore, Chandra’s right will be the exact opposite of your right.
The person to the immediate right of Chandra is hence Bipin.
Page 2 of 204
Graphic Era Hill University, Dehradun
4.
A is 2 places to the right of B means if we are facing the arrangement, it
would be like
5.
A X B. This can also be written as A is 2nd to the right of B.
There are 2 people between A and B will be shown as A X X B or as
B X X A because in this case right or left is not specified.
6.
In an arrangement, A is placed after B. This means A can have any position
that is placed after B starting from the left. Many students by default take the
statement as immediately after which is not true.
7.
Another word used for ‘before’ is preceed while another word used for ‘after’
is follows.
8.
A, B and C are Doctor, Engineer and Lawyer not necessarily in the same
order. We would come across such statements very frequently in Reasoning
problems. What it indicates is that we are not sure about which person has which
profession. It needs to be logically concluded and arrived at.
But there are questions or part of questions in which we come across statements
like A, B and C are Doctor, Engineer and Lawyer respectively which indicates that
their professions are in the same order.
Few points to remember before we start looking at the question sets.
1.
Comprehension is an important part of all reasoning problems. Many a times,
students are not able to do a problem because of their inability to comprehend
whatever is being said.
2.
While attempting a Reasoning problem, please try to do your work neatly and
in an organized manner. It will help you to get to the solution easily.
3.
Information complied must be at a one place and not scattered.
Example Set 1 : 5 Cars a Santro, a Zen, an Alto, an Indica and a Ritz were on
display in a straight line facing you. Further information is as follows:
1.
Ritz is 3rd to the right of Alto but not at a extreme position.
2.
Zen is to the immediate left of Ritz.
Page 3 of 204
Graphic Era Hill University, Dehradun
Let us look at the following example where only 2 statements have been provided
as a part of the information. Let us mark the positions as 1 to 5.
1
2
3
4
5
From statement 1, it is clear that Alto can be either at Position 4 or position 5
because of the reason that Ritz is 3rd to the right of Alto. Now if Alto is at position 4,
then Ritz will be at Position 1 which in turn violates the first condition. Therefore,
Alto is at position 5 and Ritz will be at position 2.
From Statement 2, Zen will be at position 3. We are not sure about the exact
positions of Santro and Indica.
The final arrangement will be as follows:
Santro/Indica
Ritz
Zen
Santro/Indica
Alto
Now, what if the question asks about the exact position of Santro or Indica? We will
not be able to answer the question uniquely. Such a problem set where all values or
information is not definitely determined is called as a Non-Deterministic Set.
The answer to the above question will be cannot be determined which is a popular
answer choice in reasoning.
Important Note : This is a difference between answer choices “None of
These” and “Cannot be Determined”. Answer choice “None of These”
means there is a unique answer but it not amongst the choices provided
while “Cannot be Determined” means the answer to the question asked
cannot be uniquely determined.
What if we add a 3rd statement which says Indica is between the Zen and the Alto.
Now using the additional data provided, we can definitely say that there will be a
unique arrangement
Santro
Ritz
Zen
Indica
Page 4 of 204
Alto
Graphic Era Hill University, Dehradun
Such a set where all values are uniquely determined is called a Deterministic Set.
Important Note : Sometimes, additional data is provided in the questions
in certain sets like the one above. Students make the mistake of taking up
the additional data to solve the entire set. Please remember, data provided
in a question cannot be used for other questions until and unless it is
clearly mentioned in the problem.
Circular Arrangement: Unlike Linear arrangement wherein every position in
unique by itself, in Circular arrangement questions all the positions are identical.
Therefore, in Circular arrangement questions, the position of an individual depends
on the neighbours to the left and right.
Usually in such questions 6 or 8 people are arranged in a circle. Unless mentioned
otherwise, we must try to take positions that are diagonally opposite so that the
problem can be easily solved. Also, conditions given in Circular arrangement
questions can be with reference to the right or left of people or with reference to
Clockwise and Anti-Clockwise movement. There are few additional aspects that
must be taken care in a Circular Question.
A
G
H
C
F
E
D
B
Page 5 of 204
Graphic Era Hill University, Dehradun
If the Question says, how many people are seated between A and D then the
correct answer is either 2 or 4 or the correct answer is Cannot be Determined
because the question does not specify the direction in which the counting has to be
done.
What if the Question asks about the number of people seated between H and D. In
this case, we will not have 2 answers because the number of people seated
between D and H is 3 when counted from either direction. From this we can
definitely conclude that the number of positions between 2 diagonally opposite
positions will be the same on either side. Let us see how an information like this
can be utilized.
Example : There are a number of people seated in a circular arrangement such
that the person at position number 9 is diagonally opposite the person at position
number 21. Then how many people are seated in the arrangement?
If we use the learning above, the number of people on either side of these 2
positions will be the same. There are 11 positions on either side and so the total
number of people will be 11+11+2 = 24
Page 6 of 204
Graphic Era Hill University, Dehradun
Linear Arrangement
Practice Exercise : Level 1
Set 1 : A teacher had arranged for extra classes for his students in a particular
week. 5 topics were to be taught on 5 different days of the week starting from
Monday to Saturday with a Rest day in between. The topics were Logarithms, Set
Theory, Percentages, Clocks and Calendar not in any particular order.
Additional Information is as follows:
1.
The Rest day will be either on Wednesday or Thursday.
2.
Percentages will be the 1st topic taught and will not be immediately followed
by Set Theory.
3.
Clocks and Calendar will have to be taught on consecutive days.
4.
Logarithms will be taught on Saturday.
Q1 : When is the class on Set Theory scheduled?
a.
Wednesday
b. Friday
c. Tuesday
d.
Cannot
be
d.
Cannot
be
Determined
Q2 : Which topic is scheduled on Wednesday?
a.
Clocks
b. Set Theory
c. Calendar
Determined
Q3 : If there is gap of exactly 1 day between the Rest day and the day Clocks topic
is scheduled, then how many days gap is there between Calendar and Logarithms?
a.
2
b. 3
c. 1
Determined
Page 7 of 204
d.
Cannot
be
Graphic Era Hill University, Dehradun
Q4 : Which among the following cannot be true about the rest day?
a.
Immediately after the Clocks class.
b.
2 days before the last class.
c.
1 day after the % class.
d.
None of These.
Q5 : Taking all the conditions into consideration, how many different ways can the
classes be scheduled?
a.
1
b. 2
c. 3
d. 4
Set 2 : 5 friends P,Q,R,S and T are to be seated in a straight line for a photograph.
They are wearing T-Shirts of Maroon, Green, Yellow, Grey and Black not in any
order. Further information is as under :
1.
The person wearing the Grey T-shirt is to the immediate left of the person
who is 2nd from the right end of the arrangement.
2.
Neither R nor S are at the extreme positions.
3.
T wears a Yellow T-shirt and does not have anyone sitting to his right.
4.
Q is wearing a Maroon T-Shirt.
5.
R sits in the middle and is not wearing a Black T-Shirt.
Q6 : What is the colour of the T-Shirt worn by R?
a.
Yellow
b. Grey
c. Green
d. Maroon
Q7 : Counting from the left end of the arrangement, which person is at the 2nd
position?
a.
Q
b. S
c. P
Determined
Page 8 of 204
d.
Cannot
be
Graphic Era Hill University, Dehradun
Q8 : If the person wearing Maroon is adjacent to the person wearing Yellow, what is
the colour worn by the person sitting at the 4th position from the left?
a.
Yellow
b. Black
c. Green
d.
Cannot
be
d.
Cannot
be
Determined
Q9 : How many people are sitting between P and T?
a.
1
b. 2
c. 3
Determined
Q10 : Who is wearing the Grey coloured T-Shirt?
a.
Q
b. P
c. S
d. R
Set 3 : 5 people A,B,C,D and E participated in a race and got the top 5 positions
not in any order. They were wearing Jersey numbers 3, 7, 12, 15 and 19 not in any
order. Following additional information is available:
1.
The person wearing Jersey number 12 finished at one of the top 2 positions.
2.
E came first and was not immediately followed by either B or C.
3.
D wearing the number 3 finished at the 5th position.
4.
C finished before B who was wearing Jersey number 15.
5.
Person wearing Jersey number 7 finished later than at least 1 person
Q11 : At which position did B finish the race?
a.
2nd
b. 3rd
c. 4th
d. 5th
c. E
d.
Q12 : Who was wearing Jersey number 7?
a.
A
b. C
Cannot
Determined
Q13 : If C is wearing Jersey number 7, what is the Jersey number worn by E?
Page 9 of 204
be
Graphic Era Hill University, Dehradun
a.
12
b. 19
c. 15
d.
Cannot
be
Determined
Q14 : If A is wearing Jersey number 12, then who is wearing Jersey number 19?
a.
E
b. C
c. D
d.
Cannot
be
Determined
Q15 : If all conditions are fulfilled, how many different arrangements are possible?
a.
2
b. 4
c. 3
Page 10 of 204
d. 1
Graphic Era Hill University, Dehradun
Vertical, Circular and Others
Practice Exercise : Level 1
Set 1 : 5 people A,B,C,D and E are working in 3 departments in company ABC Ltd.
Each person works in exactly 1 department out of Marketing, Finance or
Operations. While 2 of them work in Marketing, 2 work in Finance and 1 person in
Operations. Each person belongs to a different grade of Seniority from 1 to 5 with
1 being the highest and 5, the lowest.
Further information is as follows:
1.
Any 2 people with consecutive seniority levels cannot work in the same
department.
2.
D is 4 levels of Seniority below A.
3.
B is one level of Seniority above C.
4.
It is possible but not true for C and D to work in the same department.
5.
While D works in Marketing, C does not work in operations.
Q1 : In which Department does E work?
a.
Marketing
b. Operations
c. Finance
d. Cannot be
Determined
Q2 : Which is the pair of employees that work in the same department?
a.
A,D
b. C,E
c. B,E
d. B,D
c. 5
d. 2
Q3 : What is the Seniority Rank of E?
a.
3
b. 4
Q4 : In which department does the person with Seniority Rank 2 work?
a.
Marketing
b. Finance
c. Operation
Determined
Page 11 of 204
d. Cannot be
Graphic Era Hill University, Dehradun
Q5 : If another person was to be transferred from Finance or Marketing to
Operations, in how many different ways can the person be selected?
a.
1
b. 2
c. 3
d. 4
Set 2 : 8 people of the BCCI governing council were seated around a circular table
for their annual meeting. The members of the governing council were the President,
the Vice President, Secretary, Treasurer, Member, Captain, Vice Captain and Coach
of the team. Further information is as under :
1.
The Member was diagonally opposite the Vice Captain and 2nd to the left of
the President.
2.
The Secretary was seated to the immediate left of the Member.
3.
Neither the Captain nor the Coach was seated adjacent to the Vice Captain.
4.
The Vice President was diagonally opposite the Coach.
Q6 : Who was sitting diagonally opposite the President?
a.
Vice Captain
b. Treasurer
c. Captain
d. Coach
Q7 : Taking into consideration all the conditions, how many different arrangements
are possible?
a.
1
b. 2
c. 3
d. 4
Q8 : Counting in the anticlockwise direction, how many people are seated between
the Secretary and the Vice Captain?
a.
2
b. 3
c. 4
d.
Cannot
be
Determined
Q9 : Which pair is not seated diagonally opposite each other?
a.
Member, Vice Captain
President
b. Secretary, Treasurer
d. President, Treasurer.
Page 12 of 204
c.
Coach,
Vice
Graphic Era Hill University, Dehradun
Q10 : Who is seated 3rd to the right of the Captain?
a.
Coach
b. President
c. Member
d. Treasurer
Set 3 : 4 couples the Anands, the Dhagarras, the Chaterjees and the Bishts were
participating in an event where individual skills would be put to test to decide the
winner. Arrangements had been made for each one to stay alone in rooms arranged
equally along both sides of a corridor. Further information is as follows:
1.
The rooms were marked 1 to 8 with Room numbers 1,3,5 and 7 on one side
and Room numbers 2,4,6 and 8 on the other side.
2.
No person will stay in a room directly opposite to one’s spouse. Also, no one
will stay on the same side as the spouse.
3.
While Mr.Chaterjee was in Room number 1, Mrs. Anand was in Room Number
6.
4.
While Mr.Dhagarra is in Room Number 4, Mrs Bisht is in Room Number 3.
Q11 : Who is the person in Room number 7?
a.
Mrs. Dhagarra
b. Mr. Bisht
c. Mr.Anand
d. Mrs. Chaterjee
Q12 : Who is sitting directly opposite Mrs. Dhagarra?
a.
Mrs. Anand
b. Mrs. Chaterjee
c. Mr.Bisht
d.
Cannot
be
Determined
Q13 : How many pairs exist in which 2 people of the opposite gender are sitting
opposite each other?
a.
1
b. 2
c. 3
d. 0
Q14 : Which is the room number occupied by Mr.Bisht?
a.
Room No. 8
b. Room No. 5
c. Room No. 2
Determined
Page 13 of 204
d.
Cannot
be
Graphic Era Hill University, Dehradun
Q15 : Which of the following is definitely false regarding the position of Mrs.
Chaterjee?
a.
To the immediate right of Mrs. Anand
b.
Directly opposite Mr.Anand
c.
2nd to the right of Mr.Bisht
d.
2nd to the right of Mr.Dhagarra.
Page 14 of 204
Graphic Era Hill University, Dehradun
Tabular or Grid Based
Practice Exercise : Level 1
Set 1 : 6 new players L,M,N,O,P and Q were selected to represent the Indian
Cricket Team. Their skills were Fast Bowler, Opening Batsman, Wicket Keeper,
Spinner, Middle Order Batsman and Medium Pace bowler. Each one belonged to one
of the states UP, MP, Gujarat, Delhi, Maharashtra and Bihar not necessarily in the
same order. While the category bowlers includes all the 3 types, the category
Batsman includes both the Batsmen.
Further conditions are as follows:
1.
Each State is represented by exactly 1 person
2.
P belongs to Maharashtra and is not a bowler.
3.
The Wicket Keeper is either O or M and belongs to either Delhi or Bihar.
4.
Q is the Fast Bowler from MP.
5.
The Opening Batsman and the Spinner are from Bihar and UP respectively.
6.
M does not belong to Delhi.
7.
While N belongs to Gujarat, L belongs to the category “bowlers”.
Q1 : Who is the Opening Batsman?
a.
M
b. L
c. N
d. Cannot be Determined
Q2 : The Medium Pace Bowler belongs to which State?
a.
Bihar
b. Gujarat
c. MP
d. None of These
Q3 : Which among the following States does not represent a Bowler?
a.
MP
b. UP
c. Gujarat
d. Delhi
Q4 : Who is the Spinner among all the 6 players?
a.
M
b. N
c. L
d. Cannot be Determined
Q5 : Which among the following combinations is not correct?
Page 15 of 204
Graphic Era Hill University, Dehradun
a.
P-Mah-Middle Order Batsman
b.
L-UP-Spinner
c.
M-Gujarat-Medium Pace Bowler
d.
O-Delhi-Wicket keeper
Set 2 : 6 new ministers were being sworn in as a part of the government. The six
ministers were PC, PM, MB, AKA, KS, and SBC and were being sworn in as The
Ministers of Finance, Defence, Home, Civil Aviation, Telecom and Railways not in
any order.
Further conditions are as under:
1.
The First person to be sworn in was the Defence Minister.
2.
AKA and KS were sworn immediately after one another though not
necessarily in the same order.
3.
There were exactly 2 people sworn in between MB and SBC.
4.
PC was sworn in last and got civil aviation.
5.
The Minister for Finance was sworn in 3rd and was immediately followed by
the Railway Minister.
6.
Neither SBC nor MB got the Defence Ministry.
7.
AKA was sworn in as the Railway Minister and took oath after the Telecom
Minister.
Q6 : Who was the person to take oath as the Home Minister?
a.
MB
b. PM
c. SBC
d. Cannot be Determined
Q7 : At which position did the Telecom Minister take oath?
a.
2nd
b. 3rd
c. 4th
d. 1st
Q8 : Which person took oath as the Finance Minister?
a.
AKA
b. MB
c. KS
d. Cannot be Determined
Q9 : If there was a gap of exactly one between the Finance Minister and MB, then
who among the following took oath as the Telecom Minister?
Page 16 of 204
Graphic Era Hill University, Dehradun
a.
PM
b. SBC
c. AKA
d. None of These
Q10 : If all the conditions are satisfied, how many different solutions are possible?
a.
1
b. 2
c. 3
d. 4
Set 3 : 5 children Jasmine, Krati, Leena, Mishtu and Nupur treated each other at
different Outlets in the city to celebrate their results. The treats were at CCD,
Mishtaan Bhandar, Guptaji, Bisht Snacks and Goverdhan not in any order. Each
place specializes in exactly one of the items, Coffee, Lassi, Lime Juice, Shakes and
Cold Drinks not in any order and is located at Balliwala, Chowk, Bus Stand, Sabzi
Mandi and Nashvilla Road not in any order. Further conditions are as under:
1.
Leena treated with Lassi but neither at Guptaji nor Goverdhan.
2.
Bisht Snacks specializes in Shakes and is located at Balliwala.
3.
Nupur treated with Coffee at Nashvilla Road.
4.
Neither Krati nor Mishtu treated with Lime Juice.
5.
The Outlet at Chowk is known for it’s Lassi.
6.
CCD is located at Sabzi Mandi and is famous for Lime Juice.
7.
Mishtu gave the treat at Guptaji.
Q11 : Which outlet is located at Nashvilla Road?
a.
Guptaji
b. Goverdhan
c. Bisht Snacks
d. Cannot be Determined
Q12 : Which drink is Bus Stand famous for?
a.
Cold Drinks
b. Lime Juice
c. Coffee
d. Shakes
c. Chowk
d. Balliwala
c. Leena
d. Jasmine
Q13 : Where is CCD located?
a.
Nashvilla Road
b. Sabzi Mandi
Q14 : Which friend treated with Lime Juice?
a.
Krati
b. Mishtu
Page 17 of 204
Graphic Era Hill University, Dehradun
Q15 : Which is the outlet known for Lassi?
a.
Bisht Snacks
b. CCD
c. Mishtaan Bhandar
Page 18 of 204
d. Guptaji
Graphic Era Hill University, Dehradun
Grouping or Condition Based
Concept : In this category of problems, a set of conditions is given on the basis of
which teams or groups are to be formed.
Important : Since the number of teams or groups that can be formed can be
substantial, our approach to these type of problems is the reverse of what we have
learnt in arrangement based problems.
In arrangement based problems done earlier (whether deterministic or nondeterministic), we would first solve the problem to whatever extent possible and
then try and answer the questions asked. In a Grouping or Condition based
problem, we would take the general conditions given alongwith the additional
conditions and do it question by question.
It is equally important to understand the conditions given. Let us try to look at the
differences.
Example 1 : A will be in the Team only if B is also selected. The given statement
means that if A is in the Team, then B has to be definitely a part of the team but if
B is in the Team, A may or may not be in the team.
Example 2 : C and D will always be selected together. This means that either both
C and D will be selected or none of them will get selected.
Example 3 : Exactly one from E and F will get selected. This would mean if E is
selected, then F will not get selected and if F is selected, E will not get selected.
Example 4 : At least one of G and H will be a part of the team. This means the
team will either have exactly one of G or H or the team will have both G and H.
Page 19 of 204
Graphic Era Hill University, Dehradun
Grouping or Condition Based
Practice Exercise : Level 1
Set 1 : A chef is preparing sauces using 8 different ingredients-H,J,K,L,M,N,O and
P. According to the recipes, the following requirements apply to the use of
ingredients:
1.
If J is used, both K and P must be used.
2.
M and N must always be used together.
3.
If K is used, at least 2 from H,J,O must also be used.
4.
K and N cannot be used together.
5.
M,O and P cannot all be used in the same dish.
6.
H,L and P cannot all be used in the same dish.
Q1 : Which of the following is an acceptable combination of ingredients for a sauce?
a.
H,J,K,O
b. H,L,M,P
c. L,M,N,P
d. K,L,M,O
Q2 : In a sauce in which J is used, what is the minimum number of additional
ingredients that would meet recipe requirements?
a.
1
b. 2
c. 3
d. 4
Q3 : Which ingredient must be omitted from the combination H,J,K,L,P to meet the
recipe requirements?
a.
H
b. J
c. K
d. L
Q4 : Exactly how many of the 8 ingredients mentioned can be used as the only
ingredient of a sauce?
a.
1
b. 2
c. 3
d. 4
Q5 : Which of the following cannot be included in a sauce that contains M?
Page 20 of 204
Graphic Era Hill University, Dehradun
a.
H
b. L
c. N
d. J
Set 2 : K,L,M,N,P,Q,R,S,U and W are the only 10 members in a department. There
is a proposal to form a team from within the members of the department, subject
to the following conditions:
1.
A team must include exactly one from P,R,S.
2.
A team must include either M or Q but not both.
3.
If a team includes K, then it must also include L and vice versa.
4.
If a team includes one from S,U,W, then it must also include the other two.
5.
L and N cannot be members of the same team.
6.
L and U cannot be members of the same team.
The size of a team is defined as the number of members in the team.
Q6 : Who cannot be a member of a team of size 3?
a.
L
b.M
c. N
d. P
Q7 : Who cannot be a member of a team of size 5?
a.
K
b. L
c. M
d. P
Q8 : What would be the size of the largest possible team?
a.
8
b. 7
c. 6
d. 5
Q9 : What could be the size of a team that includes K?
a.
2 or 3
b. 2 or 4
c. 3 or 4
d. only 4
Q10 : In how many ways can a team be constituted so that the team includes N?
a.
6
b. 3
c. 4
d. 5
Set 3 : A Farmer plants exactly 5 different vegetables ie Beans, Carrot, Karela,
Peas and Spinach. Every year, exactly 3 kinds of vegetables are planted under the
following conditions:
1.
If Carrot is planted, he also plants Beans that year.
2.
If he plants Karela one year, it is not planted next year.
Page 21 of 204
Graphic Era Hill University, Dehradun
3.
In any year, the farmer plants not more than one of the vegetables planted
in the previous year.
Q11 : If the farmer plants Beans, Carrot, Peas one year, which of the following can
be planted next year?
a.
Carrot, Karela, Peas
b.
Karela, Peas, Spinach
c.
Beans, Carrot and Spinach.
d.
Beans, Carrot and Karela.
Q12 : Which of the following is a possible sequence for the farmer to plant in two
successive years?
a.
Beans, Carrot, Karela ; Carrot, Peas, Spinach
b.
Beans, Carrot, Peas; Beans, Carrot, Spinach
c.
Beans, Peas, Spinach; Beans, Carrot, Karela
d.
Carrot, Peas, Spinach; Beans, Karela, Peas.
Q13 : If the farmer plants Beans, Carrot and Peas in one year and Peas in the
second year, which of the following must be planted in the second year?
I.
Beans
II. Carrot
III. Karela
IV. Spinach
a.
I and II
b. III and IV
c. II and III
d. II and IV
Q14 : Which of the following is/are true?
I.
The farmer must plant each vegetable at least once in any two year period.
II.
Every year after the first year, he must plant exactly one of the vegetables
planted in the previous year.
III.
a.
The farmer never plants the same vegetable in three successive years.
Only I
b. Only II
c. Only III
d. Both I and II
Q15 : If the farmer plants Beans, Carrot and Karela in the first year, which of the
following must be planted in the 3rd year?
Page 22 of 204
Graphic Era Hill University, Dehradun
a.
Beans, Carrot, Karela
b.
Beans, Carrot, Peas.
c.
Beans, Karela, Peas.
d.
Beans, Peas, Spinach.
Page 23 of 204
Graphic Era Hill University, Dehradun
Blood Relations
Concepts : In Blood Relation problems, information is provided about people and
their relations using which the reasoning has to be done. Blood relation problems
can be categorized into few types which we will learn as a part of this topic but
before doing so there are few concepts in Blood Relation problems that need to be
understood.
1.
A family tree is always drawn vertical with people in the same generation
been shown at the same level.
2.
In Blood relation problems, the gender of a person cannot be arrived at using
the name, it has to be reasoned out.
3.
In the English language, Uncle could mean any of the following:
a.
Father’s brother : elder or younger
b.
Mother’s brother : elder or younger
c.
Father’s sister’s husband : elder or younger
d.
Mother’s sister’s husband : elder or younger
The same will be true for the relation aunt also.
4.
The differentiating factor is that relations from the Father’s side are called
Paternal and relations from the Mother’s side are called Maternal. Ex. Maternal
Grandfather would mean Mother’s Father.
5.
In Blood Relation problems, a couple would mean 2 people of the opposite
gender married to each other.
6.
Husband and Wife are called Spouse of each other. In the informal language,
they are also called as better half of each other.
7.
Children of the same parents are called siblings. We need to understand that
the word siblings can be used in different ways:
a.
If someone says, ”we are 3 siblings, it indicates that there are 3 children
including the speaker.”
Page 24 of 204
Graphic Era Hill University, Dehradun
b.
If someone says, “I have 3 siblings, it means that the speaker has in all 3
brothers or sisters combined making a total of 4 children.
8.
An unmarried man is called a Bachelor while an unmarried woman is called a
Spinster.
9.
A woman who has lost her husband is called a widow while a man who has
lost his wife is called a widower.
10.
All relations due to marriage to someone are called relations-in-law. For
example Brother-in-law can mean someone’s sister’s husband or for a man it could
also mean wife’s brother.
11.
While children of the same parents are called siblings as indicated in the
details above, a child one of whose parents is a sibling of one of the parents of
another child will be called as cousins of each other.
12.
In Blood relation problems, it is important to understand the question from
the reference point of view.
As an example, let us say that A is the Father of B and the brother of C. Also, C is
the Mother of D. Now let us look at the following questions:
Q1 : How is C related to B?
The answer would be Aunt because C is the Sister of B’s Father A.
Q2 : How is B related to C?
Please understand that Question 2 is asking the relation of B with C and is therefore
different from Question 1 above. Also, we do not know the gender of B and so B
could be either the Nephew or Neice of C. The correct answer to Question 2 will be
Cannot be Determined.
Q3 : How is B related to D?
Answer will be Cousin as the word Cousin does not require to specify the Gender.
Types of Problems in Blood Relations:
Page 25 of 204
Graphic Era Hill University, Dehradun
Type 1 : The first type of problem is in the form of a set of information that needs
to be compiled and then the questions answered. This type will usually have 3 to 4
questions to be answered as per the given data.
Type 2 : The 2nd type of questions are those asked as stand alone ones about a
person looking at a photograph of another person and expressing the relation
between them in words.
Example : Looking at the photograph of a boy a man said, “His Father’s Mother’s
only son is the Father-in-law of my wife”. How is the boy in the photograph related
to the man.
In such questions, the right strategy is to either start with reference to the person
speaking or about whom the person is speaking. Both are used in answering the
questions depending on the ease of using any one of them.
In many questions, we also use the strategy of starting from either ends and then
joining the data to get to the required answer.
Let us try to attempt the given example.
The boy’s father’s mother’s only son is the boy’s father. The man says that the
boy’s father is the father-in-law of his wife ie the boy’s father must also be his
father. Now the tricky part. The answer to the question will depend on whether the
man in question who is the speaker has any brother or not. If he does not have any
brother, then the boy in the photograph is the man himself which can be possible or
the boy in the photograph could be the man’s brother.
The correct answer to the question using the given data will be cannot be
determined.
Page 26 of 204
Graphic Era Hill University, Dehradun
Type 3 : The 3rd type of questions are those where relations are expressed using
Mathematical symbols for example A+B means A is the brother of B, A-B means A
is the Father of B and so on.
Example : If A+B means A is the Father of B and BxC means B is the sister of C,
then which among the following is definitely true?
a.
C is the brother of B.
b.
C is the daughter of A.
c.
A is the Father of C.
d.
None of These.
In the question given, we do not know the gender of C and therefore both options
(a) as well as option (b) cannot be said to be definitely true but irrespective of the
gender, A has to be the Father of C.
Correct answer is Option (c).
We will look at all the all the 3 types in our exercise.
Page 27 of 204
Graphic Era Hill University, Dehradun
Blood Relations Sets Based
Practice Exercise : Level 1
Set 1 : There are 6 people A,B,C,D,E and F in a family. There are 2 married
couples. Further information is as follows:
1.
A is the Grandson of D and the brother of F.
2.
C is the daughter-in-law of E.
3.
E is the Grandfather of F.
Q1 : How is D related to C?
a.
Mother
b. Sister
c. Mother-in-law
d. Cannot be
Determined
Q2 : How many male members are there in the family?
a.
3
b. 4
c. 5
d. Cannot be
Determined
Q3 : Which among the following is a pair of siblings?
a.
A-E
c. D-E
c. A-F
d. None of
c. Nephew
d. Cannot be
These
Q4 : How is F related to B?
a.
Son
c. Daughter
Determined
Q5 : If there is another member G who is the brother of D, how is B related to G?
a.
Nephew
c. Neice
c. Uncle
d. Cannot be
Determined
Set 2 : In a family there are 6 people P,Q,R,S,T and U. The family consists of P, his
married son, daughter-in-law, unmarried son, grand-daughter and unmarried
daughter. Further information is as follows:
Page 28 of 204
Graphic Era Hill University, Dehradun
1.
U’s sister-in-law is unhappy with her but her brother-in-law Q supports her.
2.
R has a good relationship with his Father.
3.
T spends time playing with her aunt.
Q6 : Who is the sister-in-law of S?
a.
T
b. U
c. R
d.
Cannot
c. Daughter
d. None of These
be
Determined
Q7 : How is T related to Q?
a.
Sister
b. Neice
Q8 : Who could have said this to whom, “I am the sister of the husband of your
Mother” ?
a.
T to S
b.
S to Q
c.
S to P
d.
S to T
Q9 : How many female members are there in the family?
a.
3
b. 4
c. 5
d.
Cannot
be
Determined
Q10 : Which among the following is not a pair of siblings?
a.
Q and R
b. S and Q
c. S and T
d. S and R
Set 3 : 8 people A,B,C,D,E,F,G and H are playing a game sitting across a circular
table. There are 4 pairs of couples A-E, B-F, C-G and D-H among them. Also, A and
B are siblings and C and H are also siblings. Further information is as follows:
1.
No person sits adjacent to a spouse or sibling.
2.
While A is sitting opposite to H, B is sitting opposite to D.
Page 29 of 204
Graphic Era Hill University, Dehradun
3.
H is 2nd to the left of D.
4.
C is sitting to the immediate left of A.
Q11 : Who is the person sitting opposite to G?
a.
C
b. F
c. E
d. H
Q12 : Which of the following is true about the position of E?
a.
To the immediate right of H.
b.
2nd to the left of G.
c.
3rd to the right of A.
d.
None of These
Q13 : What is the maximum position separating a couple?
a.
1
b. 2
c. 3
d.
c. E
d. B
Cannot
Determined
Q14 : Which person is 3rd to the left of A?
a.
F
b. D
Q15 : Which among the following is definitely false about the position of B?
a.
2nd to the right of A.
b.
3rd to the left of his spouse.
c.
To the immediate right of E.
d.
3rd to the right of C.
Page 30 of 204
be
Graphic Era Hill University, Dehradun
Blood Relations : Other Types of Problems
Practice Exercise : Level 1
Q1 : Pointing to the photograph of a lady, a man said, “She is the wife of the
Father-in-Law of my wife”. How is the man’s son related to the lady in the
photograph?
a.
Son
b. Grandson
c. Brother
d. None of
These
Q2 : If Alok is the brother of the daughter of Reena’s son, then how is Alok related
to the daughter of Reena’s Husband?
a.
Nephew
b. Son
c. Grandson
d. Cannot be
Determined
Q3 : A is the Father of B’s sister C. D is the daughter of B’s wife and E is the son of
C. How is D related to E?
a.
Sister
b. Sibling
c. Cousin
d. Cannot be
Determined
Q4 : Pointing to a boy, a lady said to a man, “He is the brother of my daughter’s
sister and I am the sister of your daughter”. How is the man related to the boy’s
sister?
a.
Father
b. Uncle
c. Brother
d. Grandfather
Q5 : Aisha and Rahul are siblings. Mohan is the son-in-law of Aisha’s father and
Shanti is the daughter of Rahul’s sister. How is Shanti related to Aisha’s husband?
a.
Daughter
b. Neice
c. Sister
Determined
Page 31 of 204
d.
Cannot
be
Graphic Era Hill University, Dehradun
Q6 : Pointing to the photograph of a man, a man said to a boy, “His Father is the
Grandfather of my sister and his sister is the wife of my Father”. How is the man in
the photograph related to the other man’s sister?
a.
Grandfather
b. Uncle
c. Father
Determined
Directions for Questions 7-10:
A+B means A is the Brother of B
AxB means A is the Husband of B
A-B means A is the daughter of B
A/B means A is the wife of B.
Q7 : What does the relation P-QxR mean?
a.
R is the Mother of P
b.
R is the sister of P.
c.
R is the Father of P.
d.
None of These
Q8 : What does the relation P/Q+R-S mean?
a.
S is the Father-in-law of P.
b.
R is the brother-in-law of P.
c.
S is the Mother of Q.
d.
P is the daughter-in-law of S.
Q9 : How will we depict the relation Q is the sister-in-law of R?
a.
Q+P-R/S
b.
Q/P+R-S
c.
Q+P+R/S
d.
None of These
Page 32 of 204
d.
Cannot
be
Graphic Era Hill University, Dehradun
Q10 : How will the relation “A is the daughter-in-law of B” be depicted?
a.
A/C+D-B
b.
A+C+D-B
c.
A-B+C-D
d.
None of These
Page 33 of 204
Graphic Era Hill University, Dehradun
Coding Decoding
In Coding Decoding problems, a word is given in a coded form. The student has to
understand the basis of coding ie decode the given word and on the same basis do
the coding for another given word. Coding Decoding problems can also be
categorized into certain types and we will try to look at each one of them.
Concepts and Types of Problems:
Type 1 : A word is given in a coded form and another word is asked to be coded
using the same basis. The basis of coding the original word can be further
categorized. We would like to understand some of the popular techniques used for
the basis of coding.
a.
The alphabets of the original word shifted by certain places in the series.
Example If APPLE is coded as BQQMF, then how will MANGO be coded in the same
language?
We can see that the basis is shifting of each alphabet by 1.
Therefore MANGO will be coded as NBOHP
The basis need not be +1. It can be -1 as in the following example
Example If APPLE is coded as ZOOKD, then how will MANGO be coded in the same
language?
MANGO will be coded as LZMFN as the basis of coding is -1.
The Basis could also be an increment or decrement of +1 or -1
Example If APPLE is coded as BRSPJ, then how will MANGO be coded in the same
language?
The differences will be 0,1,2 and so on.
MANGO will be coded as NCQKT.
In all of the examples above, the coding was being done in the forward direction.
This may not always be true.
Page 34 of 204
Graphic Era Hill University, Dehradun
Coding in a problem can be done in the reverse direction or a combination of
forward and reverse direction. In both cases, examples of questions in the forward
direction will be equally applicable.
b.
The alphabets of the original word used in a certain order as the basis of
coding.
Example If MANGO is coded as OGNAM, then APPLE will be coded as ELPPA.
Example If MANGO is coded as AMNOG then APPLE will be coded as PAPEL.
Notice that while the middle alphabet remains the same, the others are taken in
pairs and exchanged.
Example If MANGO is coded as OMGAN, then APPLE will be coded as EALPP.
Example If DECIDE is coded as EDICED, then DIRECT will be coded as IDERTC.
Example If DECIDE is coded as CEDEDI, then DIRECT will be coded as RIDTCE.
In such examples, if the original word is 6 alphabets long, then invariably the word
asked in the question is also of the same length.
c.
Examples where Reverse Equivalent of the alphabets are used.
ABCDEFGHIJKLM
ZYXWVUTSRQON
The reverse alphabet for A is Z, that for B is Y and so on
Example If USED is coded as FHVW then EQUAL will be coded as VJFZO
d.
Examples where the position of the alphabet is used.
For Example if TUBE is coded as 202125, then CYCLE is coded 3253125
Page 35 of 204
Graphic Era Hill University, Dehradun
Type 2 : In this type of problems, a language is used in coded form. One has to
understand the code and then answer the questions asked. This Category can be
further divided into the following problems.
a.
If Red is called Blue, Blue is called Yellow, Yellow is called Violet, Violet is
called Pink and Pink is called Brown in a certain language, then what will be the
colour of the sky as per this language?
These type of problems are easy to attempt using a little logic. We know that colour
of the sky is actually Blue and in this language Blue is called Yellow. Therefore, the
colour of the sky will be Yellow as per this language.
The problem is the same even if the problem is worded as “If Red is Blue, Blue is
Yellow and so on…….
b.
Let us look at another example
If Red means Blue, Blue means Yellow, Yellow means Violet, Violet means Pink and
Pink means Brown in a certain language, then what will be the colour of sky as per
this language?
For many students, there is no difference between example a and b whereas they
are actually 2 different questions. Our answer in the second example should be one
that actually means Blue and therefore our answer will be Red.
Please learn to differentiate between Types a and b.
c.
Let us look at a 3rd example
Ma La Ke means we are sad
La Min Se means Sad is joy
Min Kab ma means joy we must
Which word in the language means are ?
If we compare statements 1st and 2nd, then La stands for sad. Similarly, if we
compare statements 1st and 3rd, then Ma means we and if we compare statements
2nd and 3rd, then Min means joy.
Page 36 of 204
Graphic Era Hill University, Dehradun
Therefore, Ke in the language means are.
A question like the one above is also formed using digits instead of words as the
basis of coding.
Type 3 : In this type, an Input Output Machine rearranges a given group of words
in a particular sequence based on logic. We need to understand the logic being
used and find the answers to the rearrangement of another given group of words.
Sometimes, numbers are used in these problems instead of words.
For Example : Input : today go all we market will to
Step 1 : go today all we market will to
Step 2 : go to today all we market will
Step 3 : go to we today all market will
Step 4 : go to we all today market will
Step 5 : go to we all will today market
Step 5 is the final step for this sentence. It is also called the Output.
Can we identify the logic being used by the machine. It is arranging the words in
increasing order of their length from left to right. Also, for words of the same
length, it is arranging the words in the increasing order of their first alphabet.
In all the types mentioned above, numbers or digits can be used as the basis of
coding instead of alphabets or words.
Page 37 of 204
Graphic Era Hill University, Dehradun
Coding Decoding
Practice Exercise Level 1 :
Q1 : If ATTRACT is coded as UCFXFZY , then how will DENOTES be coded in the
same language?
a.
TFHIYTV
b. FTHIYTV
c. HTFIYTV
d. TFIHYTV
Q2 : If PRICE is coded as RPIEC in a certain language, then how will STAND be
coded in the same language?
a.
ASTND
b. TSAND
c. TSADN
d. STNDA
Q3 : IF APPLE is coded as 11616125 in a certain language, then how will MANGO be
coded in the same language?
a.
13112715
b. 13114715
c. 13117215
d. None of
These
Q4 : If RETIRE is coded as TERERI in a certain language, then how will SUFFIX be
coded in the same language?
a.
SUIXFF
b. FUSXIF
c. XIFFUS
d. None of
These
Q5 : If BEGIN is coded as ACDFFHHJMO in a certain language, then how will ALTER
be coded in the same language?
a.
ZBKNSUDFQS
b. ZBKNSTDFQS
c. ZBKMSUDFQS
d. None of
These
Q6 : If Bus is called Train, Train is called Ship, Ship is called Aeroplane, Aeroplane
is called Cycle and Cycle is called Car in a certain language, then what can fly as
per this language?
Page 38 of 204
Graphic Era Hill University, Dehradun
a.
Aeroplane
b. Ship
c. Cycle
d. None of These
Q7 : If Intelligent means Smart, Smart means talkative, talkative means Foolish
and Foolish means Quick in a certain language, then what will a dumb person be
called in this language?
a.
Talkative
b. Foolish
c. Quick
d.
Cannot
be
Determined
Q8 : If Red is Blue, Blue is Green, Green is Pink, Pink is Violet and Violet is Brown in
a certain language, then what will be the colour of Grass as per this language?
a.
Green
b. Blue
c. Red
d. Pink
Directions for Questions 9-11 : An Input-Output Machine converts a given Input
into Output using a certain logic. You have to understand the logic and answer the
questions that follow.
Input : Deer and Panther form of the Zoo.
Step 1 : and deer panther form of the zoo.
Step 2 : and deer form panther of the zoo.
Step 3 : and deer form of panther the zoo.
Step 3 is the final output in this case.
Q9 : In how many steps will the input “wind blows around from east to south” get
converted into the output?
a.
4
b. 5
c. 6
d. 7
Q10 : What will be the result obtained after the 2nd step?
a.
around wind blows east from to south
b.
around blows wind east from to south
c.
around blows wind from east to south
Page 39 of 204
Graphic Era Hill University, Dehradun
d.
None of These
Q11 : If the result after the 3rd step is “against comfortably India won the match
Pakistan”, then what will be the Input?
a.
won the match Pakistan against comfortably India
b.
won the match Pakistan comfortably against India
c.
won the comfortably match Pakistan against india
d.
Cannot be Determined.
Directions for Questions 12-15 : In a certain language “kal tum hum” means
“we are together”, “Hum ni tak” means “together to go” and “tak kal man” means
“round we go”.
Q12 : Which word in the language means “are”?
a.
Kal
b. tum
c. hum
d. Cannot be Determined
Q13 : What does the word “man” in the language represent?
a.
we
b. go
c. round
d. Cannot be Determined
Q14 : What will be the meaning of “man kal ni” in this language?
a.
Round we to
b.
Go we to
c.
Round we together
d.
Round we are
Q15 : Which word in the language means “go”?
a.
man
b. kal
c. tak
Page 40 of 204
d. ni
Graphic Era Hill University, Dehradun
Direction Sense
Direction sense problems are another category of the problems asked. The ability of
the students to understand directions is tested in these type of problems.
Before we start looking at the examples or the exercise we need to understand
certain parts of the concepts and the types of problems that are usually asked in
this category.
Concepts and Types of Problems :
1.
Directions on a board or on paper are universal and can be shown as per
convenience. The only point to remember is whichever direction is taken as East,
West will be directly behind, North to the left hand side while facing East and South
to the right hand side. If this convention is followed, taking East in whichever way,
the others would automatically get decided.
2.
Another point to know is Clockwise and Anticlockwise. Clockwise will always
be as per the movement of the Clock while Anticlockwise is the exact opposite of
the ways the hands of a clock move. Many a times left and right will be used
instead of Clockwise and Anticlockwise.
3.
Rotation or turn by a particular angle is also an aspect that needs to be
known. Generally, students are comfortable while dealing with turns of 90 degrees
or it’s multiples but make mistakes when the angle is 45 degrees or 135 degrees.
Understanding the above is simple.
Let us take an example of a turn of 135 degrees. When a turn of 135 degrees is to
be effected, it can be taken as 90 degrees and a further 45 degrees. Similarly, if a
turn of 45 degrees is to be effected, it can be seen as half of a 90 degree turn.
Types of Problems : There are 3 different types of problems that are asked as a
part of Direction based Questions. Let us look at all of them through an example.
Page 41 of 204
Graphic Era Hill University, Dehradun
Example : A man started from his house facing the North direction, Walked straight
for 10 m , took a right turn walked for 5 m took another turn to walk for 12 m and
then stopped.
Q1 : In which direction is he walking?
Let us first depict the directions the way we would use in the problem.
North
West
East
South
If he walks for 10 m along the north direction and then takes a right turn, he would
be walking along the East direction for 5 m. Now if he again takes a right turn, he
will be moving along the South direction.
Q2 : In which direction is he with respect to the starting point?
Many students do not differentiate between Q1 and Q2 and thereby end up making
mistakes. The 2nd question is different from the first because in the 2nd question, we
have to find the final position with reference to the starting point. The answer to
the 2nd question will be South-East.
Q3 : How far is he from the starting point?
To answer the above question, we will be invariably using the properties of right
angle triangle. As per the question, his position with reference to the starting point
is as follows:
Page 42 of 204
Graphic Era Hill University, Dehradun
5
2
We need to calculate the distance shown above. Required answer is √ 25+4
= √29 m
Page 43 of 204
Graphic Era Hill University, Dehradun
Direction Sense
Practice Exercise Level 1 :
Q1 : A man starts from his house facing the East direction, walks straight for 7 m,
takes a left turn to walk 5m, takes a right turn to walk 5 m and stops. How far is
from the starting point?
a.
12 m
b. 14 m
c. √21 m
d. 13 m
Q2 : Rohit’s house faces the West direction. He goes out from the back door of his
house, walks for 8m, takes a right turn to walk 10 m, takes another right turn to
walk 10 m and stops. In which direction is he with respect to the starting point?
a.
South
b. South-West
c. West
d. South-East
Q3 : A man is standing facing the South-East direction. He takes 145 turns each of
45 degrees in the clockwise direction and finally a 90 degree turn in the anticlockwise direction. Which direction is he facing now?
a.
East
b. South-East
c. South-West
d. West
Q4 : Mohan and Ram are standing 5 m apart facing the East direction with Mohan
to the right of Ram. They both walk straight for 7m. Mohan takes a right turn walks
for 5m, takes another right, walks for 7m and stops. Ram takes a left turn walks for
7m, takes a right turn walks for 5m and stops. What is the distance between them?
a.
24 m
b. 12√2 m
c. 20 m
d. 12 m
Q5 : Sunita starts from her house and takes a right, left and right turn to reach
School. If she comes from her school to her house along the same path, what will
be the order of turns required?
a.
Left,Right,Right
b.
Right,Left,Right
c.
Left,Right,Left
Page 44 of 204
Graphic Era Hill University, Dehradun
d.
None of These
Q6 : Amit starts from his School facing the West direction, walks straight for 8m
and then takes a 45 degree turn to his left, walks for 5m and stops. In which
direction is he walking?
a.
South-West
b. South
c. South-East
d. None of These
Q7 : Town A is to the North of Town B which in turn is South-West to Town C. In
which direction of Town C is Town A located?
a.
North-West
b. South-West
c. West
d. Cannot be Determined
Q8 : On a bright Sunny morning, 2 friends Vishal and Shashwat are standing facing
each other. If the shadow of Vishal falls to his right, then which is the direction that
Shashwat is facing?
a.
East
b. West
c. North
d. South
Q9 : A boy starts from his house, walks straight for 10m, takes a left turn to walk 4
m, takes a right turn to walk 5m and finally takes a left turn walks for 3m and
stops. How far is he from the starting point?
a.
√226
b. √74
c. √26
d. √274
Q10 : Kartik walks out of the front of his house that faces the West direction, walks
straight for 9m, takes a 135 degree turn in the anticlockwise direction, walks for
5m and then takes a 45 degree turn in the clockwise direction and stops. In which
direction is he walking?
a.
South
b. South-West
c. West
Page 45 of 204
d. None of These
Graphic Era Hill University, Dehradun
Series Completion
Concepts : Series and analogies are designed to measure a student’s ability to find
logical patterns, which may be based on simple mathematical and logical rules and
properties.
Number Series : A Sequence of numbers is given, where one needs to identify the
pattern between the numbers. This pattern needs to be applied in the series to find
out the missing number. There can be infinite ways to create a series, but all the
numbers within a series should have the same relationship between them.
Due to the various patterns that can be used to create a number series, this can
often be a difficult and time consuming question type. To solve problems on
number series, one should be familiar with the basics of numbers ie multiplication
tables, squares, cubes, powers, factorials etc.
a.
Based on Difference
This is the most basic and common form of series. It is based on the difference
between consecutive terms of the series which can be either constant or based on a
Mathematical pattern of it’s own. If the numbers obtained after the first level of
substraction do not show any pattern among them, take the difference of these
numbers. The numbers obtained after second level of substraction may now show a
pattern. Continue this process till a pattern is observed. Ocassionally, the difference
between the terms may be based on special numbers such as prime numbers,
factorials, powers or roots.
Example 1: 336,305,268,227,184,?
The difference between the 1st and 2nd term is 336-305=31
The difference between the 2nd and 3rd term is 305-268=37
The difference between the 3rd and 4th term is 268-227=41
Page 46 of 204
Graphic Era Hill University, Dehradun
The difference between the 4th and 5th term is 227-184 = 43
The difference between any 2 consecutive terms are all consecutive prime numbers.
Therefore the 6th term must be 184-47=137
Example 2 : 1,3,6,10,15,?
The difference between consecutive terms is 2,3,4,5 and therefore, the 6th term
must be 15+6=21
The above series can also be seen as 1,(1+2),(1+2+3),(1+2+3+4) and so on
which again tells us that there could be multiple logic used to form the same series.
b.
Based on Product : A product based series is one where the pattern is
identified by a process of multiplication or division. Common patterns used in these
problems are powers, factorials and multiples. A feature of such problems is that
the value of consecutive terms increases/decreases quite sharply. However, first
level substraction may often help in identifying the underlying pattern.
The series based on product can be further classified into the following types:
1.
Product : The series may be based on simple application of factors or
multiples.
Example 1 : 6,15,35,77,?
The 1st term is 2x3, the 2nd term is 3x5, the 3rd term is 5x7 and the 4th term is 7x11
ie the series is the product of consecutive prime numbers.
The next term will be 11x13 = 143
Example 2 : 6,3,3,4.5,9,?
The 2nd term can be written as 1st term x 0.5, the 3rd term can be written as 2nd
term x 1, the 4th term can be written as 3rd term x 1.5 and so on.
Required value will be 9x2.5 = 22.5
Page 47 of 204
Graphic Era Hill University, Dehradun
2.
Power : In power based problems, the increase in the value of consecutive
terms will be higher as compared to pure product based problems.
Example 1 : 2,6,30,230,?
The given series can be written as (1^1+1), (2^2+2), (3^3+3), (4^4+4)
Required answer will be 5^5+5 = 3130
3.
Factorial : The values of the factorial of the first few natural numbers is used
as the basis for this series.
Example 1 : 1,2,4,9,28, ?
The given series can be written as 0!+0, 1!+1, 2!+2, 3!+3, 4!+4
The answer therefore will be 5!+5 = 120+5 = 125
c.
Alternating Series : An alternating series is a combination of two or more
series. Each series can have different patterns applied to it and then combined to
form a series. In a combination of 2 or more series, alternate terms follow the same
pattern.
Example 1 : 3,4,7,9,11,14,15,?
As can be observed, there are 2 different series, one with a constant difference of 4
(terms being 3,7,11 and 15) and the other with a constant difference of
5(4,9,14…). The answer therefore will be the next term of the 2nd series mentioned
ie 19.
d.
Miscellaneous : These can involve a combination of patterns or series and
cannot be directly classified.
Example 1 : 10,103,18,187,?
a.
979
b. 26
c. 9
Page 48 of 204
d. 251
Graphic Era Hill University, Dehradun
The difference of consecutive terms does not form a logical series. Let us look at
the sum of the digits of each term.
10=1+0=1
103=1+0+3=4
18=1+8=9
187=1+8+7=16
Thus, the sum of digits of each term is the square of consecutive natural numbers.
Hence, the next term of the series should be such that the sum of it’s digits is 5^2
ie 25.
Therefore, 979 is the correct answer choice.
Letter Series : A Letter series generally uses the position of a letter in the
alphabet or some other property of letters such as vowels/consonants etc.
In questions on letters, one should replace the letter by it’s corresponding position
in the alphabet thereby making the pattern simpler to understand. Like number
series, a letter series can also have alternating patterns.
Example 1 : Find the next 2 terms of the series L,M,O,N,R,O…..
The position of the alphabets is 12,13,15,14,18,15
We observe that there are 2 alternating series ie 12,15,18…… and 13,14,15……
Hence the next 2 terms in the series should have positions 21 and 16 respectively.
The next 2 terms should be U and P respectively.
Alphanumeric Series: These include a combination of alphabets, numbers and
symbols. Questions can be asked individually or as part of a group question.
Example 1 : Find the missing term. 1k1,1M3,1Q7,1S9
The position of each alphabet is K:11, M:13, Q:17, S:19
We can see that the numbers on the side of the letter when taken together,
represent the position of the letter in the alphabet. Also, the numbers are
consecutive prime numbers starting from 11.
Page 49 of 204
Graphic Era Hill University, Dehradun
Hence, the next term should correspond to the next prime number ie 23. The letter
at the 23rd position of the alphabet is W and therefore the next term should be
2W3.
Page 50 of 204
Graphic Era Hill University, Dehradun
Series Completion
Practice Exercise Level 1 :
Directions for Questions 1-5 : Find the missing term in the Series
Q1 : 5,8,7,15,11,29,17,50,?
a.
27
b. 55
c. 25
d. 52
c. 721
d. 386
c. 241
d. 244
c. 59
d. 55
c. 62
d. 61
Q2 : 6,19,77,?, 2317
a.
286
b. 387
Q3 : 235,237,240,?,252,263
a.
247
b. 245
Q4 : 5,9,18,34,?,95
a.
57
b. 61
Q5 : 7,26,?,124,215,342
a.
63
b. 64
Directions for Qs 6-7:
There is an alphanumeric series given below. Answer the questions on the basis of
this series.
A37B719C9D5DKL6R6S7S
Q6 : How many letters are immediately preceeded and immediately followed by the
same digit?
a.
1
b. 2
c. 3
d. 0
Q7 : How many digits are immediately preceeded as well as immediately followed
by the same letter?
Page 51 of 204
Graphic Era Hill University, Dehradun
a.
0
b. 1
c. 2
d. 3
b. D
c. K
d. M
b. L
c. N
d. T
b. I
c. J
d. H
b. UV43
c. TU42
d. Q17
b. 49
c. 36
d. 64
c. TU
d. SU
c. hlvS
d. None of These
c. 71
d. 68
Q8 : J,F,?,A,M,J,J
a.
N
Q9: M,T,W,?,F,S,S
a.
H
Q10 : E,F,?,K,O,T
a.
G
Q11 : BC5, E5, GK18, M13,?
a.
QS36
Q12 : PJ:36::TN:?
a.
25
Q13 : AD: WZ::FI: ?
a.
RU
b. RT
Q14 : JpUi:lRwK::FgTq:?
a.
hJvS
b. hlvT
Q15 : 343 : 56 : : 512 : ?
a.
72
b. 66
Page 52 of 204
Graphic Era Hill University, Dehradun
Mathematical Reasoning
Practice Exercise Level 1 :
Set 1 : 5 students P,Q,R,S and T enrolled at the Gateforum centre to prepare for
the Gate exam and availed 5 different products : Classroom, Test Series, E-Tutor,
Powerpack and Material not in any order. The basic price of the products availed by
them was Rs.3500, Rs.5000, Rs.7000, Rs.9500 and Rs.11000 not in any order.
Based on their past performance, they were given discounts of Rs.250, Rs.500,
Rs.750, Rs.1000 and Rs.2000 not in any order. Further information is as follows:
1.
The basic price of the E-Tutor and the Powerpack was more than the average
price of the mentioned 5 products.
2.
The student who got the maximum discount ended up paying the least
amount which was same as the student who availed the Test Series.
3.
While P availed the E-Tutor which was the maximum priced product, T
availed a product having a basic price of Rs.2000 more than the basic price of the
product taken by S.
4.
The basic price of Classroom was more than the basic price of Material.
5.
R got the least discount amongst all the 5 students.
6.
After Discount, T paid double the minimum amount paid by any one of them.
Q1 : What is the discount amount offered to the student who availed E-Tutor ?
a.
Rs.1000
b. Rs.500
c. Rs.750
d. Rs.250
Q2 : What was the price paid by T after discount?
a.
Rs.4000
b. Rs.6000
c. Rs.3000
d. Rs.10250
c. Test Series
d. E-Tutor
Q3 : What was the product availed by R?
a.
Material
b. Powerpack
Page 53 of 204
Graphic Era Hill University, Dehradun
Q4 : Who paid the minimum amount after discount?
a.
Q and S
b. R and S
c. S and T
d.
Cannot
be
Determined
Q5 : Which student took the classroom product?
a.
S
b. T
c. P
d. Q
Set 2 : Coach Fletcher was worried after looking at the performance of the team.
The average scores of all the 11 players in the series was below 50 runs. Incidently,
the average scores of all the 11 players were prime numbers and the average of
the player’s average scores was also a prime number. Also, each of the prime
number was unique with no 2 numbers being the same. Further, only 2 players had
a single digit average in the series.
Q6 : What was the average of the player’s average scores?
a.
23
b. 29
c. 31
d. 37
Q7 : What was the average of the player with the lowest average among the 11
players?
a.
3
b. 5
c. 2
d.
Cannot
be
Cannot
be
Determined
Q8 : What was the highest average score amongst all the 11 players?
a.
43
b. 47
c. 37
d.
Determined
Q9 : How many players had an average score more than the average score of the
all the 11 players combined together?
a.
6
b. 5
c. 4
Page 54 of 204
d. 7
Graphic Era Hill University, Dehradun
Q10 : What is the average of the average scores of the players having the second
highest average and the second lowest average?
a.
27
b. 23
c. 24
d. Cannot be Determined
Set 3 : 2 friends A and B are playing a game of throw the dice. The Game is such
that A will win if he gets a 5 while B will win if he gets a 3. They both take turns till
one of them wins the game. Further information is as follows:
1.
If a person gets a number other than that required to win, he will pay the
other person double the number obtained. For example if A gets a 3, he will give B
Rs.6 and if B gets a 5, he will give A Rs.10.
2.
Each of them had Rs.30 before the beginning of the game.
3.
A is the first one to play.
4.
If any one of them has got a particular number during the game, that
number will not be further considered. For the winning throw, the winner will not
give any amount to the other player.
Q11 : If B wins the game on his 2nd throw, find the maximum value of the amount
with B at the end of the game?
a.
Rs.50
b. Rs.48
c. Rs.20
d. Rs.44
Q12 : If A begins the game and the sequence of the numbers obtained by A and B
in that order is 1,6,2,4,5 then find the difference between the amounts with A and
B at the end of the game?
a.
14
b. 44
c. 32
d. 40
Q13 : If one round is completed when each one gets a chance, find the average
amount with A and B together at the end of the 2nd round?
a.
Rs. 20
b. Rs. 25
c. Rs. 30
Determined
Page 55 of 204
d.
Cannot
be
Graphic Era Hill University, Dehradun
Q14 : Find the probability of A winning in the 2nd round?
a.
1/6
b. 1/3
c. 1/2
d. Cannot be Determined
Q15 : If B begins the game and wins it in the 2nd round and all numbers obtained
are prime, then what is the number obtained by B in the 1st round?
a.
2
b. 5
c. 3
d. Cannot be Determined.
Set 4 : Reyyansh, the smart kid was in the habit of saving money in his kitty bank.
He would put money in his kitty bank everyday starting with Rs. 1 on the 1st day.
On each subsequent day, he would double the money being put into the bank ie
Rs.2 on the 2nd day, Rs.4 on the 3rd day and so on.
Mehuli, Reyyansh’s sister, took out some money from the kitty bank on each day
starting from the 6th day such that she took Rs.6 on the 6th day, Rs.7 on the 7th day
Rs. 8 on the 8th day and so on.
At the end of each day, Reyyansh would count the money being unaware that it
was being taken by his sister also. If the total amount at the end of a day, becomes
more than 20, Reyyansh would take out Rs. 20 to buy a chocolate for himself and
his sister. The remaining amount would be put back for further use. Also, from the
next subsequent day, he would again start by putting Rs.1 into his kitty bank.
Q16 : Which will be the first day when Reyyansh gets to take out Rs. 20?
a.
4th day
b. 3rd day
c. 5th day
d. 6th day
Q17 : What will be the amount left in the kitty bank at the end of the 7th day?
a.
Rs.11
b. Rs. 1
c. Rs.43
d. Rs. 30
Q18 : For how many days from 6th to 10th will Mehuli not be able to take out any
amount due to insufficient funds?
Page 56 of 204
Graphic Era Hill University, Dehradun
a.
2 days
b. 1 day
c. 3 days
d. 0 days
Q19 : If Reyyansh first got to take out Rs. 20 on the 3rd of a particular month, on
which day of the month will he again get to take out Rs. 20?
a.
10th
b. 11th
c. 9th
d. 12th
Q20 : If Reyyansh starts on the 10th of Jan and gets an award of Rs. 25 on the 14th,
find the amount in the kitty bank at the end of 15th Jan?
a.
Rs.37
b. Rs.7
c. Rs.31
d. Rs.43
Set 5 : The following table gives the partial subject wise marks obtained and the
average marks obtained by 5 students of Anne Mary School in the Half Yearly
Exam. Madam Ghosh, the class teacher had entered all the data but some of it was
lost due to a virus malfunction.
Name
Maths
Akshat
90
Kushagra
English Science
Social
75
80
55
Abhinav
72
80
Siddharth
88
70
70
60
Ghosh
60
85
90
55
Average
75
80
75
65
Hindi
Average
80
75
65
88
Q21 : How many marks did Akshat get in Social?
Page 57 of 204
82
80
70
Graphic Era Hill University, Dehradun
a.
75
b. 80
c. 55
d. 70
Q22 : What is the average marks of all the 5 students in Hindi?
a.
75
b. 74.6
c. 77.2
d. 72.5
Q23 : What is the average marks obtained by Kushagra in all the 5 subjects?
a.
73.6
b. 65
c. 69.5
d. 69
Q24 : Which student got the highest marks in English?
a.
Ghosh
b. Akshat
c. Abhinav
d. Kushagra
Q25 : How many students got the same marks in exactly 2 subjects?
a.
2
b. 3
c. 4
Page 58 of 204
d. 5
Graphic Era Hill University, Dehradun
Miscellaneous
Concepts : The 3 important topics that need to be discussed under Miscellaneous
will be Set Theory, Cubes and Dice.
Set Theory : Under Set Theory, we need to understand a lot of concepts related to
sets but most of it will be covered in the Quantitative Ability part of aptitude. We
will concentrate on those aspects of Set Theory that will help us in Reasoning.
1. The Number of distinct elements of a set is called the Cardinal Number of the Set
denoted by n.
2. If the Cardinal number of a Set is n, the number of Subsets of the Set is 2^n.
Out of the total Subsets, one will be an improper Subset and the remaining (2^n-1)
subsets will be proper.
3. If A∩B is a Null Set, then A and B are called Disjoint Sets.
4. For 2 given sets A and B, n(AUB) = n(A) + n(B) if A and B are disjoint sets.
Otherwise, if A and B are not disjoint,
n(AUB) = n(A) + n(B) – n(A∩B)
5.
For 3 sets A,B and C
n(AUBUC) = n(A)+n(B)+n(C)-n(A∩B)-n(B∩C)-n(C∩A)+n(A∩B∩C)
Cubes : In this type of Questions, a Cube is taken and is painted on all the faces
either with the same colour or with different colours. The Cube is then cut into
smaller cubes and the question asked is about the number of cubes with 3 sides
painted, 2 sides painted, 1 side painted, no side painted and so on. We will first try
to understand a general solution to the problem asked.
Example : A Cube of side 5 cms is taken and is painted Red on all it’s faces. It is
now cut into 125 smaller Cubes of side 1 cm each.
Page 59 of 204
Graphic Era Hill University, Dehradun
Cubes with 3 sides painted : Cubes with 3 sides painted will be on the corners of
the main cube. A cube has 8 corners and so the smaller cubes with 3 sides painted
will always be 8.
Cubes with 2 sides painted : Cubes with 2 sides painted will be on the edges of
the main Cube. If n is the total number of Cubes along each edge, then number of
Cubes with 2 sides painted will be (n-2) (leave out the 2 Cubes on the Corners).
There will be (n-2) such Cubes along each edge.
Total Number of Edges in a Cube = 12
Number of Cubes with 2 sides painted = 12(n-2)
For the above example, n=5
Number of Cubes with 2 sides painted = 12(5-2)
= 12 x 3 = 36
Cubes with 1 side painted : Cubes with 1 side painted will be on the faces of the
main Cube. There will be (n-2)^2 cubes on each face.
Number of Faces = 6
Number of Cubes = 6(n-2)^2 = 6x9 = 54
Cubes with no side painted : Cubes with no side painted will be inside the main
cube.
Number of such Cubes = (n-2)^3
= 3^3 = 27 Cubes
Please understand that the above is an explanation of the concept using a standard
case. The question asked can be other than the standard case also.
Page 60 of 204
Graphic Era Hill University, Dehradun
Miscellaneous Problems
Practice Exercise Level 1 :
Directions for Questions 1 to 4 : A Cube of side 5 cms is painted Red on 2
adjacent faces, Green on 2 adjacent faces and Yellow on 2 adjacent faces. It is now
cut into 125 smaller cubes of side 1 cm each.
Answer the questions that follow:
Q1 : How many of the smaller cubes will have 2 faces painted Red?
a.
5
b. 10
c. 15
d. 25
Q2 : How many of the smaller cubes will have exactly 1 face painted?
a.
27
b. 81
c. 54
d. None of These
Q3 : How many of the smaller cubes will not have any face painted?
a.
0
b. 27
c. 36
d. 45
Q4 : How many cubes will have 2 faces painted Green and 1 face painted Red?
a.
1
b. 2
c. 3
d. 4
Directions for Questions 5-8 : Three children Danu, Mehu and Reyu were asked
about some missing cookies by their Mother. While one of them pulled the stool,
the 2nd one took out the jar and the 3rd one took the cookies out of the jar. When
asked, each of them gave 2 statements exactly one of which was false and the
other was true.
Danu : I. I took out the Jar.
II. Mehu’s 1st statement is false.
Mehu : I. Reyu pulled the Chair.
II. Danu did not take out the Jar.
Reyu : I. Mehu took the Cookies.
II. I did not pull the chair.
For all the questions, consider Danu’s first statement as true.
Q5 : Who pulled the chair?
Page 61 of 204
Graphic Era Hill University, Dehradun
a.
Reyu
b. Danu
c.Mehu
d. Cannot be Determined
Q6 : Which among the following is false?
a.
Mehu took out the cookies.
b.
Reyu’s second statement is false.
c.
Reyu pulled the chair.
d.
Mehu’s 2nd statement is false.
Q7 : Who took out the Jar?
a.
`
Reyu
b. Mehu
c. Danu
d. Cannot be
Determined
Q8 : In how many different ways can the conditions be met?
a.
1
b. 2
c. 3
d. 4
Directions for Questions 9-12:
4 friends were playing a game of cards in which one of the players lost in every
round. On losing, the person will double the amounts with the other 3 people. They
played 4 rounds which were lost by A,B,C and D in the same order. Also at the end
of the game, each one of them had 80 Rs. with him/her.
Q9 : What was the initial amount that A started with?
a.
85
b. 45
c. 165
d. 25
Q10 : What was the maximum amount with anyone during the game?
a.
200
b. 180
c . 165
d. 170
Q11 : Which person had the lowest amount at any point during the game?
a.
A
b. B
c. C
d. D
Q12 : Who started with the 2nd lowest amount amongst all the 4 people?
a.
A
b. B
c. C
d. D
Q13 : A vegetable vendor keeps vegetables on one side and weights on the other
side. Find the minimum number of integral weights required so that all weights
from 1 to 50 Kgs can be measured?
a.
4
b. 5
c. 6
Page 62 of 204
d. 7
Graphic Era Hill University, Dehradun
Q14 : What will be the minimum number of weights required if he decides to keep
the weights on either side of the weighing balance?
a.
4
b. 5
c. 3
d. 6
Q15 : A person was asked about the birds sitting on a tree. He said that they were
all Sparrows but 6, all Parrots but 6 and all Mynahs but 6. Find the total number of
birds?
a.
18
b. 12
c. 15
Page 63 of 204
d. 9
Graphic Era Hill University, Dehradun
Syllogism
Concepts : It is a category of Deductive Reasoning. In this type of problems, we
need to check the validity or truthfulness of a given conclusion taking the
statements (premise) given to be true. One should not question the statements
(premises) given. They have to be taken as true under all situations. Before we
attempt the problems on Syllogism, we need to understand the concepts involved.
1.
Concept of All : Example of a Premise is “All A’s are B’s”.
Can be represented as
A/B
B
A
2.
Concept of Some: Example of a premise Some A’s are B’s
It is important to understand that wherever “All” is true, some is automatically true.
Also, Some A’s are B’s means there is at least one A which is B. We cannot say
anything about the other A’s. So, saying that some A’s are B’s also means that
some A’s are not B’s in definitely not a conclusion but only a possibility. Also, if All
A’s are B’s is true then some A’s are B’s is also true.
A/B
A
B
B
A
Page 64 of 204
Graphic Era Hill University, Dehradun
A
3.
B
Concept of Some Not : Example of a premise Some A’s are not B’s
A
B
A
B
B
A
4.
Concept of No : Example of a premise No A is B
A
B
Approach in a Syllogism problem : As can be seen, a particular given premise
can be represented in different ways. Any given statement is a conclusion only if it
can be correctly validated in all possible conditions. As an example, suppose a given
Page 65 of 204
Graphic Era Hill University, Dehradun
statement is validated as a conclusion in 4 different situations but proven as
incorrect in the 5th situation. Now the given statement cannot be termed as a
conclusion. We can merely call it as a possibility.
Therefore, in view of the above argument, we do not try to prove that a given
statement can be logically derived from the premises given and therefore is a
conclusion. In fact we try to prove that it is false in a particular situation thus
eliminating in totality it’s chances of being a conclusion.
In the above paragraph, we understood and agreed on the fact that instead of
proving that a statement is a conclusion, we try and prove that the statement is not
a conclusion. This would be possible in the worst case scenario which can be
defined as under:
For a positive conclusion, the worst case scenario is the venn diagram with the
minimum overlap.
For a negative conclusion, the worst case scenario is the venn diagram with the
maximum overlap.
Page 66 of 204
Graphic Era Hill University, Dehradun
Syllogism
Practice Exercise 1:
Directions for Questions 1 to 10 : In each question below, there are two
statements followed by two conclusions numbered 1 and 2. You need to decide
which of the given conclusions logically follows from the statements given. Mark
your answer as
a. If only conclusion I follows
b. If only conclusion II follows
c. If neither I nor II follows
d. If both I and II follow
Q1 :
Q2 :
Q3 :
Q4 :
Q5 :
Statements 1. Some players are singers.
2. All singers are tall
Conclusions 1. Some players are tall
2. All players are tall
Statements 1. Some hats are caps
2. Some caps are mats
Conclusions 1. Some caps are hats
2. Some mats are caps.
Statements 1. Some vegetables are fruit
2. No fruit is black.
Conclusions 1. Some blacks are vegetables
2. No vegetable is black.
Statements 1. Some fools are intelligent
2. Some intelligent are great.
Conclusions 1. Some fools are great
2. All great are intelligent.
Statements 1. All stones are water
2. Some waters are clean.
Conclusions 1. Some stones are clean
2. No stone is clean.
Page 67 of 204
Graphic Era Hill University, Dehradun
Q6 :
Q7 :
Q8 :
Q9 :
Statements 1. All keys are locks
2. All locks are screws.
Conclusions 1. All screws are keys
2. Some screws are not keys.
Statements 1. Some books are pen
2. No pen is pencil.
Conclusions 1. Some books are pencils
2. No book is pencil.
Statements 1. All dogs are monkeys
2. No monkey is a cat.
Conclusions 1. No dog is a cat
2. No cat is a dog.
Statements 1. Some phones are watches
2. All watches are guns.
Conclusions 1. All guns are watches
2. Some guns are phones.
Q10 : Statements 1. All umbrellas are airplanes
Conclusions 1. Some umbrellas are birds
2. Some airplanes are birds
2. Some birds are umbrellas.
Directions for Questions 11-15 : In the given questions, which of the following
are valid arguments?
Q11 : a. Some printers are fast. No fast is slow. No printer is slow.
b. No poet is a thinker. Some thinkers are leaders. No leader is a poet.
c. No poet is a thinker. Some thinkers are leaders. Some leaders are not poets.
d. Only cats are dogs. No cat is a pig. No dog is a pig.
a. A
b. B and C
c. C and D
d. B
Q12 : a. All music is good. All good is great. All music is great.
b. All men are wise. All wise are leaders. All leaders are men.
c. All men are wise. All wise are leaders. Some men are leaders.
Page 68 of 204
Graphic Era Hill University, Dehradun
d. All boys are men. Some men are smart. Some boys are smart.
a. A
b. B
c. D
d. A and C
Q13 : a. I go out only on Sundays. Today is a Sunday. I am out.
b. I go out only on Sundays. Today is not a Sunday. I am not out.
c. I go out only on Sundays. I am out. Today is a Sunday.
d. I go out only on Sundays. I am not out. It is not a Sunday.
a. B and C
b. B
c. A and D
d. A
Q14 : a. All princes have moles. Raj has a mole. Raj is a prince.
b. All princes have moles. Raj is a prince. Raj has a mole.
c. Only princes have moles. Raj is a prince. Raj has a mole.
d. No princes have moles. Raj is not a prince. Raj does not have a mole.
a. B
b. C and D
c. A and D
d. C
Q15 : a. All kings are men. Some men are weak. Some weak are kings.
b. All kings are men. All men are weak. Some weak are kings.
c. All kings are men. All men are weak. All king are weak.
d. All kings are men. All men are weak. All weak are kings.
a. A
b. B and C
c. C and D
Page 69 of 204
d. D
Graphic Era Hill University, Dehradun
Data Sufficiency
Concepts
Data Sufficiency or DS as it is popularly known, is one of the types of questions
asked in aptitude Tests. As the name indicates, the objective is to find out whether
the data given is sufficient to answer the question asked.
A good student always looks forward to a Data Sufficiency problem because of 2
reasons:
1.
In a Data Sufficiency problem, one does not need to find out or arrive at the
actual answer. As soon as it is decided, whether a unique answer can or cannot be
arrived at, one can stop doing the question and mark the answer option
accordingly.
2.
As an effect of the point above, a data sufficiency problem does not take as
much time as a quant problem would.
As a part of our approach, we will try to understand the structure of a data
sufficiency problem, the answer options, the approach and the errors that are
commonly committed.
In a Data Sufficiency problem, a Question stem or Question is followed by 2 given
Statements. The usual answer options are as follows:
a.
If the Question can be answered by using any one of the 2 Statements alone
but not by the other alone.
b.
If the Question can be answered by using either of the 2 statements alone.
c.
If the Question can be answered by using the statements together.
d.
If the Question cannot be answered by using the given statements.
The approach to attempt a Data Sufficiency problem is to check whether the
Question can be answered using the first statement alone. Then we will do the
Page 70 of 204
Graphic Era Hill University, Dehradun
same using the 2nd statement. Based on this, we will be able to decide whether our
answer can be a or b. If the answer is not a or b, we will check whether the
statements combined can answer the questions asked. If the question can be
answered, the answer option is c otherwise d.
Some common errors related to the process :
1.
Do not assume that the options a to d will always be the same or in the same
order. Therefore, whenever you are doing a DS problem, do read the options to
check whether the options are the same or in anyway different.
2.
Many students make the mistake of picking up data or pieces of information
from statement b while checking the question using only statement a or vice versa.
If the question is being checked using a single statement, the other statement
should not be used in any form.
3.
Another mistake made happens particularly when the problem looks a little
difficult to solve. Many of us instead of checking with the individual statements,
directly combine both the statements thereby checking for only option c or d while
one of the 2 statements alone could also have given an answer.
Few of the Conceptual Errors:
1.
Data Sufficiency always demands a unique answer in any situation. More
than one answer is not acceptable in DS.
Example What is the value of x?
I.
x^2-5x+6=0
If we solve for x, the quadratic equation will give the values as x=2 or x=3.
In the same situation, we have more than one answer. This is not acceptable in DS.
We always need a unique solution. It is here that DS is different from Quant.
Page 71 of 204
Graphic Era Hill University, Dehradun
2.
Even a definite No is an answer. We normally tend to think that only a
definite yes can be taken to be a answer to the question asked. This is not true as
can be seen from the following example:
Example : Can I fill up the Tank of 100 litres capacity completely?(Measurements
are accurate and there should be no overflow).
I.
I have a Bucket which can exactly measure 5 litres.
II.
I have a Mug which can exactly measure 3 litres.
Many students make the mistake of thinking that only statement I alone can
answer the question asked because the important aspect for them is that the Tank
should be filled whereas the important aspect is whether we can answer the
question uniquely about the Tank getting filled.
Therefore, in the above example, using Statement I alone, the answer is Yes, the
Tank can be filled completely and so Statement I alone is able to answer the
question asked.
Similarly, using Statement II alone, the answer is a definite No, the Tank cannot be
filled completely and so Statement II alone is also able to answer the Question
asked.
So, as per the standard answer options of DS, the correct answer to the question is
option b.
Page 72 of 204
Graphic Era Hill University, Dehradun
Data Sufficiency
Practice Exercise 1 :
Directions for Questions 1-15 : In each of the following questions, a question
stem is followed by 2 different statements. You need to decide whether the
statements given are sufficient to answer the question asked. Mark your answer
option as
a. If the Question can be answered by using any one of the 2 statements alone
but not by the other alone.
b. If the Question can be answered by using either of the 2 statements alone.
c. If the Question can be answered by using the 2 statements together.
d. If the Question cannot be answered by using the given statements.
Q1 : Is it Friday today?
1. 1st of February of this year was a Monday.
2. Today is March 2nd.
Q2 : What day is it today?
1. 1st of February of this year was a Monday.
2. Today is March 2nd.
Q3 : What is the sum of x and y?
1. The LCM of x and y is 51.
2. One of the 2 numbers is 17.
Q4 : In how many days can A and B finish the work if they work on alternate days?
1. A alone can do the work in 30 days.
2. B alone can do the work in 20 days.
Q5 : In a football match at half-time, M and M club was trailing by 3 goals. Did it
win the match?
1. In the 2nd half, M and M scored 4 goals.
2. The opponent scored 4 goals in the match.
Q6 : What is the value of a+b?
1. The product axb = 30
Page 73 of 204
Graphic Era Hill University, Dehradun
2. a is 7 more than b.
Q7 : I eat Chicken only on Tuesdays, Saturdays and Sundays. Did I eat Chicken
today?
1. I ate Chicken yesterday.
2. I ate Chicken 2 days before yesterday.
Q8 : What is the value of x?
1. X^3 = 8
2. x^2 = 4
Q9 : P,Q,R,S and T are the only members in a family out of which at least 3 are
females. If P is the father of R and Q has 2 sons, then who is the Uncle of R?
1. Q is the grandmother of R.
2. T is the aunt of R.
Q10 : In how much time can a Train pass a man moving in the same direction?
1. Length of the Train is 150 m and it’s speed is 25 m/sec.
2.The man moving in the opposite direction could have passed the Train in 5
seconds.
Q11 : Which of the two values a^1/a or b^1/b is greater?
1. a and b both are greater than 2.
2. a is less than b.
Q12 : What is the profit or loss percentage?
1. The mark-up % is 40 and a discount of 10% is given.
2. The Cost Price is Rs.200.
Q13 : From a pack of cards I draw 4 cards at random. Is there a king of hearts in
them?
1. All 4 cards are drawn one by one without replacement and all of them are
either kings or queens or jacks.
2. All 4 cards are kings.
Q14 : What is the value of x if it is a natural number less than or equal to 100?
1. x has an odd number of factors.
2. x is the cube of a natural number.
Page 74 of 204
Graphic Era Hill University, Dehradun
Q15 : In how much time can the leak alone empty the tank?
1. The Pipe can fill the tank alone in 15 minutes but it takes 5 minutes more
due to the leak.
2. The Pipe along with an identical pipe can fill half the tank in 5 minutes.
Data Interpretation
The primary objective of Data Interpretation or DI as it is popularly known is to
understand a given data and use it for effective decision making. The Data given
could be in the form of a Table, Bar Graph, Line Graph, Pie Chart, Frequency
Polygon, Caselet or any other form. We need to understand the given data,
assimilate it in the required form and use it to answer the questions asked.
There are few basic points that must be kept in mind as far as DI is concerned.
Page 75 of 204
Graphic Era Hill University, Dehradun
3.
Calculating ability of an individual forms the heart of all DI problems.
Therefore, one of the primary skills to be developed in DI is the ability to calculate
accurately and quickly.
4.
Approximation plays an important role in DI with reference to the ease with
which the answer can be arrived at.
5.
The options are an integral part of the problem in DI in the sense that they
play a far more important role in helping us decied the level of approximation that
can be used.
6.
Do not assume anything from your side other than what has been mentioned
in the problem.
7.
Read the data very carefully.
Concepts in Data Interpretation :
1.
Growth and Growth Rate : Growth and Growth Rate are both expressed on
the immediate previous value until and unless mentioned otherwise. While Growth
is in absolute terms, Growth Rate is in percentage terms.
Sales of Co. XYZ in Rs. Cr.
2009
2010
2011
2012
120
130
140
150
Ex 1 : Find the Growth of Production in 2011?
Many of us would give an answer of Rs.10 Cr. to the question above which is not
correct. The right answer to the above question will be cannot be determined
because the Data given is about Sales while the question is about the Production.
Ex 2 : Find the Growth in Sales of Co. ABC in 2009?
The answer to this question will also be cannot be determined because we do not
have the data for 2008.
Page 76 of 204
Graphic Era Hill University, Dehradun
Ex 3 : Find the Growth in Sales of Co. ABC in 2011?
The correct answer to the question above will be Rs. 10 Cr.
Ex 4 : Find the Growth Rate in Sales of Co. ABC in 2011?
=((140-130)/130) * 100 = 7.7%
2.
Percentage Change in Growth Rate : Many of us do not differentiate
between the calculation of percentage change in growth rate and growth rate.
Actually, they are 2 different concepts.
Percentage change in Growth Rate gives the % change between 2 values that are
expressed in percentage terms.
3.
Percentage Points: It is the difference between 2 values that are expressed
in percentage terms.
4.
Market Share : is the total % of the Market serviced by a Company, Product
or Brand. Market Share can be calculated either on Volume basis (No. of Units sold)
or Value basis (Revenue or Sales or Turnover). Also average product price is equal
to the total market value divided by the total market volume.
Data Interpretation
Practice Exercise Level 1
Set 1 : The Following Table gives the sales of 4 companies in Rs. Lacs across 4
years from 2004 to 2007. Read the data carefully and answer the questions that
follow:
Page 77 of 204
Graphic Era Hill University, Dehradun
2004
2005
2006
2007
A
120
130
145
160
B
150
166
160
190
C
170
185
190
200
D
110
125
140
150
Q1 : By what % is the sales of company B in 2005 more than that of Company A in
2004?
a.
46 %
b. 38.33 %
c. 20 %
d. 33.33%
Q2 : By how much value is the average sales of company C more than the average
sales of company D during the period?
a.
55 lacs
b. 45 lacs
c. 38 lacs
d. None of
These
Q3 : If the 4 companies account for a 40% market share by value in 2007, what
was the total sales of the Market?
a.
16 Crore
b. 20 Crore
c. 14 Crore
d.17.5 Crore
Set 2 : The pie chart shows the % Market Share by volume of 5 Companies P,Q,R,S
and T in the Television Market in the year 2011. The total Sales Volumewise, in the
Market in the year 2011 is 4,00,000 TV sets. The table provides the Number of
units of 4 different products sold by the Company P in the same year.
Page 78 of 204
Graphic Era Hill University, Dehradun
Q4 : If S is the others Category and Company X has a share of 30% in this
category, find the number of TV sets sold by Company X in the year 2011?
a.
10,000
b. 20,000
c. 12,000
d. 15,000
Q5 : What is the % contribution of Products P2 and P3 in the total sales unitwise of
Company P in the year 2011?
a.
50%
b. 40%
c. 60%
d. 75%
Set 3 : 2000 students applied for admission to the various programs at AU
University. Out of the total applicants, 20% did not write the admission Test. The
following table gives the Cumulative Frequency in % of the mark range received by
those students who appeared for the admission test.
Cumulative Frequency in
Marks
%
≤10 Marks
15
≤ 20 Marks
25
≤ 30 Marks
40
≤ 40 Marks
60
Page 79 of 204
Graphic Era Hill University, Dehradun
≤ 50 Marks
85
≤ 60 Marks
100
Q6 : What is the number of students who got Marks in the range of 21 to 30 in the
admission test?
a.
640
b. 240
c. 400
d. None of These
Q7 : If more than 40 marks is required to qualify for the next round, find the
difference between the number of students who qualified for the next round and
those who failed to qualify for the next round?
a.
160
b. 240
c. 420
d. 320
Q8 : By what % is the number of students in the Range of 41 to 50 Marks more
than those in the range of 0-10 Marks?
a.
66.66%
b. 33.33%
c. 10%
d. 20%
Set 4 : The following Pie Chart gives the % distribution of the Total Expense of a
Family under various heads in the month of August.
Savings = Income – Expenditure.
The Income of the Family for the Month of August is Rs.50,000/-.
Page 80 of 204
Graphic Era Hill University, Dehradun
Q9 : If the family saves 30% of it’s income in the month of August, then find the
total expense on Petrol in the month of August?
a.
3500
b. 7000
c. 10000
d. 8000
Q10 : If the total expenses on Education and Misc. heads in the month of August is
Rs.7000/-, then find the savings of the family in the month of August?
a.
Rs.25,000
b. Rs.30000
c. Rs.15000
Set 5 :
Page 81 of 204
d. Rs.22000
Graphic Era Hill University, Dehradun
The Bar Graph shows the Total Sales value in Rs.Lacs and the profit% for a
Company ABC ltd. for 4 years from 2008 to 2011. Answer the Questions based on
the above data.
Q11 : If the total Sales is equivalent to the Selling Price, What is the value of profit
for ABC Ltd. for the year 2010?
a.
Rs.35 lacs
b.Rs. Rs.38.75 lacs
c. Rs.40 lacs
d. None of
These
Q12 : Which year has shown the highest % change in profit % ?
a.
2008
b. 2009
c. 2010
d. 2011
Q13 : By what % is the sales in 2011 more or less than the cost in 2008?
a.
50%
b. 100%
c. 200%
Page 82 of 204
d. 250%
Graphic Era Hill University, Dehradun
Puzzles
Q1 : Fifty minutes ago, it was four times as many minutes past three o’ clock, then
how many minutes is it to six o’ clock?
Q2 : A Vegetable vendor has a weighing balance such that he keeps the vegetables
on one side and the weights on the other. He will sell vegetables in integral values
only. If he has to sell any value from 1 to 100 kgs. find the minimum number of
weights required.
Q3 : In the above question, if he now decides to keep the weights on both sides,
find the minimum number of weights required.
Q4 : In a family, each boy has as many sisters as brothers but each girl has twice
as many brothers as sisters. Find the number of children in the family?
Q5 : A little girl sells oranges from door to door. One day she sold 1/2 an orange
more than half her oranges to the first customer. To the second customer, she sold
1/2 an orange more than half of the remainder and to the third and last customer
she sold 1/2 an orange more than half she had leaving her none. Find the minimum
number of oranges that she had.
Q6 : The value of a 2 digit number like 13 increases in value by 18 when the digits
are reversed. How many other such 2 digit numbers exist?
Q7 : I go to School everyday to pick up my children. I meet them at 4 pm daily
when they finish their school pick them up and we all come back home. One day
the School closed at 2 pm and the children started walking towards home. I met
them on the way and we all reached home 20 minutes earlier than usual. For how
long did the children walk?
Q8 : A railway track runs parallel to a road until a bend brings the road to a level
crossing. A cyclist rides to work along the road everyday at a constant speed of 12
miles per hour. He normally meets a train that travels in the same direction at the
crossing.
Q9 : One day he was late by 25 minutes and met the Train 6 miles ahead of the
level crossing. What is the speed of the Train?
Q10 : A ship I know is twice as old as its boiler was when the ship was as old as the
boiler is now. If the combined age of the ship and boiler is 30 years, find the
individual ages of the Ship and the Boiler.
Page 83 of 204
Graphic Era Hill University, Dehradun
Q11 : A box contains 5 Red, 4 Blue and 3 Yellow balls. How many minimum
marbles must be picked out to ensure that all the marbles of one colour have been
drawn out.
Q12 : There are 32 things available. All of them are identical and of the same
weight except for one which is heavier. Find the minimum number of weighings to
find out the faulty one?
Q13 : I entered a store and spent one-half of the money that was in my purse.
When I came out, I found that I had as many paise as I had rupees and half as
many rupees as I had paise when I went in. How much money did I have when I
entered?
Q14 : 3 Farmers paid Rs.1000 for a small pasture. One farmer grazed his 9 mules,
another his 12 cows for twice the time and the last man put in some goats for 2.5
times as the second man’s cows and paid half the cost of the pasture. Can you find
out how many goats did the last man have, if 6 cows eat as much as 4 mules and
10 goats as much as 3 cows. How much did the first and the second man pay?
Q15 : My friend gave me 2 candles when the lights went out. Though they were of
the same length, he told me that one would burn for 4 hours and the other for 5
hours. After I had been reading for some time, I put the candles out as the lights
came on again. And I noticed that what remained of one candle was exactly four
times the length of what was left of the other. Can you find out how long those two
candles were burning?
Q16 : A man owns a horse, a cow and a sheep. He also owns a pasture. If the
horse and cow can eat the contents of the pasture in 40 days, while the horse and
sheep can do it in 60 days and the cow and the sheep in 90 days. How long will all
of them together take to eat the contents?
Q17 : A Tennis tournament has 30 participants. If a player is eliminated when he
loses 2 matches, find the maximum number of matches to be played to decide the
winner?
Q18 : One of the words listed below is my secret word
AIM
DUE
MOD
OAT
TIE
With this list in front of you, If I were to tell you any one letter of my secret word,
then you would be able to tell me the number of vowels in my secret word. Which
word is my secret word?
Page 84 of 204
Graphic Era Hill University, Dehradun
Q19 : Zita, her brother, her daughter and her son are tennis players. As a game of
doubles is about to begin:
a.
b.
c.
Zita’s brother is directly across the net from her daughter.
Her son is diagonally across the net from the worst player’s sibling.
The best player and the worst player are on the same side of the net.
Who is the best player?
Q20 : A,B and C often eat dinner out. Each orders either Tea or Coffee after Dinner.
a.
b.
c.
If A orders Coffee, then B orders the drink that C orders.
If B orders Coffee, then A orders the drink that C doesn’t order.
If C orders Tea, then A orders the drink that B orders.
Who do you know always orders the same drink after Dinner?
Q21 : A
B
C
D
E
F
G
H
I
Each of the digits 1 to 9 is represented by a different letter in the figure
such that A+B+C = C+D+E = E+F+G= G+H+I = 13
above
Which digit does E represent?
Q22 : One of Mr. H, his wife, their son and Mr.H’s Mother is a Doctor and another is
a Lawyer.
a.
If the Doctor is younger than the Lawyer, then the Doctor and the Lawyer
are not blood relatives.
b.
If the Doctor is a woman, then the Doctor and the Lawyer are blood
relatives.
c.
If the Lawyer is a man, then the Doctor is a man.
Q23 : A, B and C each have some children.
Page 85 of 204
Graphic Era Hill University, Dehradun
a.
A has at least one girl and twice as many boys as girls.
b.
B has at least one girl and 3 times as many boys as girls.
c.
C has at least one girl and 3 more boys than girls.
d.
When I tell you the number of children we have together, a number less than
25, you will know how many children I have, but not how many children each of the
others has. Who is the speaker?
Q24 : 2 people are playing a game in which a person starts the game by saying any
number from 4 to 14. The 2nd person on his turn can add any number from 4 to 14
to the already spoken number and say a new number. The first person will again
add a number from 4 to 14 to the last number spoken and so on. The game
continues till one of them speaks the number 100.
a.
The person who speaks 100 first wins the game. If A starts the game, what
number should A speak in order to be sure of winning the game.
b.
The person who speaks 100 first wins the game. If A starts the game and
speaks 14 what is the number that B must add to ensure that he is the one to win
the game.
c.
The person who speaks 100 first will lose the game. If A starts the game and
speaks 5, then B adds 7 to it and speaks 12, then A adds another 5 and speaks 17,
what number should B speak so as to win the game.
Q25 : My house has a number
a.
If my house number is a multiple of 3, then it is a number from 50 through
59.
b.
If my house number is not a multiple of 4, then it is a number from 60
through 69.
c.
If my house number is not a multiple of 6, then it is a number from 70
through 79.
What is my house number?
Q26 : The owner of the mansion has been murdered. The visitors to the mansion
were A,B and C.
a.
The murderer, who was one of the three visitors, arrived at the mansion later
than at least one of the other two visitors.
b.
A detective, who was one of the three visitors, arrived at the mansion earlier
than at least one of the other two visitors.
c.
The detective arrived at the mansion at midnight.
Page 86 of 204
Graphic Era Hill University, Dehradun
d.
e.
f.
Neither A nor B arrived at the mansion after midnight.
The earlier arriver of B and C was not the detective.
The later arriver of A and C was not the murderer.
Who was the murderer?
Page 87 of 204
Graphic Era Hill University, Dehradun
Practice Exercises
Level 2
Arrangement Based 1
Set 1 : 6 Books one each on English, Maths, Science, History, Hindi and Sanskrit
are to be arranged in a straight line on a book shelf. Further conditions are as
follows:
1.
There are exactly 2 books between the Science and the Sanskrit books but
the Sanskrit book is not at either end.
2.
The History book is to the immediate right of the Maths book.
3.
Counting from the left end of the arrangement, the English book is at the 3rd
position and 2nd to the left of the Science Book.
Q1 : Counting from the right end of the arrangement which book is to the
immediate left of the 3rd book from the right?
a.
Sanskrit
b. Science
c. Maths
d. History.
Q2 : How many books are there between the Maths and the Sanskrit Book.
a.
1
b. 2
c. 3
d. 0
Q3 : Which are the 2 Books at the extreme ends?
a.
Science, History
b. Science, Maths c. Maths, Science
d. None of These
Q4 : Which book is to the immediate left of the Maths Book?
a.
Science
b. History
c. Sanskrit
d. None of These
Set 2 : 5 Departments of The Excellent University and The Library are located on
the 6 different Floors of a six Storey Building such that exactly one of them
Page 88 of 204
Graphic Era Hill University, Dehradun
occupies each Floor. The Departments are Acads, Lab, Finance, HR and Marketing.
Finance, HR and Marketing are known as Auxilliary Departments whereas the
Library and the Lab are called Support Departments.
1.
None of the Auxilliary Departments are located on Adjacent Floors and the
same is true for the Support Departments.
2.
The Ground Floor of the Building is called Floor 1 while the top Floor is called
Floor 6.
3.
The Finance department is located on one of the odd numbered Floors.
4.
The Marketing department is adjacent to the Acads department and is
immediately above the Library.
5.
The Finance department occupies one of the Floors above the HR
department.
Q5 : Which Department is located on the 4th Floor?
a.
Library
b. Finance
c. Acads
d.
Cannot
be
Determined
Q6 : On which Floor is the HR department located?
a.
3rd
b. 1st
c. 5th
d. Cannot be Determined
Q7 : What can be said about the following 2 Statements?
1.
The Marketing Department can be located on the 5th floor of the building.
2.
The Acads department is on the Ground Floor.
a.Both Statements are true.
b. Both Statements are False.
c. Statement 2 is True while statement 1 is False.
d. Statement 1 is True while Statement 2 is False.
Page 89 of 204
Graphic Era Hill University, Dehradun
Q8 : If the Library is located on the 4th floor of the building, then which department
is located on the topmost floor?
a.
Acads
b. Lab
c. Marketing
d. Cannot be Determined.
Set 3 : 6 Experts from the field of Sociology, Philosophy, Science, Politics, Pub. Ad
and Geography are sitting on a rectangular table with 3 experts on each side. Each
expert has written a review on a book written by any one of the other 5 experts.
Further information is as follows:
1.
Any 2 people who are experts in fields starting with the same alphabet are
not sitting adjacent to each other.
2.
The Philosophy expert is sitting next to experts who have written reviews on
Politics and Sociology respectively.
3.
One of the 2 people sitting in the middle has reviewed the book on
Philosophy.
4.
The expert who has reviewed the book on Geography is sitting on the same
side as the Geography expert.
5.
None of the experts have reviewed their own book.
6.
The Geography expert has exactly one expert as his immediate neighbour
and has not reviewed the book written by the Sociology expert.
Q9 : Which expert has reviewed the Philosophy book?
a.
Sociology
b. Politics
c. Geography
d. Pub. Ad.
Q10 : Which of the following represents the correct position of the Science expert?
a.
Immediate left of the Geography expert.
b.
Opposite to the expert who has reviewed Philosophy.
c.
Immediate left of the expert who has reviewed Geography.
d.
Opposite to either the Politics or the Pub.Ad. expert.
Q11 : Which book has been reviewed by the Geography expert?
Page 90 of 204
Graphic Era Hill University, Dehradun
a.
Pub. Ad
b. Science
c. Philosophy
d. Politics.
Q12 : Which expert is opposite the Philosophy expert?
a.
Politics
b. Science
c. Sociology
d.
Cannot
be
Determined.
Set 4: 6 Experts M,N,O,P,Q and R belonging to the fields of Marketing, HR, Finance,
Operations, IT and IB are sitting across a circular table for a meeting such that
each expert is sitting opposite another expert. Each expert belongs to exactly one
of the mentioned fields. Further Information is as follows:
1.
The Marketing Expert is 3rd to the right of the Expert in IB.
2.
P is not an expert in either Finance or IT and is sitting opposite to R, the
Operations expert.
3.
Q, the expert in IB is to the immediate left of the HR expert.
4.
While N is an expert in IT, O is the Marketing expert.
5.
M is at a position which is not adjacent to the HR expert.
Q13 : The Marketing expert is sitting between experts from which of the areas?
a.
Finance and IB
d.
Operations and IT.
b. Operations and Finance
c. HR and Operations
Q14 : Who is seated Opposite the expert from IB?
a.
O
b. M
c. P
d. R
Q15 : Which of the following is true about the position of the HR expert?
a.
To the immediate left of O.
b.
3rd to the left of the Operations expert.
c.
3rd to the right of M.
d.
None of these.
Page 91 of 204
Graphic Era Hill University, Dehradun
Q16 : If a team of 3 members is formed consisting of experts sitting on consecutive
positions, which among the following cannot be a possible team formed?
a.
ONP
b. RON
c. RNP
d. MQP
Set 5 : 5 students M,N,O,P and Q took 2 different Tests each on Computer Funda
and Prog. Basics given by their teacher Vijaya and each one was given a rank by
the teacher based on the number of marks scored in the Test with the highest
marks getting Rank 1 and the lowest getting Rank 5. Rank 1 is considered higher
than Rank 2, Rank 2 is considered higher than Rank 3 and so on.
1.
Incidently, no 2 students scored the same marks in any of the tests.
2.
P got a higher Rank than O in Prog. Basics but a lower Rank than M in
Computer Funda.
3.
M is the only person who got the same Rank in both the Tests.
4.
Q got the highest Rank in Computer Funda while O got Rank 3 in Prog.
Basics.
5.
P did not get Rank 1 in any of the 2 Tests.
Q17 : What is the Rank obtained by N in Computer Funda?
a.
2
b. 3
c. 4
d. 5
Q18 : Who among the following got the lowest Rank in Prog. Basics?
a.
P
b. O
c. M
d. Q
Q19 : Who among the following got the same Rank in Prog. Basics as that obtained
by O in Computer Funda ?
a.
M
b. N
c. Q
d. None of These.
Q20 : What can be said about the average of the 2 Ranks obtained by P in the 2
Tests mentioned?
Page 92 of 204
Graphic Era Hill University, Dehradun
a.
Same as the average of the 2 Ranks obtained by M.
b.
Lower than the average of the 2 Ranks obtained by Q.
c.
Higher than the average of the 2 Ranks obtained by O.
d.
None of These.
Page 93 of 204
Graphic Era Hill University, Dehradun
Level 2
Arrangement Based 2
Set 1 : 6 students in a class were ranked on the order of the number of marks
received by them in a Test with Rank 1 been given to the child with the highest
marks and Rank 6 to the child with the lowest marks. Further information is as
under:
1. A is 4 ranks above E but E is not the lowest scorer in the test.
2. B and F have got 2 consecutive ranks in the test.
3. C is the 2nd highest scorer in the Test.
Q1. Who has got the 4th Rank in the Test?
a. B
b. F
c. E
d.Cannot be Determined
Q2. Who is the lowest scorer in the Test?
a. A
b. D
c. C
d. F
Q3. What can be said about the following 2 statements
a. There are at least 2 students who have scored better marks than F.
b.
F has scored better marks than exactly 2 students.
a.
Both statements are true.
b.
One of the 2 statements is definitely true while the other is false.
c.
One of the 2 statements is definitely true while the other may be true.
d.
Both statements are false.
Page 94 of 204
Graphic Era Hill University, Dehradun
Set 2 :
4 Practice tests have to be scheduled for the students of SLT batch at ACE Ltd. The
tests have to scheduled according to the following conditions:
1. Each day on which a Test is scheduled is called a slot.
2. The Tests have to be scheduled in the week starting on Monday and
ending on Sunday.
3. There has to be a gap of at least 1 day between any 2 slots.
4. The English Test cannot be scheduled after Thursday.
5. The Test of Maths will be the last Test to be conducted.
6. The Reasoning Test and the English Test cannot be conducted on 2
consecutive slots.
7. General Knowledge is the 4th area on which a Test is to be scheduled.
Q4. In how many different ways can the Tests be scheduled?
a. 1
b. 2
c. 3
d. 4
c. English
d.Cannot
Q5. Which is the first test to be scheduled ?
a. Maths
b. GK
be
Determined
Q6. On which day is the General Knowledge Test Scheduled ?
a. Monday
b. Wednesday
Determined
Page 95 of 204
c. Tuesday
d.Cannot be
Graphic Era Hill University, Dehradun
Set 3 : 4 people of the Gupta family Mr.Rakesh, his wife Rashmi, son Rohan and
daughter Raadhika and 4 people of the Goyal family Mr.Ritesh, his wife Roshni, son
Rahul and daughter Rashmi are sitting around a circular table for having dinner at a
family get together.
1.
Each person is sitting between 2 people of the opposite gender.
2.
Each child has at least 1 parent as his/her immediate neighbour.
3.
Rakesh and Ritesh are sitting directly opposite each other but neither of them
has his son sitting at the adjacent position.
4.
Rashmi is sitting 3rd to the left of Rakesh.
Q7. In how many different ways can the seating be done?
a.
1
b. 2
c. 3
d. 4
Q8. Which 2 children are sitting between their parents?
a.
Rohan and Raadhika
b. Rashmi and Rahul
c.
Rohan
and
Rahul
d.Raddhika and Rashmi.
Q9. Which of the following is definitely false regarding the position of Rashmi?
a.
2nd to the left of Ritesh.
b.
Diagonally opposite to Raadhika
c.
Diagonally opposite to Rohan
d.
3rd to the right of Rahul.
Set 4 :
6 people are living on 6 different floors of a 7 storey building. These 7 storeys also
includes the ground floor which is also the Floor no 1 of the building. The 6 people
Page 96 of 204
Graphic Era Hill University, Dehradun
are a Doctor, an Engineer, a lawyer, a Teacher, a Dentist and a Businessman by
profession not in any particular order. Further conditions are as under:
1.
The ground floor of the building is being used as a recreation club.
2.
2 teams were formed consisting of people living on Alternate floors. While
the Engineer and the Teacher were in one team, the other team had the
Businessman and the Dentist.
3.
The Dentist lives on the topmost floor.
4.
The teacher had only 1 family as his immediate neighbor and that family is
the Doctor’s family.
5.
The lawyer has more no of families living above him in the building as
compared to the Engineer.
Q10. Who lives on the 6th Floor?
a.
Lawyer
b. Engineer
c. Teacher
d.Cannot
be
Determined
Q11. How many floors are there between the Businessman and the Dentist?
a.
2
b. 1
c. 3
d. 4
Q12. Which of the following families are not immediate neighbours ?
a.
Doctor, Lawyer
b. Businessman, Engineerc. Dentist, Teacher d.None
these
Page 97 of 204
of
Graphic Era Hill University, Dehradun
Set 5 :4 boys Alok,Amit,Atul and Ashish and 4 girls Anu,Anita,Astha and Ankita are
sitting on opposite sides of a rectangular table, 4 on each side. Further Information
is as follows :
1.
Alok is 2nd to the right of Ankita and opposite to Ashish.
2.
Astha is sitting at one of the corners and does not have a girl opposite to her.
3.
Atul and Anita occupy adjacent positions.
4.
There are 2 people between Astha and Ashish.
Q13 : Who is sitting opposite Ankita ?
a.
Atul
b. Anita
c. Alok
d.Cannot be determined
Q14 : How many of the mentioned persons definitely have 2 people of the opposite
gender as their neighbours ?
a.
1
b. 2
c. 3
d. 4
c. Atul
d. Anita
Q15 : Which person is sitting opposite Amit ?
a.
Astha
b. Alok
Q16 : If a team is formed consisting of the 4 persons who are not sitting at any of
the corner positions, then the team will have
a.
2 girls, 2 boys
b. 1 girl, 3 boys
c. 3 girls, 1 boy
d. all 4 girls.
Set 6 : 5 students Anubhav, Bipin, Charu, Dheeraj and Eklavya were seated in a
row to take a Mock Test. Each person belonged to a different branch of Engineering
from CS,ME,EC,EE,and CE. Further conditions are as follows :
1.
Charu’s branch is EC and she is not seated at either ends.
2.
The person with CE specialization is 2nd to the right of Dheeraj.
Page 98 of 204
Graphic Era Hill University, Dehradun
3.
Bipin’s branch is neither ME nor CS.
4.
Ekalvya is at the extreme right of the arrangement.
5.
The person with ME specialization is at the middle position.
6.
The Electrical Engineer is to the immediate left of Bipin.
Q17 : Who is the person with EE specialization?
a.
Bipin
b. Eklavya
c. Anubhav
d. Cannot be Determined.
Q18 : What is the specialization of the person sitting 4th from the left of the
arrangement?
a.
EE
b. EC
c. CS
d.
Cannot
Determined
Q19 : Which of the following is true about the position of Ekalvya?
a.
To the immediate left of Dheeraj.
b.
2nd to the left of Anubhav.
c.
To the immediate left of Charu.
d.
2nd to the left of Bipin.
Q20 : Who is the person specializing in ME?
a.
Dheeraj
b. Bipin
c. Ekalvya
Page 99 of 204
d. Anubhav
be
Graphic Era Hill University, Dehradun
Level 2
Arrangements : Non Deterministic 1
Set 1 : 5 events were planned as a celebration of the F1 grand prix being
conducted in India. The 5 events were Metallica Concert, Lakme Fashion Show,
Laughter Mania, Quiz show and Dance night not in any order. The events were to
be scheduled one on each day from Monday to Friday of the week subject to the
following conditions:
1.
The Metallica Concert should not be the last event scheduled.
2.
Dance night and Quiz show should not be on consecutive days.
3.
Lakme Fashion show was scheduled on Thursday.
4.
The number of events that followed Laughter Mania was more than the
number of events that preceeded it.
Q1 : If the Metallica concert was scheduled after the Quiz show, when was the
Dance night scheduled?
a.
Tuesday
b. Friday
c. Wednesday
d.
Cannot
be
Determined
Q2 : If the Laughter Mania was scheduled on Tuesday, when was the Quiz show
scheduled?
a.
Monday
b. Wednesday
c. Friday
d.
Cannot
be
Determined
Q3 : How many of the mentioned events can possibly be scheduled on the first
day?
a.
2
b. 3
c. 4
Page 100 of 204
d. 5
Graphic Era Hill University, Dehradun
Q4 : If the Dance night is scheduled immediately before the Lakme Fashion show,
then which event was scheduled on Friday?
a.
Laughter Mania
b. QuizShow
c. Metallica
d.
Cannot
be
Determined.
Set 2 : 4 different eating joints Rakims, Kawala Inn, Chaldiram and Minaz are
located at 4 different places Pantheon Road, Najafgarh, Bada Bazaar and Rasoipur
in Delhi city. Each one specializes in exactly 2 items out of South Indian Platter,
Chicken Biryani, Kebabs and Sweets. Further conditions are as follows :
1.
No 2 eating joints specialise in the same 2 items.
2.
The outlet in Najafgarh specialses in Kebabs and Sweets.
3.
Rakims is located at Pantheon Road.
4.
Kawala Inn has exactly 1 item common with both Rakim and the outlet at
Najafgarh.
5.
Of the 2 outlets where Chicken Birayani is famous, one is definitely at Bada
Bazaar. Similarly, of the 2 outlets where South Indian platter is famous, one is
definitely at Rasoipur.
6.
Of all the possible combinations of each item with every other item, Kebab
only forms a combination with Sweets.
7.
Minaz is not famous for Sweets.
Q5 : What are the 2 items that Minaz is famous for?
a.
Kebabs, Chicken Birayani.
b.
South Indian Platter, Chicken Birayani.
c.
South Indian Platter, Sweets.
d.
Chicken Birayani, Sweets.
Q6 : If Kawala Inn is located at Rasoipur, the 2 outlets famous for Chicken Birayani
are ?
Page 101 of 204
Graphic Era Hill University, Dehradun
a.
Rakims, Minaz
b. Minaz, Kawala Inn
c. Kawala Inn, Chaldiram
d.Chaldiram,Rakims
Q7 : Where is Minaz located in Delhi?
a.
Pantheon Road
b. Bada Baazar
c. Rasoipur
d.
Cannot
be
Determined
Q8 : If Minaz is located at Bada Baazar, then what is Rakims famous for?
a.
Chicken Birayani, Sweets
b.
Kebabs, Sweets
c.
South Indian Platter, Sweets
d.
Chicken Birayani, South Indian Platter
Set 3 : 8 people A,B,C,D,E,F,G and H are sitting around a circular table at positions
numbered 1 to 8 in Clockwise direction and playing a game of passing the parcel.
The persons are not necessarily sitting in the same order. Each person on getting
the parcel will pass it to the next person in the following order of priority :
Priority 1 : Pass it to the person sitting opposite.
Priority 2 : Pass it to the person sitting 2 places to the right.
Priority 3 : Pass it to the person sitting to the immediate right.
Each person strictly follows the priority during the game ie a person will first see
priority no 1, then Priority 2 if priority 1 is not available and then priority 3 if both 1
and 2 are not available.
1.
If a person has got the parcel once, he or she cannot get the parcel for the
2nd time otherwise the game will get finished.
2.
The game started with C who is sitting at position No.3.
Page 102 of 204
Graphic Era Hill University, Dehradun
3.
H was the 3rd one to receive the parcel followed by B,E and G in the same
order.
4.
The Game did not finish till all of them got one chance each ie the Game only
finished when the 8th person received the parcel.
Q9 : The 7th person to receive the parcel was at position number
a.
4
b. 6
c. 7
d. 2
Q10 : If there are exactly 3 people sitting between A and F, who is sitting opposite
C?
a.
E
b. D
c. A or F
d. F
Q11 : If A gets the parcel sometime before H while D gets the parcel immediately
after G, then who is the last one to get the parcel?
a.
D
b. F
c. D
d. None of These
Q12 : Person at which position gets the parcel immediately after the person at
position No. 1?
a.
No. 5
b. No.8
c. No. 7
d. No. 6
Set 4 : 5 Contestants qualified for the finals of CKB contest. Their first names were
Lucky,Mohit,Nitin,Okif and Pramod while their Surnames were Gupta, Aggarwal,
Sharma, Bansal and Verma not in any particular order. Each of them finished on a
particular position from 1st to 5th. Also, no 2 people finished on the same position.
1.
Bansal got the 1st position amongst all the contestants.
2.
Verma got a better position than Gupta but lower than Okif.
3.
Mohit, whose surname was not Gupta had an equal number of contestants
who finished before and after him.
4.
Pramod got a better position than at least 2 other people.
Page 103 of 204
Graphic Era Hill University, Dehradun
5.
Okif finished at exactly 2 positions ahead of Nitin but Nitin was not ranked
5th.
Q13 : If Aggarwal finished at position No.4, then what was Okif’s Surname?
a.
Gupta
b. Sharma
c. Aggarwal
d. Verma
Q14 : What can be said about the following 2 statements?
1.
Aggarwal finished 3 positions ahead of Gupta.
2.
There was a gap of exactly 2 positions between Sharma and Verma.
a.
Both Statements are False.
b.
While Statement 1 is False, Statement 2 may be true.
c.
Both Statements may be true.
d.
While Statement 1 may be true, Statement 2 is definitely false.
Q15 : If Mohit’s Surname is Verma, then what is the Surname of the person who
finished at 5th place?
a.
Aggarwal
b. Sharma
c. Gupta
d. Cannot be Determined
Q16 : If Aggarwal is as much ahead of Gupta as Mohit is behind Bansal, then what
is the Surname of Nitin?
a.
Gupta
b. Sharma
c. Verma
d. Cannot be Determined
Set 5 : 5 companies were scheduled to visit GEU for placement during a particular
week. The companies were Infy, Cogni, Wipro, Accen and Infogain and would visit
one on each day from Monday to Friday, not in any particular order.
Besides Aptitude, each company would test the Technical skills of the students in
exactly 1 area out of C, C++, Datastructure, Database and .Net. Further conditions
are as follows:
Page 104 of 204
Graphic Era Hill University, Dehradun
1.
If Infy visits on Monday, the Company testing in .Net would not visit before
Wednesday.
2.
Accen will test the student’s skills in C++.
3.
Cogni will visit on Thursday and will test skills in either Database or C.
4.
The Company testing in Datastructure will visit on Friday.
5.
The first Company to visit will be either Infogain or Infy.
6.
If Infogain visits on Monday, Accen will visit immediately before Infy.
Q17 : If Infogain is the first Company to visit, then which Company will test skills in
Datastructure?
a.
Accen
b. Infogain
c. Infy
d. Wipro
Q18 : On which day will Accen visit for placement?
a.
Wednesday
b. Tuesday
c. Thursday
d. Friday
Q19 : If Wipro is certain to visit on the last day, the Company visiting on
Wednesday will test what skills?
a.
Database
b. C
c. .Net
d.
Cannot
be
Determined
Q20 : If Infy is the first company to visit, then the Company testing in .Net will visit
on which day?
a.
Tuesday
b. Wednesday
c. Thursday
Page 105 of 204
d. Friday
Graphic Era Hill University, Dehradun
Level 2
Arrangement : Non Deterministic 2
Set 1 : A Test was given to 5 students which required each one of them to attempt
all the Questions. The Test had a total of 5 questions with each correct answer
worth 5 Marks. However, there was a penalty of 2 Marks for giving a wrong answer.
Incidently, each person got different numbers of correct answers in the Test with
everyone getting at least one question correct.
Further conditions are as under:
1.
C scored 18 Marks in the Test.
2.
The number of Correct answers of E was more than that of A.
3.
The number of wrong answers of B is less than the number of wrong answers
of D.
4.
The number of correct answers obtained by E was equal to the sum total of
the number of correct answers obtained by A and B together.
Q1: What is the total marks scored by E in the Test?
a.
11
b. 25
c. 4
d. None of These
Q2 : How many Questions answered by A were correct?
a.
3
b. 2
c. 1
d. Cannot be Determined
Q3 : Which among the following can be the average marks scored by B and D
together?
a.
1.5
b. 6
c. 4
d. 5
Q4 : If A scored more than B in the Test, the number of questions correctly
answered by B is?
Page 106 of 204
Graphic Era Hill University, Dehradun
a.
2
b. 3
c. 1
d. Cannot be Determined
Set 2 : Sanjay was at the New Delhi station waiting to take a Metro Train. There
were 5 Trains scheduled in the next 50 minutes each one on a different line and
going to a different destination. The destinations were Airport, ISBT, Rohini,
Dwarka and Badarpur not in any particular order. Also, they were on The Red, Blue,
Yellow, Silver and Black lines not in any particular order.
1.
The destination of the first train is Dwarka, and the Yellow line will be the last
train to arrive.
2.
Trains going to Airport and ISBT will arrive immediately one after the other
not in any order.
3.
The Blue line will be going to Rohini and the yellow line to the Airport.
4.
The Red line is not the first Train to arrive. Also, it’s destination is not ISBT.
Q5 : Which Metro line is scheduled to go to Badarpur?
a.
Yellow
b. Silver
c. Red
d. Black
Q6 : If the Red Metro line train is scheduled immediately before the Silver Metro
line train, then what will be the destination of the Black Metro line train?
a.
Dwarka
b. ISBT
c. Airport
d. Badarpur
Q7 : Which 2 Metro line trains cannot be scheduled to depart immediately one after
the other?
a.
Dwarka and Badarpur.
b.
Blue and Silver.
c.
ISBT and Dwarka.
d.
Red and Black.
Q8 : If there are exactly 2 trains scheduled between the Yellow line and the Red
line trains, then what will be the destination of the Silver line train?
Page 107 of 204
Graphic Era Hill University, Dehradun
a.
Airport
b. Dwarka
c. ISBT
d.
Cannot
be
Determined
Set 3 : 8 people P,Q,R,S,T,U,V and W are playing a game sitting around a circular
table. The game is such that 4 people are facing the table while the other 4 are
sitting away from the table. Also, the 4 people sitting towards the table are at
alternate positions and the same is true for the people sitting away from the table.
1.
S is sitting towards the table and is opposite to Q.
2.
T is 3 places to the left of Q.
3.
R is 2nd to the right of T and to the immediate right of V.
4.
P and W are not at adjacent positions.
Q9 : If P is directly opposite R, What can be said about the position of W?
a.
3rd to the left of P.
b.
2nd to the right of R.
c.
Opposite to S.
d.
To the immediate left of Q.
Q10 : How many possible positions can U occupy?
a.
1
b. 2
c. 3
d. None of These
Q11 : If W is directly opposite to T, who is sitting to the immediate left of P?
a.
T
b. U
c. Q
d. Cannot be Determined
Q12 : Which of the following can never be true about the position of P?
a.
2nd to the left of U.
b.
To the immediate right of Q.
c.
3rd to the right of R.
d.
Opposite to T.
Page 108 of 204
Graphic Era Hill University, Dehradun
Set 4 : Mr.Mittal, the course co-ordinator had to arrange faculties during a
particular week to teach Maths on an adhoc basis to the first year students. The
classes were to be scheduled on 4 different days of the week from Monday to
Saturday covering the topics of Discrete Maths, Finite Automata, Numerical Analysis
and Integration not in any particular order.
1.
The faculties assigned to take classes were Mr.Anand, Mr.Lalit, Mr.Vishal and
Ms. Gaur.
2.
Each of the faculties would take exactly one topic out of the mentioned 4
topics.
3.
Finite Automata would be scheduled after Integration but before Discrete
Maths.
4.
Of the remaining 2 days, one would be a holiday while the other would be
used to schedule a test.
5.
Numerical Analysis is the first topic to be scheduled.
6.
The Test can be conducted only after completion of at least 2 of the
mentioned topics.
7.
While Anand teaches Discrete Maths, Vishal is not the first among the
faculties to take a class.
8.
Lalit will take a class only after Ms.Gaur has taken a class.
Q13 : On which of the following days can the class of Ms.Gaur be scheduled?
a.
Tuesday
b. Wednesday
c. Thursday
d. Friday.
Q14 : If the holiday is on Tuesday and the Test on Saturday, who will take the class
on Thursday?
a.
Vishal
b. Lalit
c. Anand
d. Cannot be
Determined.
Q15 : If the Test is on Thursday and Vishal takes a class exactly 2 days before the
holiday, what is the topic taught by Lalit?
Page 109 of 204
Graphic Era Hill University, Dehradun
a.
Integration
b. Finite Automata
c. Numerical Analysis
d.
Discrete Mathematics.
Q16 : If the holiday is on Wednesday and there is a gap of exactly 1 day between
Finite Automata and Discrete Maths, then on which day is the Test scheduled?
a.
Thursday
b. Friday
c. Saturday
d.
Cannot
be
Determined.
Set 5 : The Captains of the participating teams in Cricket World Cup had assembled
to get the formal photo clicked. The teams were India, Sri Lanka, Australia, New
Zealand, Pakistan, South Africa, West Indies and England and the captains of the
teams were Mhoni, Blarke, Wammy, Smith, Pettori, Bismah, Hilshan, Alistar not in
any order.
Further conditions are as follows :
1.
They were to be arranged in 2 rows facing the Camera. The first one is called
the Front row, while the one behind it is called the second Row.
2.
While India, Sri Lanka and Pakistan represent Asia, the other teams were
said to be a part of Rest of the world.
3.
Each row will have representations of both Asia as well as Rest of the world.
4.
India is the only Asian country not represented in the 2nd row.
5.
All indicated positions are from the left when facing the arrangement.
6.
Alistar, the South African Captain was 3rd in the 2nd row.
7.
India’s captain was at the rightmost end of the arrangement.
8.
Bismah, the England captain was directly behind Smith the captain of West
Indies.
9.
Alistar was seated between Mhoni and Blarke.
10.
The number of persons between Pettori and Smith was same as that between
Wammy and Hilshan.
Page 110 of 204
Graphic Era Hill University, Dehradun
Q17 : If Blarke is seated behind Pettori, who is the sitting directly behind the Indian
captain?
a.
Bismah
b. Mhoni
c. Alistar
d.
Cannot
be
Determined
Q18 : Which of the following can be true?
a.
Smith is the captain of India.
b.
Alistar is sitting directly behind the Indian captain.
c.
Blarke is the captain of Pakistan.
d.
The captains of Pakistan and Srilanka are seated adjacent to each other.
Q19 : If the captains of West Indies and Australia are adjacent and Wammy is next
to the Indian captain, who is the captain of Australia?
a.
Pettori
b. Wammy
c. Hilshan
d. Cannot be
Determined.
Q20 : At how many possible positions can Pettori be seated?
a.
1
b. 2
c. 3
positions.
Page 111 of 204
d.
All
4
Graphic Era Hill University, Dehradun
Level 2
Tabular/Grid based 1
Set 1 : Mr.Sharma took his 5 children to a party one evening. Each child selected a
snack and a cold drink. The Snacks were Burger, Hot Dog, Pizza, Patties and
Sandwich and the cold drinks were Cepsi,Poke,Liranda,Mimca and Up-7. Further
conditions were as follows :
1.
While Burger,Hot Dog and Pizza are classified under Chilly snacks, Patties
and Sandwich are non Chilly snacks.
2.
Reyu had Mimca and a Chilly snack.
3.
Person who had Cepsi choose a non-chilly snack.
4.
Itisha’s snack was not in the same category as Utkarsh.
5.
Aditya had a Sandwich.
6.
No 2 children choose the same snack or cold drink.
7.
Farhan choose Hot Dog and Up-7.
8.
The kid who choose Patties did not choose Cepsi.
9.
Utkarsh choose Poke which does not go well with a non-chilly snack.
Q1 : What was the Snack choosen by Reyu?
a.
Burger
b. Patties
c. Hot Dog
d.
Cannot
c. Utkarsh
d. Farhan
be
Determined
Q2 : Which child choose Liranda?
a.
Itisha
b. Reyu
Q3 : Which other Child(ren) choose the Snack from the same category as Farhan?
a.
Itisha,Utkarsh
b. Reyu
c. Aditya,Reyu
Q4 : What was the snack choosen by Itisha?
Page 112 of 204
d. Reyu,Utkarsh
Graphic Era Hill University, Dehradun
a.
Patties
b. Sandwich
c. Burger
d. Pizza
Set 2 : 8 people were booked to attend a conference on the East and West wings of
the ROYALE hotel. While the Rooms numbers of the East wing are 17,18,19 and 20
those on the West wing are 21,22,23 and24. Further conditions are as under:
1.
A and H are booked in adjacent rooms but none of their room numbers is a
prime number.
2.
D’s room is a prime number but not on the same wing as A.
3.
F is the neighbour of B and there are exactly 2 rooms between F and D.
4.
If C and E exchange their rooms then E will have H as a neighbour.
5.
Each person is allotted one room.
Q5 : Which room number has been allotted to A?
a.
21
b. 22
c. 23
d. 24
Q6 : Who has been allotted room number 18?
a.
D
b. E
c. C
d. Cannot be Determined
Q7 : Who among the following has been given a room which is a prime number?
a.
E
b. H
c. F
d. G
c. C and H
d. C and B
Q8 : Who are the neighbours of F?
a.
G and B
b. B only
Set 3 : 5 types of cookies have been placed on a shelf with each type of cookie in a
different jar. The Cookies are Krackjack, Britannia, Parle-G, Oreo and Cadbury Bite
and the number of cookies of each type is 2,3,5,7 and 9 not in any order. 5
Children Albert,Bony,Cia,Dimple and Ekta were asked to choose exactly 1 type of
cookie. Further conditions are as follows :
Page 113 of 204
Graphic Era Hill University, Dehradun
1.
The Children choose the cookies in the alphabetical order of their names
starting with Albert.
2.
The person who chose last, took the jar of Oreo but not the maximum
number of cookies.
3.
The Jar containing Cadbury Bite had 3 Cookies.
4.
If Cia had choosen Krackjack, she would have had the maximum number of
cookies.
5.
The 4th child in order of choosing cookies took the jar having 5 cookies.
6.
The minimum number of cookies of any type were Parle-G.
7.
Bony choose the jar with more cookies than Cia but less than Albert.
Q9 : Who choose the Jar having Krackjack?
a.
Bony
b. Albert
c. Cia
d. Ekta
Q10 : Which cookie was choosen by Dimple?
a.
Parle-G
b. Oreo
c. Britannia
d. Cannot be Determined
Q11 : What can be said about the following 2 statements?
1.
The number of Cookies in the jar containing Britannia was 5.
2.
Ekta choose the jar having Oreo.
a.
Both Statements are true.
b.
Both Statements are false.
c.
Statement 1 is true while statement 2 is false.
d.
Statement 1 is false while statement 2 is true.
Q12 : The jar containing Oreo had how many cookies?
a.
7
b. 9
c. 5
d. 3
Set 4 : The top 5 finishers in the AIRTEL Indian Grand Prix were being
photographed. Their names were Bernie, Vijay, Narain, Schumi and Fernando not in
Page 114 of 204
Graphic Era Hill University, Dehradun
any order. They were representing 4 teams Mclaren, Force India, Ferrari and Honda
such that each team had at least 1 representation in the top 5 positions. Out of the
top 5 positions, the top 3 positions are popularly known as Podium positions. The
players were wearing T-shirts with numbers 7, 10, 15, 18 and 21 not in any
particular order. Further conditions is as under:
1.
Starting from the left, the players were occupying positions in the order in
which they finished the race ie 1st placed person occupied the 1st place and so on.
2.
Fernando is not from Honda and had only 1 person sitting next to him.
3.
The person with number 18 occupied the 4th position.
4.
Team Ferrari had 2 drivers in podium positions.
5.
Narain represents Mclaren and was placed 3rd in the Race.
6.
Bernie wearing number 7 finished 5th in the race.
7.
While the T-shirt number of one of the Ferrari drivers was greater than that
of Vijay, the T-shirt number of the other was lower than that of Vijay.
Q13 : If Bernie represents Force India, then which team is represented by Vijay?
a.
Honda
b. Ferrari
c. Mclaren
d. Force India
Q14 : If Schumi wears T-shirt number 15, what is the number worn by Fernando?
a.
10
b. 18
c. 21
d.
Cannot
be
c. Bernie
d.
Cannot
be
Determined
Q15 : Which driver represents Team Honda?
a.
Schumi
b. Vijay
Determined
Q16 : If the T-shirt number of Narain is between that of the 2 Ferrari drivers, then
what is the number worn by Narain?
Page 115 of 204
Graphic Era Hill University, Dehradun
a.
15
b. 10
c. 21
d.
Cannot
be
Determined
Set 5 : 6 students of a class were rated as Excellent (E), Very Good (VG) or Good
(G) in 3 different sports Cricket, Football and Basketball. Each one was definitely
rated in all 3 Sports. Excellent (E) is a better rating than Very Good (VG) which in
turn is better than Good (G). The following table provides partial information about
the ratings given to the students.
RATINGS IN SPORTS
STUDENT
NAME
CRICKET FOOTBALL BASKETBALL
A
B
E
G
C
D
VG
VG
E
F
1.
F is the only student to have got the same rating in all the 3 sports.
2.
Both D and E obtained the same rating in Football.
3.
The rating obtained by A in Cricket is same as the rating given to C in
Basketball. Also, A and D were the only 2 students to get all the 3 ratings.
4.
Equal number of students got the 3 different ratings in Cricket. The same is
also true for both Football and Basketball.
5.
E got a better rating than A in both Cricket and Football.
Page 116 of 204
Graphic Era Hill University, Dehradun
Q17. What is the Rating obtained by A in Football?
a.
G
b. VG
c. E
d.Cannot
be
Determined
Q18. What is the sequence of the ratings given to student E in Cricket, Football and
Basketball respectively ?
a.
E,E,VG
b. E,E,E
c. G,E,E
d. VG,VG,G
Q19. What is the rating given to F in Football?
a.
G
b. VG
c. E
d.Cannot
Determined
Q20. Which students got the rating “Excellent” in Basketball ?
a.
E and C
c. A and C
c. A and E
Level 2
Tabular/Grid Based 2
SET 1 :
Page 117 of 204
d. C and D
be
Graphic Era Hill University, Dehradun
Certain number of cookies were distributed to 5 children such that each child
received a distinct no. of cookies from Cadbury, Alpenliebe, Lemon, Mangobite and
Paspas. Further information is as follows :
1.
The average number of cookies with the 5 children was 20 which was also
the number of cookies received by Aryan.
2.
Reyu, the child who got Alpenleibe got the 2nd highest number of cookies.
3.
Each child received cookies of only one type and the number of cookies
that each one got were consecutive numbers.
4.
Mehuli got neither the highest nor the least number of cookies.
5.
The number of lemons were more than Mangobite and Paspas but less
than Alpenliebe and Cadbury.
6.
Danu, the eldest among all children got the least number of cookies which
wasn’t paspas.
7.
Medhu got the highest number of cookies among all the children.
Q1. What is the number of cookies got by Danu ?
a. 18
b. 19
c. 21
d. 20
Q2. Which child got the cookie lemon?
a. Mehu
b. Danu
c. Aryan
d. Medhu
Q3. What is the average number of cookies received by Aryan and Medhu ?
a. 20
b. 21
c. 19
determined
Q4. Which cookie was obtained by Danu ?
Page 118 of 204
d.Cannot
be
Graphic Era Hill University, Dehradun
a. Mangobite
b. Lemon
c. Cadbury
d.Cannot be determined
Set 2 :
8 Teams, India, England, South Africa, Pakistan, Australia, Sri Lanka, New Zealand
and West Indies have qualified for the Quarter Finals of World Cup 2011. The
Matches have to be played according to the following order:
Quarter Final 1
:
B4 v/s A3
Quarter Final 2
:
B2 v/s A1
Quarter Final 3
:
B3 v/s A4
Quarter Final 4
:
B1 v/s A2
B4 indicates that the team is from group B and is in the 4th position in the group to
qualify for the Quarter Finals.
Further conditions are as under:
1.
West Indies are the defending Champions because of being the winners of
the previous World Cup in 2007.
2.
India will play in Quarter Final 2 but not against South Africa or West
Indies.
3.
Pakistan has got the 4th Position in Group A after the league matches.
4.
New Zealand and Sri Lanka have got the top 2 positions in group B after
the league matches in the same order.
5.
The 3rd Quarter Final is a match between 2 teams neither of whom have
won the world cup earlier.
6.
South Africa and West Indies are in group B.
Page 119 of 204
Graphic Era Hill University, Dehradun
7.
New Zealand will play arch rivals Australia in the Quarter Finals.
Q5. Which 2 teams have definitely not won the world cup earlier ?
a. Australia & New Zealand
Zealand & South Africa
b. India and Sri Lanka
c.
New
d. South Africa & Pakistan
Q6. How will the position of Sri Lanka after the league matches be indicated ?
a. B2
b. A4
c. B3
d. A1
Q7. Which team will have position A2 after the league matches ?
a. England
b. Australia
c. India
d. West Indies
Set 3 :
5 people Alpha, Beta, Gamma, Delta and Theta went for shopping and each
purchased 1 pair of Trousers and a T-Shirt for himself/ herself. The following table
gives the prices in Rs. of the various brands for the pair of Trousers and the T-Shirt
separately.
Brand
Trousers
T-Shirt
Levis
300
250
Numero Uno
450
300
Addidas
250
250
Reabok
500
500
Pepe
1.
400
700
All of them purchased Trousers of different brands. The same is true for
the T-shirts purchased.
Page 120 of 204
Graphic Era Hill University, Dehradun
2.
The amount of money spent by Delta is twice the amount spent by Alpha.
3.
Exactly 2 people purchased both the items of the same brand.
4.
The price of the Trousers and the T-Shirt purchased by Alpha was the
same but this was not true for Delta.
5.
Exactly one of Gamma or Theta purchased both items of different brands.
6.
Beta spent a lower amount in purchasing a T-shirt of the same brand as
trousers but overall a higher amount as compared to Theta.
Q8. Who purchased the Numero Uno Trousers?
a. Alpha
b. Beta
c. Gamma
d.Cannot
be
Determined
Q9. What is the total amount spent by Delta?
a. Rs. 1200
b. Rs. 1000
c. Rs. 1250
d. Rs. 1100
Q10. What can be said about the following 2 statements
1.
Delta purchased Trousers of higher value than the T-shirt .
2.
Theta purchased Trousers of higher value than the T-shirt
a.
Both statements are true.
b.
Both statements are false.
c.
Statement 1 is False while Statement 2 is true.
d.
Statement 2 is False while Statement 1 is true.
Q11. Which combination related to the pair of trousers purchased by Theta is
correct?
Page 121 of 204
Graphic Era Hill University, Dehradun
a. Pepe, 250/-
b. Addidas , 300/- c. Pepe, 400/-
d.
Levis,
300/Set 4 : 6 students appeared for the TAC exam and got percentiles of
90,95,99,93,97 and 92 not in any particular order. They also got calls for admission
from the 6 MIIs, the top institutes for MBA in the country.
Further conditions are as under:
1.
Each person got at least 1 call and no 2 people got the same number of calls.
2.
A got a percentile of 97 and total number of calls which is a prime number.
3.
The number of calls received by both E and F is more than C but less than
the number of calls received by A.
4.
The person with the highest percentile did not get the maximum number of
calls.
5.
D got 90 percentile and calls from exactly 2 institutes.
6.
Neither C nor F got the maximum percentile among the 6 students.
7.
B has got a better percentile than at least 3 people.
Q12. Who scored the maximum percentile among the 6 students?
a.
C
b. E
c. F
d. B
Q13. What is the number of calls received by F ?
a.
3
b. 4
c. 5
d.Cannot
be
determined
Q14. If arranged in descending order of percentiles, which student would rank 5th
among these students?
Page 122 of 204
Graphic Era Hill University, Dehradun
a.
Either A or C b. Either C or E
c. Either E or F
d. Either C or F
Set 5 :
M and P transporters have to ship 5 items of different value to destination D. The
items are valued at Rs. 15,000 , Rs. 20,000 , Rs.25,000 , Rs.30,000 and Rs.35,000
in no particular order and have to be picked from 5 different locations Alg., Bsr.,
Hpr., Mrt., and Ddn. In no particular order.
1.
The first item will be picked up at 9 am and thereafter every subsequent item
will be picked up at a gap of 2 hours each.
2.
M and P plans to start at 9 am from Mrt. where the value of the item is more
than the item picked up last.
3.
The item worth Rs.25,000 is picked up at 3 pm.
4.
The value of the item at Hpr. is more than the one picked up first but lower
than the item at Bsr., which is picked up at 1pm.
5.
The item at Ddn. is not picked up last.
Q15. If the item to be picked up at Ddun. is lower in value than the one picked up
at Hpr., then at what time is the item at Hpr. picked up ?
a.
11 am
b. 1 pm
c. 3 pm
d.Cannot be Determined
Q16. What is the value of the item picked up at Mrt.?
a.
30,000
b. 25,000
c. 20,000
d.Cannot
be
Determined
Q17. When will the item at Hpr. be picked up?
a.
11 am
b. 3 pm
c. 5 pm
Page 123 of 204
d.Cannot be Determined
Graphic Era Hill University, Dehradun
Set 6 :
6 people placed bets on India winning it’s matches in the preliminary stage of world
cup 2011. Each person placed bets on India winning a different number of matches
from 0 to 6. If a person places a bet on India winning a particular match and India
wins the Match, the person will get 3 times the betting amount which happens to be
Rs.150 per match. On the other hand, if India loses the match the amount will get
forfeited. In case of a tie or no result due to any other reason, the money bet is
given back.
1.
The 4th Match between India and England was a tie.
2.
While E did not place a bet on any match, C had a net profit of Rs.900/-.
3.
F spent a total amount of Rs.600/- for betting but did not bet on the 1st and
last match of the league phase both of which was won by India.
4.
There was exactly 1 match won by India in which everyone except E had
placed their bets. Also, no person bet on India winning all it’s matches.
5.
The number of matches won by India in the 2nd part of the league (matches
4/5/6) was more than that won in the 1st part of the league(matches 1/2/3).
6.
B placed the maximum number of bets among the 6 persons. Also, there was
exactly 1 match in which both B and F did not bet.
Q18. The Match won by India in which everyone except E placed a bet is?
a.
Match No 4 b. Match No 5
c. Match No. 6
No.3
Q19. What is the profit made by A by betting?
a.
No profit, no loss
b. Loss of Rs.150/-
Page 124 of 204
c. Profit of Rs.150/
d.
Match
Graphic Era Hill University, Dehradun
d.Cannot be Determined
Q20. Which of the following is a complete and exhaustive list of the matches lost by
India?
a.
1st,2nd,3rd
b. 2nd,3rd,4th
c. 2nd,3rd
Page 125 of 204
d. 1st,3rd,4th
Graphic Era Hill University, Dehradun
Level 2
Grouping / Condition based Exercise 1
Set 1 : For the main exam of SAI, a student has to choose exactly 5 subjects to
appear. The subjects have been divided into 2 Groups containing 5 subjects each
and the student has to select at least 2 subjects from each group.
Group 1
: Physics, Chemistry, Maths, English and Social.
Group 2 : Psychology, Sociology, Political Science, Public Administration and
Accounts.
Further conditions are as follows :
1.
Physics and Public Administration cannot be choosen together.
2.
If Maths is choosen, Sociology should be definitely choosen.
3.
If 3 subjects are choosen from Group 1, Social should be definitely choosen.
4.
If Chemistry is choosen, Physics should not be choosen.
5.
At least 1 subject from Psychology and English must be choosen.
Q1 : If 3 Subjects are choosen from Group 1 including Physics and Maths, then
which of the following subjects must definitely be selected?
a.
Political Science
b. Psychology
c. Accounts
d.Public
Administration
Q2 : Which of the following combination is not possible?
a.
Physics, Maths, Sociology, Psychology, Accounts.
b.
Chemistry, Maths, Public Administration, Accounts, English
c.
English, Maths, Sociology, Social, Psychology
d.
Psychology, Political Science, Accounts, Physics, Social
Page 126 of 204
Graphic Era Hill University, Dehradun
Q3 : If Physics is choosen while English and Sociology are not choosen, then how
many combinations are possible?
a.
4
b. 3
c. 2
d. 1
Q4 : If Physics and English are the only subjects choosen from Group 1, then which
among the following is definitely selected?
a.
Sociology
b. Public Administration
c. Accounts
d.
Cannot be Determined
Set 2 : Little Kunju was in a dilemma. His Dog Bruno had recently delivered 6
puppies. They had been named as Rocket, Tuffy, Gypsy, Chilli, Steffi and Daffy. The
Puppies had to be placed in 2 cages with each cage having exactly 3 puppies.
1.
If Daffy is in cage 1, Gypsy and Chilli will not be together.
2.
If Rocket is in cage 2, Chilli will be in cage 1.
3.
Steffi cannot be in the same cage as Tuffy.
4.
If Rocket is in cage 1, Tuffy will also be in cage 1.
Q5 : If Rocket is in cage 1, which of the following is definitely false?
a.
Steffi is in cage 2.
b.
Gypsy is in cage 1.
c.
Daffy is in cage 2.
d.
Daffy is in cage 1.
Q6 : If Daffy is in cage 1, how many different arrangements are possible?
a.
1
b. 2
c. 3
d. 4
Q7 : If Chilli is in cage 2, which of the following is definitely true?
a.
Gypsy is in cage 2.
b.
Daffy is in cage 1.
Page 127 of 204
Graphic Era Hill University, Dehradun
c.
Tuffy is in cage 2.
d.
Daffy is in cage 2.
Q8 : If all conditions are followed, how many possible arrangements exist?
a.
4
b. 5
c. 6
d. 7
Set 3 : MSD, the Indian captain had to select the team of 11 players for the next
match. He had an option to play 6 batsmen and select any 5 bowlers from Bhaj,
Sree, Patel, Khan, Ravi, Chawla, Ashish or play 7 Batsmen in the form of an
additional Batsman from either Suresh or Yusuf and select any 4 bowlers from the 7
bowlers available for selection. Out of the 7 bowlers Bhaj, Ashish, Ravi and Sree are
fast bowlers while Chawla, Patel and Khan are spinners. Even though the game has
allrounders, none of the Batsmen can be counted as bowlers or the bowlers
considered as Batsmen. Further conditions are as under:
1.
For every 2 fast bowlers selected, a spinner has to be definitely selected in
the team.
2.
If Suresh is selected, neither Ravi nor Khan can be selected.
3.
Bhaj will play only if Chawla is also selected.
Q9. Which of the following cannot be the combination of the team?
a.
6 Batsmen, Yusuf, Bhaj, Sree, Chawla, Khan
b.
6 Batsmen, Ashish, Chawla, Patel, Bhaj, Sree
c.
6 Batsmen, Bhaj, Sree, Chawla, Ashish, Ravi
d.
6 Batsmen, Suresh, Sree, Patel, Ashish, Chawla
Q10. If Suresh is selected as the 7th Batsman, in how many different ways can the
team be formed?
Page 128 of 204
Graphic Era Hill University, Dehradun
a.
4
b. 3
c. 2
d. 1
Q11. If MSD decides to play with 6 Batsmen and Sree gets injured and therefore
not eligible for selection, who is definite to get selected?
a.
Chawla
b. Bhaj
c. Patel
d. Ravi
Set 4 : 3 Friends Aman, Bala and Chander go out for having food. Each friend will
order exactly one of the items from Pizza, Burger and Sandwich subject to the
following conditions:
1.
If Aman orders Burger, Bala will order the same item as ordered by Chander.
2.
If Bala does not order Pizza, Chander will order the same item as ordered by
Aman.
3.
If Chander orders Sandwich, all three of them will order different items.
Q12 : If Aman orders Burger, then the item ordered by Bala will be?
a.
Burger
b. Pizza
c. Sandwich
d.
Cannot
be
Determined.
Q13 : If Bala orders Burger, how many Items can Chander possibly order from?
a.
One
b. Two
c. Three
d.
Cannot
be
Determined.
Q14 : If Bala orders Sandwich while Chander orders Pizza, then the item ordered by
Aman will be?
a.
Pizza
b. Burger
c. Sandwich
d. Any one of the
three items.
Q15 : If Chander orders Sandwich, then in how many different combinations can
the 3 friends order their items?
Page 129 of 204
Graphic Era Hill University, Dehradun
a.
3
b. 0
c. 2
d. 1
Set 5 : A team of teachers has to be choosen out of 8 available teachers to
represent the university at the Syllabus Council meet. The 8 teachers are Rawat,
Abhishek, Vinod, Tarun, Kapil, Shalini, Vatsala and Prateek. The selection must
follow the additional conditions :
1.
The team must have a minimum of 4 members.
2.
If Rawat is selected, then Vatsala has to be definitely selected.
3.
Exactly one from Prateek or Tarun has to be selected.
4.
Shalini and Rawat cannot be selected together.
5.
Kapil and Shalini have to be selected together.
Q16 : If Rawat is a member of a team of 4 members, in how many ways can the
team be selected?
a.
2
b. 3
c. 4
d. 5
Q17 : If Shalini is selected, what can be the maximum number of members in the
team?
a.
5
b. 6
c. 4
d. 7
Q18 : Who cannot be a member of a team of 6 members?
a.
Kapil
b. Tarun
c. Abhishek
d. Rawat
Q19 : Who among the following cannot be members of the same team?
a.
Rawat, Vinod
b. Kapil, Abhishek
c. Kapil, Rawat
Vinod
Q20 : Which among the following is not an acceptable combination?
a.
Rawat, Vinod, Abhishek, Tarun, Vatsala.
Page 130 of 204
d.
Vatsala,
Graphic Era Hill University, Dehradun
b.
Vinod, Abhishek, Prateek, Vatsala, Shalini.
c.
Shalini,Kapil, Vatsala, Tarun
d.
Tarun, Vatsala, Shalini, Kapil, Vinod, Abhishek.
Level 2
Grouping and Condition Based Exercise 2
Set 1 : The Curator of the pitch at Ferozeshah Kotla ground in Delhi was in a
dilemma. He was to re-lay the pitch in the next 6 days starting Monday before the
Test Match. There are 3 activities in preparing a Pitch Filling, Rolling and Watering.
Further conditions are as follows:
1.
He would like each of these activities to be done twice during the next 6
days. Each activity will require exactly 1 expert to be involved.
2.
Pitch Filling has to be the first activity and will be definitely done on Monday.
3.
Except for Filling, no other activity can be done for 2 consecutive days.
4.
Rolling cannot be done unless both the filling activities are completed.
There are 4 people P,Q,R and S to do these tasks but having expertise in certain
areas only. While P is an expert in both Filling and Rolling, Q is an expert in
Watering and Rolling. Of the remaining 2 people, R is an expert in Watering only
and S is an expert in filling only.
5.
An expert cannot work on consecutive days.
Q1 : The earliest day on which the first Rolling can happen is ?
a.
Tuesday
b. Wednesday
c. Thursday
d. Friday
Q2 : If R works on Tuesday, which expert will work on Thursday?
a.
P
b. Q
c. S
Determined.
Page 131 of 204
d. Cannot be
Graphic Era Hill University, Dehradun
Q3 : If P works on the 2nd day and R works on Thursday, which activity is done on
Wednesday?
a.
Watering
b. Filling
c. Rolling
d. Cannot be
Determined.
Q4 : If each activity is scheduled for exactly 2 days during the week, in how many
ways can the activities be scheduled?
a.
4
b. 3
c. 2
d. 1
Set 2 : Wiger Toods, the champion golfer was preparing his return to full time
competitive golfing. His trainer had arranged for him to practice 6 aspects of his
game to be practiced 2 on each day for 3 consecutive days, Wednesday, Thursday
and Friday during the week. The aspects were Swing, Stance, Putt, Plan, Push and
Drive. Further conditions were as under:
1.
There will be 2 activities practiced each day, one in the morning and the
other in the afternoon.
2.
If Swing is practiced in the morning on a day, then Putt cannot be practiced
on the immediate following day.
3.
Plan has to be practiced in the afternoon but not on Thursday.
4.
Stance has to be practiced in the afternoon and Drive on Friday morning.
Q5 : If Swing is practiced on Wednesday and Putt on Thursday, then the 3 aspects
practiced in the morning are?
a.
Push, Stance, Drive
b.
Push, Putt, Drive
c.
Swing, Putt, Drive
d.
Swing, Stance, Drive
Q6 : If Swing is practiced on Wednesday morning, then the aspect practiced on
Thursday morning is?
Page 132 of 204
Graphic Era Hill University, Dehradun
a.
Push
b. Plan
c. Putt
d.
Cannot
be
Determined.
Q7 : If Stance is practiced on Friday, then the aspect practiced on Wednesday
morning is ?
a.
Putt
b. Push
c. Plan
d.
Cannot
be
Determined.
Q8 : If Putt is practiced on Friday, in how many ways can the practice aspects be
scheduled?
a.
1
b. 2
c. 3
d. 4
Set 3 : 8 Contestants of the Miss World contest M,N,O,P,Q,R,S and T were being
evaluated in the final round of the contest. The evaluations were done on 2
aspects: Personality and Overall Thinking. After the evaluations were done they
were divided into 2 groups based on their performance.
1.
For each aspect, the top 2 performers and the bottom 2 performers formed
Group A, while the other 4 contestants formed group B.
2.
Group A based on Personality is M,P,Q and S ie these 4 people got ranks of
1,2,7 and 8 in Personality not in any order.
3.
Group B based on Overall Thinking is P,T,Q and O not in any order.
4.
M , T and O are the only contestants to get the same Rank in both the
aspects.
5.
R got Rank 4 in personality while N got Rank 2 in Overall thinking.
Q9 : If M and T got consecutive Ranks in Overall Thinking, who must have got Rank
7 in personality?
a.
M
b. P
c. Q
Page 133 of 204
d. S
Graphic Era Hill University, Dehradun
Q10 : If Q got the 1st Rank in Overall Thinking, how many possible positions can R
get in Overall thinking?
a.
1
b. 2
c. 3
d. 4
Q11 : If T and O did not get consecutive Ranks in Personality, what is the Rank
obtained by N in Personality?
a.
3
b. 5
c. 6
d.
Cannot
be
Determined.
Q12 : If S got the 7th Rank in Personality and N got a better Rank than M in Overall
thinking, which among the following cannot get Rank 1 in Overall Thinking?
a.
S
b. R
c. M
d.
Cannot
be
Determined.
Set 4 : A Group of 4 nations has to be selected out of 8 members of the security
council consisting of USA, France, Japan, India, China, Germany, UK and Sri Lanka.
While India, China, Japan and Sri Lanka are from Asia, the other countries are from
Rest of the World.
1.
Out of the 4 countries, at least 1 country each must be from Asia and Rest of
the World. Further, the group must include at least 1 from India and USA.
2.
If Sri Lanka is choosen, neither France nor Germany can be choosen.
3.
If China is choosen, exactly one from USA or UK has to be choosen.
4.
If Japan is choosen, India cannot be a choosen country.
Q13 : Which among the following cannot be an acceptable combination?
a.
Japan, Sri Lanka, USA, UK.
b.
India, China, USA, France.
c.
India, Sri Lanka, Germany, UK.
d.
Japan, China, USA, France.
Page 134 of 204
Graphic Era Hill University, Dehradun
Q14 : If India is one of the 2 countries choosen from Asia, how many possible
combinations exist?
a.
4
b. 7
c. 6
d. 5
Q15 : If there are 2 Countries choosen from Rest of the world, which of the
following combinations is not possible for the countries choosen from Asia?
a.
India, China
b. India, Sri Lanka
c. China, Sri Lanka
d.
Japan, China.
Q16 : If UK is the only country choosen from Rest of the world, which country is
definitely not choosen from Asia?
a.
India
b. Sri Lanka
c. China
d. Japan
Set 5 : 8 Businesses A,B,C,D,E,F,G and H are moving into a new 8 Storey building.
Any 2 Businesses will not locate on the same floor. Further, each Business will
occupy one entire Floor. The Floors of the new building are numbered 1 to 8 from
ground floor to the top floor.
Also, the following additional requirements must be met.
1.
B is to locate on the 6th Floor.
2.
D is to locate on the 4th Floor.
3.
E is to locate somewhere below D’s Floor.
Q17 : If F locates on a floor directly below E, F can locate on Floor number
a.
2
b. 1
c. 3
d. Cannot be
Deetermined
Q18 : If A locates on a Floor exactly 3 Floors above E, A must be on Floor number
a.
4
b. 5
c. 6
Page 135 of 204
d. 7
Graphic Era Hill University, Dehradun
Q19 : If A locates on a Floor somewhere above C and if C locates on a Floor
somewhere above H, which of the following must be definitely false?
a.
C locates on a Floor somewhere below D and above E.
b.
C locates on a Floor somewhere below E.
c.
A locates on a Floor somewhere above E and below D.
d.
A locates on a Floor somewhere above D.
Q20 : If F locates on a Floor somewhere above G and if G locates on a Floor directly
above H, which of the following could be true?
a.
G locates on Floor 7.
b.
H locates on a Floor somewhere above D.
c.
H locates on a Floor directly above B.
d.
G locates on Floor number 3.
Page 136 of 204
Graphic Era Hill University, Dehradun
Level 2
Blood Relations 1
Set 1 : There is a group of 5 persons A,B,C,D and E. In the group there is one
Badminton player, one Chess player and one Tennis player.
1.
A and D are unmarried ladies and do not play any game.
2.
No lady is a Chess player or a Badminton player.
3.
There is a married couple in the group of which E is the husband.
4.
B is the brother of C and is neither a Chess player nor a Tennis player.
Q1 : Which of the groups has only ladies?
a.
ABC
b. BCD
c. CDE
d. None of
c. D
d. E
c. D
d. None of
c. Brother
d. None of
These
Q2 : Who is the Tennis player?
a.
B
b. C
Q3 : Who is the wife of E?
a.
A
b. B
These.
Q4 : How is B related to E?
a.
Son
b. Brother-in-law
These.
Q5 : Looking at the photograph of a boy, a lady said, ” His mother is the wife of my
Father-in-law’s son. How is the boy related to the lady?
Page 137 of 204
Graphic Era Hill University, Dehradun
a.
Son
b. Nephew
c. Brother
d. Cannot be
Determined.
Q6 : If Rahul is the son of Kishore’s brother, then how is Rahul’s sister Neha related
to Kishore’s daughter?
a.
Sister
b. Aunt
c. Cousin
d. None of
These
Q7 : Lady A to another Lady B : “My Husband is your Brother-in-law.”
Lady B to Lady A : “Correct. Also, his Father-in-Law and my Father is the same
man”
How is Lady A related to the Husband of Lady B?
a.
Daughter
b. Sister
c. Sister-in_law
d. Cannot be
Determined.
Directions for Questions 8 to 10 :
P+Q means P is the daughter of Q.
P@Q means P is the son of Q.
P#Q means P is the brother of Q.
P$Q means P is the sister of Q.
P-Q means P is the father of Q.
P/Q means P is the mother of Q.
Q8 : If A$B-C-D#E, then by how many generations are A and E separated?
a.
0
b. 1
c. 2
Page 138 of 204
d. 3
Graphic Era Hill University, Dehradun
Q9 : If D/A/E$B#C, then how is A related to C?
a.
Mother
b. Father
c. Sister
d. Brother
Q10 : If E-A-B@D/C, then which of the following is false?
a.
E is C’s grandfather.
b.
D is A’s husband.
c.
B is C’s brother.
d.
None of the above.
Set 2 : 8 People were going out for an evening. They belonged to 3 generations of
a family and consisted of 3 married couples and 2 children.
1.
The first Car had A and her Father-in-law.
2.
D’s grandson was in Car 2 alongwith D’s daughter.
3.
A’s son E was not in the same car as his sister C.
4.
F is B’s Brother-in-law and is married to G.
5.
H and his spouse were in different Cars.
Q11: Which one of the following is a complete list of the people in Car 1?
a.
A,B,H,C
b. A,H,D,B
c. A,H,C,F
d.
Cannot
c. Neice
d. Daughter
be
Determined
Q12 : How is E related to G?
a.
Son
b. Nephew
Q13 : If no Couple is in the same Car, in which Car is F travelling?
a.
Car1
b. Car2
c. Either 1 or 2
as D
Q14 : How is G related to D?
Page 139 of 204
d. In the same Car
Graphic Era Hill University, Dehradun
a.
Daughter-in-law
b. Sister
c. Son
d. Daughter
Q15 : While pointing to a picture, A said to B:”This man is your Father’s brother’s
only sister’s mother’s husband.” How is B related to the man in the picture?
a.
Son
b. Grandfather
c. Grandchild
d. Cousin.
Q16 : Introducing Nithya to his daughter, Aman tells his daughter that she is his
brother’s wife’s mother-in-law’s daughter. How is Nithya related to Aman?
a.
Sister
b. Mother
c. Sister-in-law
d.
None of these
Q17 : If A happens to be the only daughter of B’s mother’s husband’s father’s wife,
how is A related to B?
a.
Sister
b. Mother
c. Neice
d. Aunt
Q18 : Pointing to a man in a photograph, a lady said to a boy :” His Father’s wife’s
only daughter-in-law is my sister.” How is the Man’s son related to the lady?
a.
Son
b. Nephew
c. Brother
d. None
of
These
Q19 : Rina’s Grandmother has only one child named Ajay whose mother’s
daughter-in-law Anjali has a daughter named Pallavi. How is Rina related to Pallavi?
a.
Aunt
b. Cousin
c. Sister
d. Mother
Q20 : If A is the brother of B, C is the mother of B, D is the daughter of A, E is the
sister of B, F is the brother of D, then who among the following is the Aunt of F?
a.
A
b. E
c. D
Level 2
Page 140 of 204
d. B
Graphic Era Hill University, Dehradun
Blood Relations 2 :
Set 1 : 4 couples A,B,C,D,E,F,G and H went to a party one day. While 2 of them
were housewives, the profession of the others were Doctor, Engineer, Lawyer,
Dentist, Teacher, and Banker. Further conditions are as follows :
1.
All of them belonged to 3 different generations of the same family with 1
couple in the first generation, 2 in the second generation and 1 in the third
generation.
2.
D, the Doctor is the Father of the Banker.
3.
G, the Dentist is the Brother-in-law of the Teacher.
4.
The Lawyer is married to one of the Housewives.
5.
H belongs to the first generation and is the Grandmother of the Lawyer.
6.
C, the Teacher does not belong to the first generation and is the sister of B’s
Mother.
7.
One of the Housewives is the Daughter-in-law of A, the Banker.
8.
E is not a Housewife.
9.
These are the only members in the family.
Q1 : How is G related to F?
a.
Father
b. Father-in-Law
c. Brother
Determined.
Q2 : Which of the following statements is definitely false?
a.
The Banker is the daughter of H.
b.
The Lawyer is the Doctor’s grandson.
c.
H is the mother of E.
d.
A and C are sisters.
Q3 : What is the profession of E?
Page 141 of 204
d.
Cannot
be
Graphic Era Hill University, Dehradun
a.
Banker
b. Teacher
c. Architect
d. None of
c. Son-in-law
d. Cannot be
These.
Q4 : How is B related to G?
a.
Son
b. Nephew
Determined.
Q5 : If Vishal is the brother of the daughter of Nilesh’s son, then how is Rohini, the
Wife of Nilesh related to Vishal?
a.
Mother
b. Grandmother
c. Sister
d. Aunt.
Q6 : Pointing to a lady in a photograph, Anand said:”Her Mother’s father is the
Father-in-law of my wife’s Husband”. How is Anand related to the lady’s Maternal
Grandmother?
a.
Nephew
b. Cousin
c. Son-in -law
d. Husband
Q7 : Mohan and Usha are married. Rani is the daughter of Usha’s brother. Vikas is
the Father-in-law of Mohan. How is Rani’s brother Nitin related to Vikas’s wife?
a.
Grandson
b. Son
c. Cousin
d.
Cannot
be
Determined.
Q8 : Pointing to a girl in a photo, Bijaya said:”Her brother’s Mother’s husband is the
Maternal Uncle of my son.”. How is the Lady’s mother related to Bijaya?
a.
Sister
b. Sister-in-Law
c. Aunt
d. Cousin
Q9 :
If Alok is the Father of Nisha’s Husband’s brother, then how is Maria, the
mother of Nisha’s husband related to Nisha’s daughter?
a.
Aunt
b. Sister-in-Law
Grandmother
Page 142 of 204
c. Mother
d.
Graphic Era Hill University, Dehradun
Q10 : A is the Paternal Uncle of B’s wife. C is the Husband of D. If D is the sister of
the Father of B’s wife, then How is A related to C?
a.
Son
b. Brother
c. Husband
d. None of
These.
Set 2 : Mr. Mansingh has 5 sons- Arun, Mahi, Rohit, Nilesh and Sourav and 3
daughters – Tamanna, Kuntala and Janaki. Three sons of Mr.Mansingh were born
first followed by 2 daughters. Sourav is the eldest child and Janaki the youngest.
Three of the Children are studying at Trinity School and three are studying at
St.Stefan. Tamanna and Rohit study at St.Stefan School. Kuntala, the eldest
daughter plays chess. Mansarover school offers Cricket only, while Trinity school
offers only chess. The children who are at Mansarover school have been born in
succession. Mahi and Nilesh are cricketers while Arun plays Football. Rohit who was
born just before Janaki, plays Hockey.
Q11 : Arun is the __________ child of Mansingh.
a.
2nd
b. 3rd
c. 6th
d. 5th.
c. Mansarover
d.
Cannot
be
c. Football
d.
Cannot
be
Q12 : Sourav is a student of which school?
a.
Trinity
b. St.Stefan
Determined.
Q13 : What game does Tamanna play?
a.
Cricket
b. Hockey
Determined.
Q14 : Which of the following pairs was not born in succession (ignore the order)
a.
Mahi and Nilesh
b.
Kuntala and Arun
c.
Rohit and Janaki
Page 143 of 204
Graphic Era Hill University, Dehradun
d.
Arun and Rohit.
Q15 : X does not have any siblings. If Y tells X that ”My Father is your Father’s
son”, then how is Y related to X?
a.
Brother
b. Father
c. Son
d. Cannot be Determined
Q16 : A said to B : “You are my Brother’s mother’s Husband’s only sister’s
husband’s only son’s sister.” How are A and B related?
a.
Brother
b. Sister
c. Siblings
d. Cousin
Directions for Questions 17 to 20 :
A+B means A is B’s Mother.
A-B means A is B’s Father.
AxB means A is B’s Brother.
A/B means A is B’s Sister.
Q17 : Which of the following means X is the Uncle of Z?
a.
YxX/U-Z
b. XxU+Y+Z
c. YxX/U+Z
d. XxU/Y-Z
Q18 : Which of the following means Z is the grandson of W?
a.
W-Y+Z/W
b. W-Y-Z
c. W+Y+ZxV
Q19 : If V-W+X/Y-Z, then how is V related to Z?
a.
Great Grandfather
b.
Great Grandson
c.
Grandfather
d.
Great Grandmother
Page 144 of 204
d. W/YxZ
Graphic Era Hill University, Dehradun
Q20 : If X/WxY+V-UxZ, then which of the following is not true?
a.
X is V’s Aunt.
b.
Y is U’s Grandmother.
c.
V is Z’s Father.
d.
W is V’s paternal Uncle.
Page 145 of 204
Graphic Era Hill University, Dehradun
Level 2
Mathematical Reasoning 1 :
Set 1 : 5 Teams qualified for the 2nd Round of World Cup Hockey held at Delhi. The
5 Teams were India, Australia, Netherlands, Pakistan and England. In the 2nd
round, each team would play exactly 1 match with every other team. For a win
each team gets 3 points, 1 point is awarded for a draw and no point for a loss. The
Following table gives the points table after the first 5 matches.
India
:
5 Points
Australia
:
0 Points
Netherlands :
2 Points
Pakistan
:
4 Points
England
:
1 Point
Further Information is as follows :
1.
Netherlands drew it’s match with England.
2.
India beat Australia in the 2nd match of the 2nd round.
3.
No team has played all it’s matches till the 5th match.
4.
India and Netherlands have played the same number of matches till the 5th
match.
Q1 : The number of matches lost by Netherlands till the 5th match is ?
a.
1
b. 2
c. 3
d. None of These
Q2 : The Team against which India has not played a match till the 5th Match is ?
a.
Australia
b. Netherlands
Page 146 of 204
c. Pakistan
d. England
Graphic Era Hill University, Dehradun
Q3 : The number of drawn matches in the first 5 matches is?
a.
4
b. 3
c. 2
d. 1
Q4 : The remaining matches of Pakistan will be against which countries?
a.
Australia, Netherlands .
b.
Netherlands, India.
c.
Australia, England.
d.
England, India.
Set 2 : 4 persons Mohit, Manohar, Prashant and Dinesh each had some initial
money with them. They were all playing bridge in a way that the loser doubles the
money of each of the other three persons from his share. They played 4 rounds and
each person lost one round in the same order as mentioned above. At the end of
the 4th round, each person had Rs.32,000/-.
Q5 : What was the amount that Mohit started with?
a.
60000
b. 34000
c. 66000
d. 28000
Q6 : What was the amount with Manohar at the end of the first round?
a.
68000
b. 72000
c. 64000
d. 80000
Q7 : Who had the lowest amount at any time during the tournament?
a.
Mohit
b. Manohar
c. Prashant
d. Dinesh
Q8 : What was the amount with Prashant at the end of the 2nd round?
a.
36000
b. 72000
c. 16000
d. 68000
Set 3 : A,B,C,D and E play a game of Cards. A says to B “If you give me 5 cards, I
will have as many cards as E has”. In reply, B says to A “If you give me 5 cards, I
will have as many cards as D has”. A and E together have 20 cards more than what
Page 147 of 204
Graphic Era Hill University, Dehradun
D and B have together. Also, B has 4 cards more than C and the total number of
cards with all of them is 201.
Q9 : How many cards does C have?
a.
34
b. 37
c. 31
d. 35
c. D
d. E
Q10 : Who has the maximum number of Cards?
a.
A
b. B
Q11 : The average number of Cards with D and E is how much more than the
average number of cards with B and C together?
a.
23
b. 22
c. 17
d. 12
Q12 : If E gives 6 cards to C, find the difference in the number of cards between B
and C?
a.
3
b. 2
c. 4
d. None of
These
Set 4 : Bhola and his three friends sell 4 different items on the Train from
Dehradun to Delhi. The 4 items are Apples, Chatpata, Newspaper and Colddrink.
Each person sells exactly one item but not in any order. Further each person has
earned some money. Further conditions are as follows:
1.
They together earned Rs.1200 on a particular day with Bhola earning Rs.300
more than the person who sold Newspaper.
2.
The Average earning of Ramesh and Chandu is Rs.250.
3.
Ramesh sells either Apple or Colddrink while Chandu does not sell
Newspaper.
4.
The person who earned the highest amount sold Colddrink while the person
who sold Chatpata earned Rs.100 less than Yogesh.
Page 148 of 204
Graphic Era Hill University, Dehradun
Q13 : Who much did the person who sold newspapers earn?
a.
500
b. 400
c. 300
d. 200
c. Yogesh
d. Bhola
c. Chatpata
d. Colddrink
Q14 : Who is the person who sold Chatpata?
a.
Ramesh
b. Chandu
Q15 : What is the item sold by Ramesh?
a.
Newspaper
b. Apple
Q16 : What is the maximum difference in the earnings of any 2 people?
a.
200
b. 300
c. 400
d. 500
Set 5 : A vegetable vendor has a weighing balance where he keeps the vegetables
on one side and the weight on the other pan. He has exactly one weight of each
type and each of the weights is an integral value ie 1Kg, 2Kg and so on. He will not
be using multiple weighings in any case.
Q17 : If he has to weigh any integral value from 1 to 100 Kg. then the minimum
number of weights required will be?
a.
6
b. 7
c. 5
d. 8
Q18 : The highest weight used in the above case will be?
a.
32 Kg.
b. 16 kg.
c. 64 kg.
d. 24 kg.
Additional Data for Questions 19 and 20 :
He alters his weighing plans such that he can now put the vegetables on either side
of the balance ie the weights can be used alongwith the vegetables also.
Q19 : What will be the minimum number of weights now required to weigh any
integral value upto 40 kg.?
Page 149 of 204
Graphic Era Hill University, Dehradun
a.
3
b. 5
c. 2
d. 4
Q20 : What will be the maximum weight used in the above question?
a.
27 Kg.
b. 36 Kg.
c. 18 Kg.
Page 150 of 204
d. None of These
Graphic Era Hill University, Dehradun
Level 2
Mathematical Reasoning 2 :
Set 1 : The Indian team for the tour of Australia was being selected and the coach
was in a dilemma. He had to choose 2 players out of 5 players P,Q,R,S and T to
complete the team. The coach decided to look at the performance of each of these
players in the last match. Further conditions are as follows:
1.
The runs scored by each player in the last match was a prime number
between 1 to 20.
2.
No 2 players scored the same number of runs in the last match. Further the
difference between the runs scored by any 2 players is at least 2.
3.
P was one of the 2 players to score in single digits.
4.
R scored 2 runs more than S. Also, there was at least 1 player who scored
more than R.
5.
There were exactly 2 pairs of players, the difference between whose scores
was 8 and Q was a player in one of those pairs.
Q1 : What is the runs scored by the player who made the highest score amongst all
these 5 players in the last match?
a.
15
b. 17
c. 19
d.
Cannot
be
Determined.
Q2 : If T scored 17 runs in the match, then which among the following cannot be
the runs scored by any of the players in the match?
a.
3
b. 5
c. 7
d.
Determined.
Q3 : How many runs did S score in the last match?
a.
5
b. 11
c. 13
Page 151 of 204
d. 17
Cannot
be
Graphic Era Hill University, Dehradun
Q4 : If the highest score was 19, then what could be the highest value of the single
digit score in the match?
a.
5
b. 3
c. 2
d. 7
Q5 : If the highest score was 17, find the average score of the 5 players in the
match?
a.
10.6
b. 9.8
c. 10.2
d. None of These
Set 2 : The students in the 3 sections of PKG in Polykids were being evaluated for
their performance. There were 3 sections in PKG called Sections A, B and C.
1.
The Ratio of the number of boys across the sections A,B and C is 1:1:2.
2.
There were 30 girls in Section B and the ratio of the total number of boys
and girls in the 3 sections is 3:5.
3.
If 10 girls are transferred from Section A to Section C, the number of girls in
both the sections will be the same.
4.
The total number of boys is 48 less than the number of girls.
Q6 : What is the total strength of the 3 PKG sections in the school?
a.
120
b. 192
c. 72
d. None of These
Q7 : If 5 girls from Section B leave the School, find the ratio of the number of girl
students in Sections A and B?
a.
2:1
b. 5:11
c. 11:5
d. 11:6
Q8 : What is the Ratio of the number of Boys to Girls in Section C?
a.
48:35
b. 2:3
c. 2:1
d. 6:5
Q9 : What is the number of Boys in Section B in Polykids?
a.
24
b. 36
c. 18
Page 152 of 204
d. 21
Graphic Era Hill University, Dehradun
Q10 : What is the average of the number of students in Sections A and C ?
a.
162
b. 67
c. 81
d. 64
Set 3 : A Punter placed a bet of 100/- each on the happening of three different
events A,B and C. The returns to the Punter if the events happen will be as follows :
A
:
Money gets Doubled.
B
:
Money becomes 1.5 times.
C
:
Money gets returned.
The Punter will lose his money if events A or B do not happen but will get half of
the bet amount back even if event C does not happen.
Q11 : The maximum percentage return that the Punter can expect is ?
a.
50
b. 25
c. 30
d. 40
Q12 : If the Punter ends up in a situation of no profit no loss, then how many of the
events have definitely happened?
a.
1
b. 2
c. 3
d.
Cannot
be
Determined
Q13 : If A is the only event that happened, what is the total amount that Punter will
get back?
a.
300
b. 250
c. 200
d. 350
Q14 : What is the minimum percentage return that Punter will get?
a.
16.66
b. 33.33
c. 25
d. 35
Set 4 : There are 3 bags, each containing 30 balls. Balls are of three colours-Red,
Green and Yellow. Each bag contains at least 1 ball of each colour. All balls of same
Page 153 of 204
Graphic Era Hill University, Dehradun
colour are of the same weight but each of them weighs different from the balls of
other colours. All balls are of same size.
Partial information regarding the number of balls of different colours in different
bags alongwith the total weights of the bags are tabulated below.
A
Red Balls
B
5
C
12
Green Balls
Yellow Balls
Total weight (in gms.)
510
450
440
Further Information is as follows:
1.
If all Yellow balls in Bag B were replaced with all Yellow balls in Bag C, then
there is no change in the total weight of any bag. Similarly, if all Green balls of Bag
A and all Green Balls of Bag C are interchanged, there is still no change in the
weight of any bag.
2.
The weight of each ball is an integer and the bags have negligible weight.
3.
There is only one bag which contains equal number of balls of each type.
4.
The weight of each Green ball is 15 gms.
Q15 : What is the ratio of the total number of Green, Yellow and Red coloured Balls
in that order?
a.
1:1:1
b. 2:3:2
c. 26:37:27
d. 27:36:26
Q16 : One Yellow ball is taken out of Bag A and put into Bag C and then one Red
ball is taken out of Bag C and put into Bag A. Find the total number of Red and
Page 154 of 204
Graphic Era Hill University, Dehradun
Green balls in bag A taken together as a percentage of the total number of Yellow
balls in Bag C?
a.
87.5
b. 127.3
c. 118.2
d. 81.3
Q17 : If the weights of yellow coloured and green coloured balls is interchanged,
then which bag will be the heaviest?
a.
A
b. B
c. C
d. A and C
Q18 : Which bag contains the least number of Green balls?
a.
A
b. C
c. Both A and C
d. Both A and B
Q19 : What is the ratio of the number of Red balls in Bag A to the number of Yellow
balls in Bag C?
a.
5:8
b. 1:2
c. 2:1
d. 8:5
Q20 : Which bag contains the largest number of Red balls?
a.
A
b. B
c. C
Determined
Level 1
Page 155 of 204
d.
Cannot
be
Graphic Era Hill University, Dehradun
Miscellaneous 1 :
Set 1 : 3 children Dan Bhai, Reyu and Mehu were asked question about whether
they had finished the drink given to them. One of them was given Milk, the other
Juice while the 3rd one was given Boost to drink. Under fear of being scolded each
of them gave 2 statements exactly one of which was false and the other was true.
Dan Bhai : I was not the last to finish. Reyu was given Milk.
Reyu : I was given Boost. Mehu was the first one to finish.
Mehu : I was the first one to finish. Dan Bhai was not given Milk.
Q1 : If Mehu was the first one to finish, then what drink was Reyu given?
a.
Milk
b. Boost
c. Juice
d. Cannot be
Determined.
Q2 : If Reyu was given Boost, at what position did Dan Bhai finish?
a.
1st
b. 2nd
c. 3rd
d. Cannot be
Determined.
Q3 : If Dan Bhai was given Milk, who finished the drink last?
a.
Reyu
b. Dan Bhai
c. Mehu
d. Cannot be
Determined.
Q4 : If Mehu finished ahead of Reyu, at what position did Dan Bhai finish the drink?
a.
1st
b. 2nd
c. 3rd
d. Cannot be
Determined.
Set 2 : There was an effort to study the relative importance that beneficiaries of 5
states assigned to 5 different development programmes implemented by their
governments. The programmes were Jawahar Rozgar Yojana(JRY), Indira Awas
Page 156 of 204
Graphic Era Hill University, Dehradun
Yozna (IAY), Mid-Day Meal (MDM), Rural Health Mission (RHM) and National Rural
Employment Guarantee Scheme (NREG). The level of dissimilarity between 2 states
is the maximum difference in the rank allotted by the 2 states to any of the 5
mentioned programmes. The following table indicates the rank order of the 5
programmes for each state.
Rank
Orissa
Bihar
Rajasthan
Kerala
Karnataka
1
JRY
MDM
IAY
NREG
NREG
2
RHM
JRY
MDM
IAY
JRY
3
MDM
RHM
JRY
RHM
MDM
4
NREG
IAY
NREG
JRY
RHM
5
IAY
NREG
RHM
MDM
IAY
Q5 : Which of the following states is least dissimilar to Orissa?
a.
Bihar
b. Rajasthan
c. Kerala
d. Karnataka
Q6 : Which of the following states is most dissimilar to Orissa?
a.
Bihar
b. Rajasthan
c. Kerala
d. Karnataka
Q7 : Three of the following 4 pairs of states have identical levels of dissimilarity.
Which is the odd one out?
a.
Kerala and Bihar
b.
Bihar and Karnataka
c.
Rajasthan and Kerala
d.
Karnataka and Rajasthan
Page 157 of 204
Graphic Era Hill University, Dehradun
Q8 : For which state is the difference in the Ranks allocated to NREG and RHM, the
maximum?
a.
Karnataka
b. Kerala
c. Orissa
d. Rajasthan
Set 3 : A cube of side 4 cms is taken and painted such that 2 opposite sides are
painted Red, 2 opposite sides are painted Green and 2 opposite sides are painted
Blue. It is now cut into 64 small Cubes.
Q9 : How many of the small cubes will have exactly 1 face painted?
a.
12
b. 16
c. 18
d. None of
These
Q10 : How many of the small cubes will have no face painted?
a.
6
b. 8
c. 12
d. 16
Q11 : How many small cubes will have all the 3 faces painted?
a.
10
b. 6
c. 8
d. 12
Q12 : How many small cubes will have 2 faces painted?
a.
12
b. 16
c. 20
d. 24
Set 4 : Ganesh Cultural Centre for promoting arts has appointed 3 instructors for
music, dance and painting. Music instructor takes sessions from 12 Noon to 4pm on
Monday, Thursday and Sunday. The sessions of Dance instructor are scheduled on
Tuesday, Thursday, Wednesday and Sunday between 10am and 2pm. The 9am to 1
pm slot on Tuesday, Friday and Thursday and also 2pm slot on Wednesday,
Saturday and Sunday is filled up by Painting Instructor. Each slot is of 2 hours
duration.
Q13 : On which slot is the Music and Dance slot simultaneously held?
Page 158 of 204
Graphic Era Hill University, Dehradun
a.
Tuesday, 2pm slot.
b.
Wednesday, 2pm slot.
c.
Monday, 12 Noon slot.
d.
Sunday, 2pm slot.
Q14 : Which of the following can be simultaneously conducted on Sunday, 12 Noon
slot to 1pm?
a.
Music, Dance
b.
Music, Painting
c.
Painting, Music, Dance
d.
Painting, Dance
Q15 : On which day(s) of a week the dance and painting sessions are
simultaneously held?
a.
Sunday and Wednesday
b.
Tuesday and Friday
c.
Tuesday and Thursday
d.
Only on Tuesday
Q16 : For how many days of the week is the Painting class held?
a.
6 days
b. 7 days
c. 5 days
d. 4 days
Set 5 : There are 6 people U,V,W,X,Y and Z in a family. Each of them was asked a
question about their relation with the other members in the family. There are 3
male and 3 female members in the family. Each person in response gave a
Statement which is either definitely true or definitely false. The statements they
gave are as follows:
U : I am the Grandfather of W.
V : X is my Wife.
Page 159 of 204
Graphic Era Hill University, Dehradun
W : Z’s statement is False.
X : I am the Mother of Z and W.
Y : Exactly 2 of us have given false statements.
Z : Exactly one of V or Y is true.
Incidently, it was found that 3 people gave correct statements while the other three
gave false statements.
Q17 : If Z’s statement is False, then how is U related to Z?
a.
Mother
b. Sister
c. Grandfather
d. Grandmother
Q18: If W is one of the persons to make correct statements, then how is Z related
to X?
a.
Son
b. Wife
c. Daughter
d.Cannot
be
Determined.
Q19 : If V’s statement is false, which of the following represents one of the
couples?
a.
U and X
b. Y and X
c. U and Y
d. X and V
Q20 : If Z’s statement is true, while U’s statement is false, how is V related to Z?
a.
Father
b. Son
c. Grandfather
Determined.
Level 2
Page 160 of 204
d.
Cannot
be
Graphic Era Hill University, Dehradun
Miscellaneous 2 :
Set 1 : 4 students went to the Kumar Sweet shop in Ballupur to have Snacks. Each
one of them had exactly 1 Snack(One piece each) out of 4 different Snacks
Samosa, Dhokla, Kachori and Rasmalai. The prices of one piece of these items
being 8, 10, 12 and 14/- in no particular order. No 2 people had the same snack.
When asked, each one made 2 statements exactly one of which was True while the
other was false.
Madan : I had a Dhokla costing 12/- . Neetu had the costliest item.
Neetu : The Snack Omita had was priced at 14/- . Pradeep had a Dhokla.
Omita : The average amount spent by all of us together was exactly same as the
price of one of the items. Pradeep did not have a Dhokla.
Pradeep : Omita had a Kachori. The price of a Samosa is not 14/-.
Q1 : What was the snack that Omita had?
a.
Kachori
b. Samosa
c. Rasmalai
d. Dhokla
Q2 : If Neetu had the item priced at 8/-, what is the cost of the Kachori?
a.
8
b. 10
c. 12
d. Cannot be
c. Neetu
d. Cannot be
Determined.
Q3 : The person who had Dhokla is ?
a.
Madan
b. Pradeep
Determined
Q4 : What is the average amount spent by Omita and Madan together?
a.
11
b. 12
c. 13
Page 161 of 204
d. 10
Graphic Era Hill University, Dehradun
Q5 : If Pradeep had Kachori, what is the average amount spent by Omita and Neetu
together?
a.
11
b. 10
c. 12
d. Cannot be
Determined
Set 2 : 6 people A,B,C,D,E and F have the following attributes:
1.
A is fairer than B but shorter than F.
2.
C is darker than D but taller than A.
3.
B is fairer than E but taller than F.
4.
F is fairer than A but taller than C.
5.
E is fairer as well as shorter than C.
6.
D is darker as well as shorter than E.
Q6 : Who is shortest among all?
a.
A
b. D
c. E
d.
Cannot
be
b. B
c. F
d.
Cannot
be
Determined.
Q7 : Who is the tallest?
a.
A
Determined.
Q8 : If Ranks are allotted with Rank 1 to the fairest person and so on till Rank 6 for
the darkest person, who will definitely get Rank 4?
a.
A
b. E
c. C
d. None of These
Q9 : Who gets the same Rank in Height as obtained by C in Fairness?
a.
D
b. A
c. B
Q10 : If A is taller than E, then who is the shortest amongst all ?
Page 162 of 204
d. C
Graphic Era Hill University, Dehradun
a.
E
b. C
c. D
d.
Cannot
be
Determined.
Set 3 : A Cube of side 5 cms is taken and is painted with Red, Green and Blue
colours. Each of the colours is painted on 2 adjacent faces of the cube. It is now cut
into 125 cubes each of side 1cm. Answer the following questions based on this
data:
Q11: How many smaller cubes will have exactly 2 faces painted and with the
colours Red and Green?
a.
9
b. 12
c. 18
d. 24
Q12 : How many of the smaller cubes will have only Blue colour on their faces?
a.
18
b. 21
c. 16
d. None of These
Q13 : How many smaller cubes will not have any colour on their faces?
a.
8
b. 12
c. 27
d. 36
Q14 : How many cubes will have 2 faces Green and 1 Face Red coloured?
a.
8
b. 24
c. 16
d. 1
Q15 : How many Cubes will not have Red colour on any of their faces?
a.
50
b. 125
c. 75
d. None of these
Set 4 : To Qualify for the SSC, a candidate has to take 2 papers in the Main exam
after qualifying the Prelims. Subjects have been categorized into 2 groups. Group A
subjects comprise English, Hindi, Maths, Psychology and Sociology while Group B
subjects consist of Geography, Sanskrit, Pub.Ad., History and Physics. The following
Table gives the Subject that was taken in Prelims by 10 candidates who qualified
for the Mains.
Page 163 of 204
Graphic Era Hill University, Dehradun
A : Hindi
D: Sanskrit
G: Geography
B: Socilogy
E: Hindi
H: Maths
C: Physics
F: Pub.Ad.
I: History
1.
J: Pub.Ad.
If a candidate has taken a group A subject in prelims, he or she cannot
take any subject from that group for Mains.
2.
If Hindi is one of the subjects choosen for Mains, Psychology is
definitely the 2nd subject.
3.
History and Sanskrit cannot be choosen together for Mains.
4.
If Physics is choosen in Mains, Geography is definitely choosen in
Mains.
5.
If Hindi is choosen in prelims, then Sanskrit has to be definitely
choosen in Mains.
6.
If Physics is a subject in prelims, then Maths is definitely choosen in
Mains.
Q16 : How many of the 10 candidates definitely cannot choose Sociology as a
subject in the Mains?
a.
3
b. 4
c. 5
d. 6
Q17 : In how many different ways can E select the 2 subjects for the Mains Exam?
a.
2
b. 3
c. 4
d. 5
Q18 : If a candidate decides to choose Geography as one of the subjects in Mains,
in how many different ways can the 2nd subject be choosen?
a.
9
b. 1
c. 7
d. 8
Q19 : Who among the following will definitely need to choose Maths as one of the
subjects?
Page 164 of 204
Graphic Era Hill University, Dehradun
a.
E
b. C
c. D
d. J
Q20 : If all the conditions are done away with, in how many different ways can a
candidate select the 2 subjects?
a.
90
b. 10
c. 45
Level 2
Page 165 of 204
d. None of These
Graphic Era Hill University, Dehradun
Coding Decoding, Series Completion, Direction Sense 1:
Q1 : If ALPHA is coded as BJSPF in a certain language, how is THETA coded in the
same language?
a.
UBFIS
b. BVHLY
c. UBHYL
d. BLYHV
Q2 : If Water means Wind, Wind means Stone, Stone means Mud, Mud means Sky
and Sky means Rain, then where will birds fly as per this language?
a.
Sky
b. Rain
c. Mud
d. None of These
Q3 : In a certain language, ‘Sun shines brightly’ is written as ‘ba lo sul’, ‘Houses are
brightly lit’ as ‘kado udo ari ba’ and ‘Light comes from Sun’ as ‘dapi kup lo nro’ ,
what words will be written for “sun’ and ‘brightly’?
a.
Lo,Ba
b. Ba,Lo
c. Sul, Lo
d. Ba, Sul.
b. 72
c. 64
d. 74
b. 4508
c. 5008
d. 6004
b. S
c. M
d. P
Q4 : 23,33,50,?,105,143
a.
63
Q5 : 14,30,124,750,?
a.
6008
Q6 : B,D,I,?,I
a.
K
Directions for Questions 7 and 8 :
There is an Alphanumeric Series given as below. Answer the questions on the basis
of this series.
A37B719C9D5DKL6R6S7S
Page 166 of 204
Graphic Era Hill University, Dehradun
Q7 : How many letters are immediately preceded and immediately followed by the
same digit?
a.
1
b. 2
c. 3
d. 0
Q8 : How many digits are immediately preceded as well as followed by the same
letter?
a.
0
b. 1
c. 2
d. 3
Directions for Questions 9 to 11 :
A word arrangement machine, when given a particular input rearranges it following
a particular rule. Following is the illustration of the input and the steps of
arrangement:
Input : She was interested in doing art film.
Step 1 : art she was interested in doing film.
Step 2 : art was she interested in doing film.
Step 3 : art was in she interested doing film.
Step 4 : art was in film she interested doing.
Step 5 : art was in film doing she interested.
Step 5 is the last step of the given input.
Q9 : Which of the following will be the last step for the input given below?
Input: He is going out to search air.
a.
Out is air to going search he.
b.
Out is air to search going he.
c.
Search he out is air to going.
Page 167 of 204
Graphic Era Hill University, Dehradun
d.
None of the above.
Q10 : If Step 2 of an input is “not is the casino considering legal action”, which step
is “not is casino action legal the considering”?
a.
Step 3
b. Step 6
c. Step 4
d. None of These
Q11 : How many steps will be required to get the final output from the following
input?
Input: Father needs to check on the boy.
a.
4
b. 5
c. 6
d. 3
Directions for Questions 12 to 13 : Mahesh started from his office and walked 5
Kms North. Then he took a right turn and walked for 10 Kms. He again turned right
walked for 8 Kms and stopped.
Q12 : In which direction is he with respect to the starting point?
a.
South
b. South-West
c. South-East
d. North
c. 9
d. 12Km
Q13 : How far is he from the starting point?
a.
13 Km
b.√109
Q14 : If ANGUISH is coded as BIOTHJV in a certain language, how is DEPENDS
coded in the same language?
a.
ETFEQOF
b. QOFETFE
c. FOEQTFE
d. None of These
Q15 : If + means /, / means x, x means – and – means + in a certain language,
then find the value of 2/3-5x7+9
a.
-76/3
b. 92/9
c. 76/9
Q16 : Complete the Series 0, 8, 24, 48, 120, ?
Page 168 of 204
d. None of These
Graphic Era Hill University, Dehradun
a.
137
b. 149
c. 168
d. 173
Q17 : If PONDER is coded as 1615144518 in a certain language, how is SUCCINT
coded in the same language?
a.
19203491422
b. 19234391423
c. 23194391423
d. 19213391420
Q18 : Seema and Rekha are standing at a distance of 10m facing East. They both
walk for 5m. Seema takes a right turn walks for 3m and stops while Rekha takes a
left turn walks for 2m and stops. How far are they from each other?
a.
5m
b. 15m
c.10m
d.
Cannot
be
Determined.
Q19 : If Blue is called Red, Red is called Yellow, Yellow is called Brown, Brown is
called Black and Black is called Orange in a certain language, then what will be the
colour of Blood in this language?
a.
Blue
b. Red
c. Yellow
d. Brown.
Q20 : If “Tu Hum Ne” means “we are good”, “Hum se Pit” means “There we go” and
“Pit so Wak” means “is There Rain”, then which word in the language means “go”?
a.
Pit
b. Hum
c. se
Determined.
Page 169 of 204
d.
Cannot
be
Graphic Era Hill University, Dehradun
Level 2
Coding Decoding, Series Completion, Direction Sense 2:
Directions for Questions 1 to 2 :
Certain words are coded in a language below. You need to understand the basis of
coding and answer the questions asked.
Leap :
4132
Pole
:
7143
Lazy :
1256
Year
5328
:
Q1 : What will be the code for Reap in the language?
a.
6748
b. 6342
c. 8243
d. 6834
Q2 : Which of the following will not be used in coding “PLEA”?
a.
5
b. 1
c. 2
Page 170 of 204
d. 3
Graphic Era Hill University, Dehradun
Directions for Questions 3 to 5:
Find the odd one out
Q3 : 143, 221, 321, 437, 667
a.
321
b. 221
c. 437
d. 667
b. 27
c. 2125
d. 46656
b. 19
c.
d. None of These
Q4 : 1,4,27,256,2125,46656
a.
4
Q5 : 2, 5, 9, 19, 39, 75
a.
5
Q6 : Amit’s house faces south direction. He goes straight, takes a right turn, then
left and again right to reach his School. If he starts from his school and reaches
home, which of the following could be the sequence followed?
a.
Straight,Right,Right,Right
b.
Straight,Left,Right,Left
c.
Straight,Right,Left,Right
d.
None of These
Q7 :
Bhaskar starts from his office facing the West direction, travels for 7kms,
takes a left turn to travel 5 kms, again takes left to travel 3 kms, takes a 45degree
turn to his right and stops. In which direction is he facing?
a.
East
b. South-West
c. North-West
d.
South-
East
Directions for Questions 8 to 11:
A number arrangement machine when given a particular input rearranges it
following a particular rule.
Page 171 of 204
Graphic Era Hill University, Dehradun
Input : 245,316,436,519,868,710,689
Step 1 : 710,316,436,519,868,245,689
Step 2 : 710,316,245,519,868,436,689
Step 3 : 710,316,245,436,868,519,689
Step 4 : 710,316,245,436,519,868,689
Step 4 is the last step for the given input.
Q8 : If the given input is 655,436,764,799,977,572,333, which of the following step
will be 333,436,572,655,764,799,977?
a.
3rd
b. 2nd
c. 4th
d. None of these
Q9 : How many steps will be required to get the final output from the following
input 544,653,325,688,461,231,857?
a.
6
b. 5
c. 4
d. None of These
Q10 : Step 3rd for an input is 432,433,542,666,734,355,574, what will be the first
step for the input?
a.
666,542,432,734,433,574,355
b.
542,666,734,432,433,574,355
c.
355,574,433,432,734,666,542
d.
Cannot be Determined.
Q11 : What will be the 3rd step for the Input 653,963,754,345,364,861,541?
a.
541,345,754,963,364,816,653
b.
541,345,364,653,963,754,861
c.
541,345,364,963,754,861,653
d.
541,345,364,653,861,754,963
Page 172 of 204
Graphic Era Hill University, Dehradun
Directions for Questions 12 to 14: Complete the series
Q12 : 1, 2, 6, 21, 88, ?
a.
245
b. 445
c. 315
d. 513
b. 14
c. 12
d. 8
c. 17
d. None of These
Q13 : 97,63,18,?
a.
9
Q14 : 0, 2, 3, 5, 8, 10, 15, ?
a.
16
b. 23
Directions for Questions 15 and 16 : In a certain language “we will win” means
“me tu hum”, “win and lose” means “jo hum se” and “we lose all” means “jo tu bol”
Q15 : Which word in the language stands for will?
a.
me
b. tu
c. hum
d.
Cannot
be
Determined
Q16 : How will “all and win” be coded in that language?
a.
Jo hum tum
b. hum se me
c. bol me se
d. bol hum se
Q17 : If hands means neck, neck means nose, nose means mouth and mouth
means leg in a certain language, what do we walk with in this language?
a.
Neck
b. Mouth
c. Leg
d. Nose
Q18 : 2 friends A and B are standing one day in the morning with A facing the
North direction. If A and B are also facing each other, then the shadow of B will fall
a.
To his Right
b. In Front
c. To his Left
Page 173 of 204
d. None of These
Graphic Era Hill University, Dehradun
Q19 : If POUND is coded as OQNPTVMOCE, then how will DOLLAR be coded in the
same language?
a.
ABNPKMKAZBQS
b. ABPNKMAKZBQS
c. CENPKMKMZBQS
d.
None of These
Q20 : Complete the Series 26,99,226,399,626 ?
a.
799
b. 699
c. 899
d. None of These
Sectional Tests for Practice
Section Test 1 :
Directions for Questions 1 to 4 :
The Captains of the 4 Teams participating in the Asia Cup Tournament have
assembled for a group photograph. The Teams are India, Pakistan, Sri Lanka and
Bangladesh. They are wearing their national caps which are Blue, Green, Maroon
and Orange in colour not in any order.
Further information is as follows :
1.
The Indian captain is 2nd to the left of the person with the Blue coloured cap.
2.
The person at the extreme left of the arrangement is wearing the green
coloured cap.
3.
The Pakistan Captain and the Indian captain are not at adjacent positions.
4.
The Bangladesh captain is wearing the Orange coloured cap.
5.
The Srilankan Captain has the person wearing blue coloured cap seated to
his immediate right.
Q1 : The captain of which country is wearing the Green coloured cap?
a. India
b. Pakistan
c. Sri Lanka
d. Bangladesh
Q2 : The person to the immediate right of the Indian captain is wearing the cap of
which colour?
a. Red
b. Green
c. Blue
Page 174 of 204
d. Maroon
Graphic Era Hill University, Dehradun
Q3 : If the Tournament Director sits exactly in the middle, then what will be the
position of the Sri Lankan captain counting from the rightmost end of the
arrangement?
a. 2nd
b. 3rd
c. 4th
d. 5th
Q4 : What coloured cap is the Indian captain wearing?
a. Orange
b. Red
c. Green
Q5 : Complete the Series
a. 963
d. Blue
15, 105, 315, ?
b. 729
c. 693
d. None of These
Q6 : Showing a photograph of a man a lady said to a girl, “He is the Father of your
Father’s wife’s only brother’s son.” How is the girl’s mother related to the Man’s
wife?
a. Sister
b. Aunt
c. Sister-in-law
d. Daughter
Q7 : 8 people are sitting around a circular table all facing inwards. A is directly
opposite H while G is 2nd to the right of H. While B is sitting between H and G, E is
sitting between A and G. If A is facing the South direction, then which direction is G
facing?
a. South-West
b. West
c. North-East
d. North-West
Q8 : Choose the wrong number in the series 105, 85, 60, 30, 0, -45, -90?
a. 30
b. 0
c. -45
d. 60
Q9 : A is fairer than B but not as fair as D. D is darker than C who in turn is darker
than F. If F is fairer than E, then who among them is the fairest?
a. E
b. C
c. F
d. D
Directions for Questions 10-12 :
In the questions given below, a group of digits are represented by letters. The
questions have a word or a number which is represented using the series provided.
You have to find out which of the four combinations correctly represent the digit or
the number provided?
Digits :
9
2
1
7
5
3
6
4
8
Page 175 of 204
Graphic Era Hill University, Dehradun
Letters : B
V
M
L
D
P
A
F
R
Further conditions are as follows :
1.
If the first and the last digits are odd, the code for the last digit is same as
that for the first digit.
2.
If the first and the last digits are even, the code for the first digit is same as
that for the last digit.
3.
If the first digit and the last digits are odd and even digits not in any order,
the code for the entire set is written in the reverse order.
Q10 : 175364
a. MLDPAF
b. LDPAMF
c. FAPDLM
d. None of These
b. BMDLDP
c. BMDRAB
d. MDBRAM
Q11 : 915861
a. BMDMVB
Q12 : Which of the following will be represented by FALL?
a. 4677
b. 7764
c. 7674
d. 4776
Directions for Questions 13-15 :
In a family, there are 8 people L,M,N,O,P,Q,R and S across 3 generations. There
are 2 married couples among them. Further conditions are as follows:
1.
2.
3.
4.
P is the grandmother of O and the Mother-in-law of S.
L, the brother of N is the only Maternal Uncle of O.
Q is the mother of R.
M is the father of N.
Q13 : If the children in the 3rd generation are not counted, exactly how many male
members are there in the family?
a. 3
b. 4
c. 5
d. Cannot be determined
c. Daughter
d. Cannot be Determined.
Q14 : How is R related to O?
a. Nephew
b. Neice
Q15 : If Y is the brother of P, how is N related to Y?
a. Neice
b. Nephew
c. Sister
Page 176 of 204
d. Cannot be Determined
Graphic Era Hill University, Dehradun
Q16 : Complete the Series
BDG
KPV ?
a. CKS
b. CKT
c. CLT
d. CLS
Q17 : If Cycle is called Bus, Bus is called Train, Train is called Ship and Ship is
called Plane, which of the following runs on rails as per the language?
a. Train
b. Bus
c. Ship
d. Cycle
Q18 : I walked in the North direction for 20 m, turned left to walk 5m again turned
left walked for 25 m and stopped. How far am I from the starting point?
a. 10 m
b. 5.75 m
c. 20 m
d. 5√2 m
Q19 : If ACCOLADE is coded as BEFSQGKM in a certain language, how will
DECREASE be coded in the same language?
a. EFESFBTF
b. EGFVJGZM
c. EFFVJGZM
d. EGFVIHZM
Q20 : In a certain language, “Green Grass Everywhere” is written as ‘dik pa sok’
and ‘cow eats grass’ is written as ‘nok ta pa’. How is Cow written in that code
language?
a. nok
b. ta
c. nok or ta
Page 177 of 204
d. None of These
Graphic Era Hill University, Dehradun
Section Test 2 :
Directions for Questions 1 to 4 :
5 leaders of the SBP were scheduled to address a rally for the elections. The leaders
were P,Q,R,S and T and would address as per the following conditions:
1.
T will not be the last one to address.
2.
R being the chief will address at least before 2 other leaders.
3.
There will be exactly 1 leader who will address the gathering between the
address of Q and S.
4.
P will not be the first one to address the rally.
Q1 : If Q is the first one to address, who will be the 4th one to address the rally?
a. S
b. T
c. R
d. P
Q2 : If S is the last one to address the rally, in how many possible ways can the 5
leaders address the rally?
a. 1
b. 3
c. 4
d. 2
Q3 : If R is the last one to address the rally, which among the following statements
is definitely true?
a.
Q will address the rally before S.
Page 178 of 204
Graphic Era Hill University, Dehradun
b.
S will address the rally before P.
c.
Exactly one from Q or S will address the rally before P.
d.
R will address the rally before P.
Q4 : If T is the 3rd one to address the rally, who will be the last one to address the
rally?
a. P
b. Q
c. S
d. R
Q5 : In a certain code “GUIDE” is written as “49132” and “BEAM” is written as
“8257”. How is “IMAGE” written in that code?
a. 17542
b. 17452
c. 12745
d. 15472
Q6 : If D is the only brother of the wife of the Father of A’s son, how is D related to
A?
a. Brother
b. Brother-in-law
c. Uncle
d. Cannot be Determined
Q7 : Complete the Series
0,5,8,17,24,?
a. 35
b. 33
c. 37
d. 34
Q8 : Complete the Series ZA , XC, UF, ?
a. QJ
b. PJ
c. QK
d. None of These
Q9 : A man starts from his house that faces the South-East direction. He walks
straight for 8 m, takes a 45 deg turn in the anticlockwise direction and walks 5 m.
In which direction is he with reference to the starting point?
a. East
b. South-East
c. North
Directions for Questions 10-12 :
PXQ means Q is the maternal aunt of P.
P-Q means P is the mother of Q.
P+Q means P is the brother of Q.
Q10 : What does P+Q-RXS mean?
Page 179 of 204
d. North-West
Graphic Era Hill University, Dehradun
a.
S is the nephew of R.
b.
S is the grandson of Q.
c.
Q is the grandmother of S.
d.
None of These
Q11 : How will the relation P is the niece of S be represented?
a.
PXQ-R
b.
PXQ+S
c.
P-R+S
d.
None of These
Q12 : What does P+Q+R-S represent?
a.
S is the daughter of R.
b.
S is the neice of P
c.
Q is the uncle of S
d.
S is the nephew of Q
Q13 : City A is 5 Kms South-East of B. C is 7 kms north of D and 2 kms West of B.
In which direction of A is city D located?
a. South
b. West
c. South-West
d. North-East
Q14 : If “ARRANGE” is coded as “RRAAEGN” in a particular language, how will
“DECIDES” be coded in that same language?
a. ICEDDES
b. CEDIDES
c. DESICED
d. CEDISED
Q15 : There are exactly 5 people between A and E in a line. If there are exactly 3
people before A and also after E, find the number of people in the line?
a. 12
b. 13
c. 11
d. 10
c. 69
d. 72
Q16 : Complete the Series 4,9,17,35,?
a. 70
b. 71
Page 180 of 204
Graphic Era Hill University, Dehradun
Directions for Questions 17-19 : The students at ACE were confused about their
class schedule and therefore spoke to 3 of their teachers. Incidently, each Teacher
gave 2 statements out of which exactly 1 was true and the other was false. The 3
teachers were experts of Fluid Mechanics, Strength of Materials and Mathematics
not in any order.
Mr.Joshi : 1. I will not take the class at 10 am.
2. I am a Strength of Materials expert.
Mr. Amit : 1. Mr. Joshi is the expert of Fluid Mechanics.
2. I will take the class at 12 Noon.
Mr. Sanjay : 1. Mr.Amit will take the class at 12 Noon.
2. I am the Maths expert.
Q17 : If Mr.Joshi takes the class at 12 Noon, which area does Mr.Amit specialise in
?
a.
Strength of Materials
b.
Fluid Mechanics
c.
Mathematics.
d.
Cannot be Determined.
Q18 : If Mr. Amit takes the class at 12 Noon, who is the Fluid Mechanics expert?
a. Amit
b. Joshi
c. Sanjay
d. Cannot be Determined
Q19 : If Mr. Joshi takes the class at 10 am, when will the Maths class be scheduled?
a. Either 11 or 12.
b. 11 am
c. 12 Noon
d. Cannot be Determined
Q20 : In a certain code ‘MAIN’ is written as ‘9364’ and ‘DEAR’ is written as “8532’.
How is ‘MEND’ written in that code?
a. 9548
b. 9458
c. 9538
Page 181 of 204
d. 9528
Graphic Era Hill University, Dehradun
Section Test 3 :
Directions for Questions 1 to 4 :
In a triangular tournament of Hockey, 3 teams A, B and C were participating. Each
team played 4 matches each against the other 2 teams. A win would give 3 points,
a draw gives 1 point with no point being awarded for a lost match. The following
table gives the partial data of the points .
Team
Won
Lost
Drawn
A
4
3
1
B
Points
y
C
2
Total
8
x
z
Q1 : Find the total number of matches played?
a. 8
b. 24
c. 16
d. 12
c. 6
d. 5
Q2 : What is the value of x+y?
a. 8
b. 7
Q3 : What could be the maximum value of z?
a. 10
b. 11
c. 12
d. 13
Page 182 of 204
Graphic Era Hill University, Dehradun
Q4 : How many matches played between B and C were drawn?
a. 2
b. 4
c. 3
d. 5
Q5 : How many pairs of letters are there in the word “CREDIT” each of which has as
many letters between them in the word as in the English alphabet?
a. 0
b. 1
c. 2
d. 3
Q6 : In a certain code BROWN is written as CSPXO. How is PART written in that
code?
a. OBQU
b. QBSU
c. OZQS
d. RBTU
Q7 : A is a set of all natural numbers from 1 to 40. From this set, all the numbers
that are exactly divisible by 4 and/or has 4 as a digit in it are removed. How many
numbers will remain?
a. 27
b. 29
c. 28
d. 26
Q8 : Pointing to a boy, a lady said,”He is the son of my grandmother’s only child”.
How is the boy related to the lady?
a. Brother
b. Uncle
c. Nephew
d. Cannot be Determined
Q9 : Meghna drives 10 km towards south, takes a right turn and drives 6 km. She
then takes another right turn, drives 10 km and stops. How far is she from the
starting point?
a. 16 km
b. 6 km
c. 4 km
d. 12 km
Q10 : If P means X, Q means /, R means – and S means +, then find the value of
46 12 P 3 S 18 Q 9 = ?
a. 13.3
b. 14
c. 36.5
d. 16
Q11 : ‘VT’ is related to ‘QO’ in the same way as ‘MK’ is related to ?
a. HF
b. IG
c. RP
d. JG
Q12 : In a certain code KINETIC is written as TICDKIN. How is MACHINE written in
that code?
a. ENIGMAC
b. INEGMAC
c. INEGCAM
Page 183 of 204
d. ENIGCAM
Graphic Era Hill University, Dehradun
Q13 : There are 4 bags T,S,V and W each having different weight. Bag T is lighter
only than S. V is lighter than W and W is lighter than T. Which of the 4 bags is the
lightest?
a. S
b. W
c. T
d. V
c. 177
d. 189
c. 276
d. 213
Q14 : 9, 21, 45, 81, 129 , ?
a. 187
b. 199
Q15 : 12 , 16 , 32, 68, 132, ?
a. 196
b. 232
Q16 : ‘BF’ is related to ‘HL’ in the same way as ‘EI’ is related to ?
a. KO
b. KN
c. JN
d. JO
Directions for Questions 17-20 : The Pune team were participating in the IPL for
the 2nd time. They already had local players and wanted to now select 5 more
players. They decided to select 1WK, 2 Bowlers and 2 Batsmen.
The WKs were Mathew Wade and Adam Gilchrist.
The Bowlers were Brett Lee, Mitchel Johnson and Stuart Clark.
The Batsmen were Shaun Marsh, David Warner, David Hussey and Michael Hussey.
Further condition is as under:
1.
Mathew Wade and David Hussey cannot be selected together.
2.
Brett Lee is to be definitely selected.
3.
Exactly 1 from Shaun Marsh and Warner must be selected.
4.
Mitchel Johnson and David Hussey cannot be selected together.
Q17 : If David Hussey is selected, who has to be definitely selected?
a.
Micheal Hussey
b.
Stuart Clark.
c.
Shaun Marsh
Page 184 of 204
Graphic Era Hill University, Dehradun
d.
David Warner
Q18 : If Mathew Wade is selected, in how many ways can the 5 players be
selected?
a. 4
b. 2
c. 3
d. None of These
Q19 : Which among the following is not an acceptable combination of the 5 players
to be selected?
a.
Mathew Wade, Brett Lee, Mitchel Johnson, Shaun Marsh, Micheal Hussey
b.
Adam Gilchrist, Mitchel Johnson, Brett Lee, David Warner, Micheal Hussey
c.
Mathew Wade, Brett Lee, Mitchel Johnson, David Warner, Micheal Hussey
d.
None of These
Q20 : Who among the following cannot be selected together?
a.
Brett Lee, Mitchel Johnson
b.
Stuart Clark, Brett Lee
c.
Mitchel Johnson, Stuart Clark
d.
None of These
Page 185 of 204
Graphic Era Hill University, Dehradun
Section Test 4 :
Q1 : Pointing to the photograph of a lady, a girl said to a Man, “She is the wife of
the brother of your Maternal Grandfather’s only daughter. How is the lady in the
photograph related to the man?
a. Sister
b. Aunt
c. Cousin
d. Mother
Q2 : On a bright Sunday morning, the shadow of a man falls to his right. If he
takes a 45 deg turn to his right, which direction will he be facing?
a. South-West
b. South
c. West
d. Cannot be Determined
Q3 : Complete the Series 3,7,12,16,21,?
a. 23
b. 24
c. 25
d. 27
Q4 : If Red means Blue, Blue means Black, Black means Yellow, Yellow means
Brown and Brown means White in a certain language, what will the colour of Blood
be as per this language?
a. Blue
b. Red
c. White
d. Cannot be Determined
Q5 : If ARRANGEMENT is coded as ZUQOQFZNMFF in a particular language, how
will DETERMINE be coded in the same language?
a. CFFOSJDNQ
These
b. CFDOSJDNQ
Page 186 of 204
c. CFFDOSJDNQ
d. None of
Graphic Era Hill University, Dehradun
Q6 : If A means x, B means -, C means + and D means / in a particular language,
find the value of 5 A 10 B 15 C 20?
a. -5
b. 55
c. 45
d. None of These
Directions for Questions 7-10 :
5 children Ankit, Bimal, Chanda, Deepak and Elenor went to see an exhibition with
their Mothers. Their mothers were Priya, Queenie, Ruchi, Sheetal and Taru not in
any order. They went by Scooter, Bus, Car, Auto and Jeep not in any order.
1.
The child who went by Bus was neither Ankit nor Deepak.
2.
Priya’s child is neither Bimal nor Chanda.
3.
Elenor went by Scooter along with her mother Ruchi.
4.
Deepak is the son of Taru.
5.
While Ankit went by Jeep, Sheetal went by Car.
Q7 : Who is the child of Priya?
a. Bimal
b. Elenor
c. Ankit
d. Deepak
Q8 : Who among the following went by Auto?
a. Priya
b. Taru
c. Queenie
d. Sheetal
c. Ankit
d. Cannot be Determined.
c. Taru
d. Cannot be Determined
Q9 : Which child went by Bus?
a. Bimal
b. Chanda
Q10 : Who is the mother of Bimal?
a. Queenie
b. Sheetal
Q11 : In a certain code DIAMOND is written as EJBNPOE. How is ROUTINE written
in that code?
a. SPVOUJF
b. SUPVJOF
c. SPVUJOF
d. SPVFOJU
Q12 : Complete the Series A D F I K N P S U ?
a. W
b. X
c. Y
Page 187 of 204
d. Z
Graphic Era Hill University, Dehradun
Q13 : In a row of 40 students facing North, S’s position in the row is 15th from the
right end while R’s position in the row is 4th from the left end. If X is the number of
students before R and Y is the number of students after S in the row, find the value
of Y-X?
a. 11
b. 12
c. 13
d. 10
Q14 : Which alphabet will be the 50th in the series aababcabcd……..?
a. d
b. e
c. c
d. f
Q15 : If @ means x, & means /, % means + and $ means -, then find the value of
6 % 12 & 3 @ 8 $ 3?
a. 37
b. 35
c. 39
d. 33
Q16 : Pointing to a girl Suraj said, “She is the only daughter of my Father’s Son-inLaw”. How is the girl related to Suraj?
a. Neice
b. Cousin
c. Sister
d. Daughter
Q17 : ‘KN’ is related to ‘QT’ in the same way as ‘DG’ is related to ?
a. JN
b. IM
c. JM
d. IN
Directions for Questions 18-19 :
Q9K#P@3ENSAC*G$UM7FIV%4Z8Y
Q18 : If all the numbers are dropped from the above arrangement, which of the
following will be the seventeenth from the right end?
a. E
b. P
c. I
d. C
Q19 : Which of the following is the sixth to the left of the fifth to the left of V?
a. 3
b. A
c. N
d. S
Q20 : Complete the Series 15, 22, 36, 57, 85, ?
a. 120
b. 110
c. 150
Page 188 of 204
d. 90
Graphic Era Hill University, Dehradun
Section Test 5 :
Directions for Questions 1-4 :
8 People E,F,G,H,I,J,K and L are sitting in a circle for a meeting. All of them are
Engineers and belong to exactly one of CS, EC, EE, ME, CE, IN, BT and CH streams
not in any order. Further info. is as follows:
1.
F, the EE is sitting opposite to the CE.
2.
I, is 3rd to the right of F and belongs to the BT stream.
3.
The ME is sitting opposite J.
4.
L belongs to the EC stream and is directly opposite to G.
5.
K and H occupy the 2 places between the EC and the EE.
Q1 : How many arrangements are possible?
a. 2
b. 1
c. 3
d. 4
c. H
d. Cannot be Determined
Q2 : Who is sitting opposite I?
a. K
b. G
Q3 : Which among the following may never be true?
Page 189 of 204
Graphic Era Hill University, Dehradun
a.
The CH engineer is opposite the ME.
b.
The ME is 4th to the right of I.
c.
The CS is 2nd to the left of E.
d.
The IN is sitting adjacent to the ME.
Q4 : If the CS is to the immediate left of the EE, find the specialization of J?
a. IN
b. CH
c. CE
d. Cannot be Determined
Q5 : M is the brother of K. P is the sister of K. R is the husband of P’s Mother. How
is K related to R?
a. Son
b. Daughter
c. Sister
d. Cannot be Determined
Q6 : How many pairs of letters are there in the word BONDING each of which has
as many letters between them in the word as in the English alphabet?
a. 0
b. 1
c. 2
`d. 3
Directions for Questions 7-8 :
1.
P x Q means P is the brother of Q.
2.
P – Q means P is the sister of Q.
3.
P / Q means P is the Father of Q.
4.
P + Q means P is the Mother of Q.
Q7 : Which of the following means D is the nephew of R?
a. D x M / R
b. R – M / D
c. R – M / D x T
d. R x M + D
Q8 : Which of the following means ‘M is the maternal uncle of T’ ?
a. M x R + T
b. M x R / T
c. T x J – N / M
d. M – R + T
Q9 : If A is 2, B is 4, C is 16, what will be the value of D ?
a. 64
b. 32
c. 256
d. None of These
Page 190 of 204
Graphic Era Hill University, Dehradun
Q10 : I start from the front of my house, take a right turn walk for 10 m and then
take a left turn walk for 5 m and stop. If I am now facing the South direction, in
which direction does the front of the house face?
a. East
b. West
c. North
d. South
Q11 : Shankar’s income is more than that of Shekhar but is equal to that of
Santosh. Shekhar’s income is less than that of Suresh but more than that of
Raman. If Shankar’s income is less than that of Suresh, then whose income is the
highest?
a. Shekhar
b. Shankar
c.Santosh
d. Suresh
Q12 : Rani and Shyam have some Rupees. Ram told Shyam that if he gave him one
Rupee he would have exactly double of what Shyam had. Shyam told Ram that if
he (Ram) gave one rupee to him, both would have an equal amount. How many
Rupees Ram and Shyam have?
a.
5 and 3
b. 4 and 2
c. 7 and 5
d. 9 and 7
Directions for Questions 13-16 : 5 students appeared for a Test where the total
marks was 50. Each one got a different score in the Test. Incidently, the marks
obtained by all the 5 students were multiples of 8. Further Information is as follows
:
1.
None of them got a score less than 10.
2.
The average marks of A and E was a number which has an odd number of
factors.
3.
D got the highest marks among all the 5 students.
4.
The marks obtained by C was 75% of the marks scored by E.
Q13 : What is the average marks scored by all 5 of them together?
a. 32
b. 24
c. 30
d. 28
Q14 : Who among them scored the lowest marks?
a. A
b. B
c. C
d. E
Q15 : What is the marks scored by C?
a. 32
b. 40
c. 24
d. None of These
Page 191 of 204
Graphic Era Hill University, Dehradun
Q16 : Find the average of the marks scored by A and D ?
a. 40
b. 44
c. 48
d. 36
b. MNOPQ
c. MNPRS
d. None of These
b. 1024
c. 2048
d. 625
Q17 : Complete the Series
BC , DFG, HJKL, ?
a. MNPQR
Q18 : 1, 8, 81, ?
a. 256
Q19 : Looking at the photograph of a woman, a lady said to a boy, “she is the
daughter of my Husband’s Father-in-Law and your sister is the daughter of her
husband”. How is the boy related to the lady?
a. Son
b. Nephew
c. Cousin
d. None of These
Q20 : If “PROMOTE” is coded as 1618151315205, how will “SUCCEED” be coded in
the same language?
a. 192034554
b. 192035455
c. 192133554
Page 192 of 204
d. 192133544
Graphic Era Hill University, Dehradun
Section Test 6 :
Directions for Questions 1-4 :
Company ABC had arranged for a conference where 5 members of it’s top
management were to address the Media. The people were Akshat, Reyyansh,
Mehuli, Sarvesh and Alok and their designations were CEO, MD, VC, CGM and VP
not in any order.
Further information is as follows:
1.
Neither the CEO nor the MD would occupy extreme positions.
2.
Mehuli being the only lady in the group would occupy the middlemost
position.
3.
Alok is the VP and has the VC to his immediate right.
4.
There is a gap of exactly one between the CEO and the VC’s position.
5.
Reyyansh is to the extreme right of the arrangement.
6.
Akshat’s designation is MD.
Q1 : Who is the CGM of Company ABC?
a. Mehuli
b. Reyyansh
c. Sarvesh
Page 193 of 204
d. None of These
Graphic Era Hill University, Dehradun
Q2 : Who is at the 4th position from the right end of the arrangement?
a. Akshat
b. Sarvesh
c. Reyyansh
d. Alok
Q3 : What is the designation of Mehuli?
a. CGM
b. VP
c. CEO
d. VC
Q4 : Who among the following is at an extreme position?
a. Sarvesh
b. Mehuli
c. Akshat
d. Reyyansh
Q5 : Complete the Series 2,3,5,9,7,15,9,?
a. 19
b. 21
c. 20
d. 23
Q6 : If NAINITAL is coded as LAITNNAIN in a certain language, how will
DEHARADUN be coded in the same language?
a. NUDARAHED
b. DAEHRADUN
c. NUADRDEHA
d. None of These
Q7 : The front of my House faces the East direction. I walked out of the back door,
walked for 15 m took a 45 degree turn anticlockwise, walked for 5m, took another
turn in the anticlockwise direction, walked for 4m and stopped. How far am I from
the starting point?
a. 13 m
b. √109 m
c. 16 m
d. None of These
Q8 : Pointing to the photograph of a boy, a lady said to a girl “His Father’s wife’s
Mother-in-law is my mother and your grandfather’s daughter’s brother is also my
brother.”How is the boy related to the girl?
a. Brother
Determined
b. Nephew
c. Cousin
d.
Cannot
c. 400
d. None of These
Q9 : Find the missing term in the series?
121, 169, 189, ?, 529
a. 361
b. 441
Directions for Questions 10-14 :
Page 194 of 204
be
Graphic Era Hill University, Dehradun
A Cube of side 3 cms is taken and is painted Red on all it’s faces. It is now cut into
equal sized cubes of side 1cm.
Q10 : How many of the cubes will not have any face painted?
a. 6
b. 3
c. 9
d. 12
Q11 : How many of the cubes will have exactly 1 face painted?
a. 6
b. 3
c. 9
d. 12
Q12 : How many of the cubes will have at least 2 of it’s sides painted?
a. 12
b. 15
c. 20
d. None of These
Q13 : Find the ratio of the number of cubes with exactly 3 sides painted to those
with exactly 2 sides painted?
a. 2:3
b. 3:4
c. 3:2
d. 4:3
Q14 : Find the number of cubes with exactly 3 sides painted?
a. 4
b. 8
c. 12
d. 16
Q15 : Complete the Series aabb_caaabbcc_aa_bc_c
a. cbbc
b. abbc
c. abca
d. cbba
Q16 : If Red is called Green, Green is called Blue, Blue is called Yellow, Yellow is
called Violet and Violet is called Red, then what will be the first colour of the
rainbow VIBGYOR called in this language?
a. Violet
b. Red
c. Yellow
d. Red
Q17 : Looking at the photograph of a boy a man said, “I am the brother of the wife
of his Father’s Son-in-Law”. How is the boy related to the man?
a. Son
b. Brother
c. Cousin
d. Cannot be Determined
Q18 : If A@B means B is the brother of A, A$B means A is the Father of B and A%B
means B is the sister of A, then what does the following relation represent
P%Q@R$S
a.
S is the nephew of Q
Page 195 of 204
Graphic Era Hill University, Dehradun
b.
P is the uncle of S
c.
Q is the aunt of S.
d.
S is the neice of Q
Q19 : Reyyansh and Mehuli have certain number of pencils with them. Mehuli says
to Reyyansh, “I have 3 more than half of the number of pencils with you”. In turn
Reyyansh says, “I have 6 less than double the number of pencils with you.”. Find
the total number of pencils with them?
a. 12
b. 9
c. 15
d. 21
Q20 : The rate of burning of 2 candles is 3cm/hr and 2cm/hr respectively. When
allowed to burn for 3 hours, the remaining length of the first one is double that of
the other. If their initial lengths are x and y cms, then of the following represents
the correct relation between x and y?
a. x=2y
b. x=2y+3
c. x=2y+6
Page 196 of 204
d. 2y=x-3
Graphic Era Hill University, Dehradun
Section Test 7 :
Set 1 : Directions for Qs 1-4 : 5 girls Aashna, Bindu, Cindy, Deepa and Ekta were
participating in a Cookery show where each one prepared a dish to be displayed.
They were wearing aprons of Red, Blue, Yellow, Green and Brown colour not in any
order. As a part of the competition, each of them was given a different rank from 1
to 5. A lower rank would necessarily mean a better rank.
Further conditions are as follows:
1.
Deepa was wearing a green coloured apron and did not finish last.
2.
Bindu cooked at the 4th place and was ranked better than 2 others.
3.
Ekta was the first one to cook and her apron was neither Brown nor Red.
4.
None of the girls got the same rank as the position at which they cooked.
5.
Aashna was the last one to cook wearing a Blue apron.
6.
Cindy got a better rank than Bindu and cooked immediately after the person
wearing the Yellow apron.
7.
Aashna got a better rank than the person wearing the Red apron.
Q1 : Who was the person to cook at the 3rd position?
a. Deepa
b. Bindu
c. Cindy
Page 197 of 204
d. Aashna
Graphic Era Hill University, Dehradun
Q2 : Who was wearing the Red coloured Apron?
a. Ekta
b. Cindy
c. Deepa
d. Bindu
Q3 : Who was the person to get the first rank?
a. Ekta
b. Cindy
c. Deepa
d. Bindu
Q4 : How many persons had a better rank than Aashna?
a. 1
b. 2
c. 3
d. Cannot be Determined
Q5 : Looking at the photograph of a girl, a woman told a man, “I am the daughterin-law of your Father and her Father is the only Son-in-law of my Father”. How is
the man related to the girl in the photograph?
a. Father
b. Brother
c. Uncle
d. Cannot be Determined
Q6 : Town A is to the South-West of Town B and to the East of C. In which direction
of Town C is Town B located?
a. North
b. North-East
c. North
d. Cannot be Determined
Q7 : Complete the Series 11, 20, 38, 74, ?
a. 148
b. 146
c. 150
d. 152
Directions for Questions 8 and 9 : A man starts from his house that faces the
West direction, walks for 7 m, takes a right turn to walk 3 m takes another right
walks for 9 m and stops.
Q8 : In which direction is he with respect to the starting point?
a. North
b. North-East
c. East
d. North-West
Q9 : How far is he from the starting point?
a. 4 m
b. 5 m
c. 3 m
d. √13 m
Q10 : If BEING is coded as 2522181320 in a certain language, how will HUMAN be
coded in the same language?
a. 1906142612
b. 1907142613
c. 1906132613
d. 1906142613
Directions for Questions 11-13 : 3 people Vinod, Sanjay and Asgar were asked
about the work done by them. While one of them was the Electrician, the 2nd one
Page 198 of 204
Graphic Era Hill University, Dehradun
was the Painter while the 3rd one was the Mason. The Electrician was responsible for
the fittings, the Painter for the Painting work and the Mason for the wood work.
Exactly 2 of them had finished the work given to them.
When asked about their work each one gave 2 statements out of which exactly 1
was true and the other was false. Following are the statements given by them :
Vinod : Statement 1 : I am not the Mason.
Statement 2 : I have not finished my work.
Sanjay : Statement 1 : Asgar is the Painter.
Statement 2 : Only I have completed my work.
Asgar : Statement 1 : Vinod has finished his work.
Statement 2 : Sanjay is the Painter.
Q11 : Who among them is the Mason?
a. Sanjay
Determined.
b. Asgar
c. Vinod
d.
Cannot
be
Q12 : Who among them is yet to complete the work given to him?
a. Sanjay
Determined
b. Asgar
c. Vinod
d.
Cannot
be
c. Vinod
d.
Cannot
be
c. Painter
d.
Cannot
be
Q13 : Who among them is the Electrician?
a. Sanjay
Determined
b. Asgar
Q14 : What is the profession of Asgar?
a. Electrician
Determined
b. Mason
Q15 : If Meetu is the Mother-in-Law of Bijaya’s sister’s Husband, then how is Reyu,
the son of Bijaya related to Meetu’s husband?
a. Son
b. Grandson
c. Nephew
Page 199 of 204
d. Cousin
Graphic Era Hill University, Dehradun
Q16 : In a certain language 123 means “everyone is happy” 356 means “happy and
sad” and 289 means “day is bright”, which digit in the language means “is”?
a. 1
Determined
b. 2
c. 3
d.
Cannot
be
Q17 : If x is replaced by + , + by -, - by / , then what will be the value of the
expression 2x3+4-5 = ?
a. 5
b. 21/5
c. 21
d. 25
Q18 : 2 children are playing a game where the winning point is 60 m away from the
starting point. The first child takes a step of 3 m everytime but comes back 2 m
while the 2nd child takes a step of 4 m everytime but comes back 2 m. Who will
touch the 60 m mark for the first time?
a. 1st
Determined
b. 2nd
c. Both will reach together
d. Cannot be
Q19 : If 2 means 4, 4 means 6, 6 means 8 and 8 means 2 in a certain language,
what will be the cube root of 8 as per this language?
a. 8
b. 6
c. 4
d. 2
Q20 : How many pairs of alphabets are there in the word “JANUARY” which have as
many alphabets between them as in the English language?
a. 0
b. 3
c. 2
Page 200 of 204
d. 1
Graphic Era Hill University, Dehradun
Section Test 8
Directions for Questions 1 to 2 : Read the instructions carefully and answer
the questions that follow.
5 friends are sitting in a straight line for a photograph.
1.
2.
3.
A is 3rd to the left of B who happens to occupy a corner position.
The number of persons sitting on either side of E is the same.
D does not have A as his neighbour.
Q1 : What is the correct position of C?
a.
b.
c.
d.
2nd to the left of B.
To the immediate right of D.
2nd to the left of A.
To the immediate left of A.
Q2 : If A and B now exchange their places while C and E also do the same, which of
the following is true?
a.
b.
c.
d.
B is at an extreme position.
C is in the middlemost position.
A is 3rd to the right of E.
D is between B and E.
Q3 : A is taller than both B and D. E is taller than B but shorter than C. If C is taller
than A, then who among them is the tallest?
a.
A
b. E
c. D
d. C
Q4 : Looking at the photograph of a man a lady said, “He is the brother of the
daughter of my daughter-in-law”. How is the lady related to the Man in the
photograph?
a.
Grandmother
b. Mother
c. Sister
d. Aunt
Directions for Questions 5 and 6: Read the instructions carefully and
answer the questions that follow:
A,B,C,D,E and F are 6 children consisting of 3 boys and 3 girls and belonging to 3
different cities Delhi, Mumbai and Kolkatta. Further information is as under:
Page 201 of 204
Graphic Era Hill University, Dehradun
1.
Equal number of children belong to each city. Also, each city is represented
by both boys and girls.
2.
A and E belong to the same city but not Kolkatta.
3.
D is a girl and belongs to Mumbai.
4.
While B is a girl, F belongs to Kolkatta.
Q5: Which of the following pair represents 2 people from the same city?
a.
A and B
b. B and F
c. B and D
d. C and F
Q6 : Which of the following is true about A?
a.
Girl from Delhi
b. Boy from Mumbai
c. Boy from Delhi
d.Cannot be Determined
Q7 : Identify the next term in the series of Numbers
2, 6, 12, 20, 30 , ?
a.
40
b. 45
c. 42
d. 44
Q8 : The Front door of my house faces North. I walked out from the back door
which is exactly opposite the front door, walked straight for 3 kms, took a right turn
to walk 4 kms, took a left and walked 3 kms and finally took a right turn walked for
4 kms and stopped. How far am I from the starting point?
a.
12 kms
b. √12 kms
c. 10 kms
d. √15 kms
Q9 : If Deepak is the brother of the daughter of Mohan’s sister’s husband, how is
Mohan’s wife Sheela related to Deepak’s sister?
a.
Mother
b. Neice
c. Grandmother
d. Aunt
Directions for Questions 10 to 12 :
A Medical representative has to visit 5 companies out of A,B,C,D,E,F and G in a
particular day of the week. The visits have to start on Monday and end on Friday
and should be scheduled according to the following conditions:
1.
2.
3.
If he visits A, then he must definitely visit E during the week.
He must visit exactly one company out of C and G.
If he visits D on a day, he can neither visit B or F on the immediate next day.
Page 202 of 204
Graphic Era Hill University, Dehradun
Q10 : Which of the following is a possible list of the companies and the order in
which they were visited during the week?
a.
AECGD
b. ECDBF
c. ECBFG
d. AECFD
Q11 : Which of the following companies must be definitely visited during the week?
a.
A
b. B
c. C
d. E
Q12 : If he visits A on Monday , C on Wednesday and D on Thursday, then what is
the possible number of companies eligible to be visited on Friday?
a.
1
b. 2
c. 3
d. 4
Q13 : If CODING is coded as HOJEPD is a certain language, then how will PROGRAM
be coded in the same language?
a.
NQAROGR
b. NQBSPHS
c. NBSHPSQ
d. None of These
Q14 : Find the next number in the series 0,26,124,342, ?
a.
511
b. 216
c. 999
d. 728
Directions for Questions 15 and 16 :
There are 60 students in a class and each of them studies at least one of the 3
subjects ie physics, chemistry and maths. There are 5 students who study both
physics and chemistry and 7 students who study both maths and chemistry. There
are 3 students who study all the 3 subjects. Also, the number of students who
study Physics, Chemistry and Maths is 25 each.
Q15 : Find the number of students who study physics and maths only?
a.
6
b. 9
c. 3
d. 4
Q16 : Find the number of students who study Chemistry only?
a.
16
b. 15
c. 14
d. 13
Q17 : Looking at the photograph of a lady, a man said, “ she is the only daughter of
my wife’s mother-in-law.” How is the man related to the lady’s daughter?
a.
Father
b. Brother
c. Uncle
Page 203 of 204
d. None of These
Graphic Era Hill University, Dehradun
Q18 : I started from a point and walked north for 3kms, turned right and walked 5
kms, took a left turn to walk 5 kms and again turned right walked for 2 kms and
stopped. In which direction am I with respect to the starting point?
a.
East
b. North East
c. North
d. South East
Q19 : If DIGEST is coded as CJFFRU in a particular language, how will INSIST be
coded in the same language ?
a.
JMTHTS
b. HORJRU
c. HMRHRS
d. JOTJTU
Q20 : If “ale tu bin” means drink more water, “tu ni koh” means grow more trees
and “koh hum ne” means trees are good, then which word in the language
represents grow ?
a.
Koh
Determined
b. ni
c. tu
Page 204 of 204
d.
Cannot
be